Insurance Law Case Digests

Published on June 2016 | Categories: Documents | Downloads: 57 | Comments: 0 | Views: 479
of 65
Download PDF   Embed   Report

Comments

Content

INSURANCE LAW CASE DIGESTS
I. GENERAL PROVISIONS (Section 1) A. Origin of Insurance B. Laws Governing Insurance in the Philippines i. Insurance Code of 1978 ii. Civil Code (Articles 739, 2012, 2011, 2207) iii. Special Laws C. Insurance Contract (Section 2) i. Definition ii. Elements iii. Characteristics iv. Interpretation of Insurance Contracts A fire broke out in the motor launch ISLAMA. As a consequence of which, Francisco del Rosario and 33 others were forced to jump off the launch. This resulted in the death of Francisco and his beneficiary Remedios Jayme. Equitable insurance paid Simeon del Rosario, father of Francisco Php1000 pursuant to Sec.1 of Part 1 of the policy. On the day of receipt, Atty. Francisco wrote Equitable acknowledging the receipt of Simeon of the amount of Php1000 but informed the company that the amount is incorrect as Simeon was entitled to Php1,500, under Sec.2 Part 1 of the policy. Equitable referred the matter to the Insurance Commissioner who opined that the liability of the company is only Php1000. Thus, Equitable refused to pay. Subsequently, Atty. Francisco asked for Php3000 from Equitable. The company refused to pay. Hence a complaint for the recovery of the balance was instituted. Issue: How much should the indemnity be? Ruling: The CFI ruled that: On the face of the policy Exhibit "A" itself, death by drowning is a ground for recovery apart from the bodily injury because death by bodily injury is covered by Part I of the policy while death by drowning is covered by Part VI thereof. But while the policy mentions specific amounts that may be recovered for death for bodily injury, yet, there is not specific amount mentioned in the policy for death thru drowning although the latter is, under Part VI of the policy, a ground for recovery thereunder. Since the defendant has bound itself to pay P1000.00 to P3,000.00 as indemnity for the death of the insured but the policy does not positively state any definite amount that may be recovered in case of death by drowning, there is an ambiguity in this respect in the policy, which ambiguity must be interpreted in favor of the insured and strictly against the insurer so as to allow greater indemnity. Thus, del Rosario is entitled to Php3000. Since Equitable has already paid Php1000, a balance of Php2000 remains to be paid. SC upheld the ruling of the CFI for it is supported by the generally accepted principles of insurance, which enunciate that where there is an ambiguity with respect to the terms and conditions of the policy, the same will be resolved against the one responsible thereof. It should be recalled in this connection, that generally, the insured, has little, if any, participation in the preparation of the policy, together with the drafting of its terms and Conditions. The interpretation of obscure stipulations in a contract should not favor the party

Simeon del Rosario vs. The Equitable Insurance and Casualty Co Inc. (1963) Facts: On February 7, 1957, Equitable Insurance and Casualty Co., Inc., issued Personal Accident Policy No. 7136 on the life of Francisco del Rosario, alias Paquito Bolero, son of Simeon, binding itself to pay the sum of P1,000.00 to P3,000.00, as indemnity for the death of the insured. The provisions of the insurance policy pertinent to the case are as follows: Part I. Indemnity For Death If the insured sustains any bodily injury which is effected solely through violent, external, visible and accidental means, and which shall result, independently of all other causes and within sixty (60) days from the occurrence thereof, in the Death of the Insured, the Company shall pay the amount set opposite such injury: Section 1. Injury sustained other than those specified below unless excepted hereinafter. . . . . . . . P1,000.00 Section 2. Injury sustained by the wrecking or disablement of a railroad passenger car or street railway car in or on which the Insured is travelling as a farepaying passenger. . . . . . . P1,500.00 Part VI. Exceptions This policy shall not cover disappearance of the Insured nor shall it cover Death, Disability, Hospital fees, or Loss of Time, caused to the insured: . . . (h) By drowning except as a consequence of the wrecking or disablement in the Philippine waters of a passenger steam or motor vessel in which the Insured is travelling as a farepaying passenger; . . . . A rider to the Policy contained the following: IV. DROWNING It is hereby declared and agreed that exemption clause Letter (h) embodied in PART VI of the policy is hereby waived by the company, and to form a part of the provision covered by the policy.

EH 403 Page 1

⦿ 2010-2011 ⦿ 2011-2012 ⦿

INSURANCE LAW CASE DIGESTS
who cause the obscurity (Art. 1377, N.C.C.), which, in the case at bar, is the insurance company. . . . . And so it has been generally held that the "terms in an insurance policy, which are ambiguous, equivocal or uncertain . . . are to be construed strictly against, the insurer, and liberally in favor of the insured so as to effect the dominant purpose of indemnity or payment to the insured, especially where a forfeiture is involved," (29 Am. Jur. 181) and the reason for this rule is that the "insured usually has no voice in the selection or arrangement of the words employed and that the language of the contract is selected with great care and deliberation by expert and legal advisers employed by, and acting exclusively in the interest of, the insurance company" (44 C.J.S. 1174). Calanoc v. Court of Appeals, et al., G.R. No. L-8151, Dec. 16, 1955. . . . . Where two interpretations, equally fair, of languages used in an insurance policy may be made, that which allows the greater indemnity will prevail. (L'Engel v. Scotish Union & Nat. F. Ins. Co., 48 Fla. 82, 37 So. 462, 67 LRA 581 111 Am. St. Rep. 70, 5 Ann. Cas. 749). At any event, the policy under consideration, covers death or disability by accidental means, and the appellant insurance company agreed to pay P1,000.00 to P3,000.00. is indemnity for death of the insured. FIELDMEN’S INSURANCE CO. vs. VDA. DE SONGCO FACTS: Federico Songco owned a private jeepney. On September 15, 1960, he was induced by Fieldmen's insurance agent Benjamin Sambat to apply for a Common Carrier's Liability Insurance Policy covering his motor vehicle. He was issued a Common Carriers Accident Insurance Policy. On the next year, he renewed the policy by paying the annual premium. During the effectivity of the renewed policy, the insured vehicle collided with another car while being driven by Rodolfo Songco, a duly licensed driver and son of Federico (the vehicle owner). As a result, Federico Songco (father) and Rodolfo Songco (son) died, along with other passengers. A claim was filed but was denied by the insurance company on the pretext that what was insured was a private vehicle and not a common carrier. During the trial, it was declared by a witness that when insurance agent Benjamin Sambat was inducing Songco to insure his vehicle, the latter butted in saying, “Our vehicle is a private vehicle and not for passengers.” But the agent replied: “Regardless of whether your vehicle was an owner-type or for passengers, it could still be insured because our company is not owned by the Government. And the Government has nothing to do with our company.” The Court of Appeals rendered a decision in favor of the claimants. It held that where inequitable conduct is shown by an insurance firm, it is estopped from enforcing forfeitures in its favor, in order to forestall fraud or imposition on the insured. After Fieldmen's Insurance Co. had led the insured Songco to believe that he could qualify under the common carrier liability insurance policy, it could not, thereafter, be permitted to change its stand to the detriment of the heirs of the insured. The failure to apply the Doctrine of Estoppel in this case would result in a gross travesty of justice. ISSUE: Whether or not the insurance claim is proper? RULING: The fact that the insured owned a private vehicle, not a common carrier, was something which the company knew all along. In fact, it exerted the utmost pressure on the insured, a man of scant education, to enter into the contract of insurance. The Court of Appeals also held that since some of the conditions contained in the policy were impossible to comply with under the existing conditions at the time, the insurer is estopped from asserting breach of such conditions. The Supreme Court, in affirming the decision of the Court of Appeals, took judicial notice of the fact that nowadays, monopolies, cartels and concentration of capital, endowed with overwhelming economic power, manage to impose upon parties dealing with them cunningly prepared agreements that the weaker party may not change one whit, his participation in the agreement being reduced to the alternative of “take it or leave it” labelled since Raymond Saleilles as contracts by adherence (contrats d'adhesion), in contrast to those entered into by parties bargaining on an equal footing, such contracts (i.e. insurance policies & international bills of lading) obviously call for greater strictness and vigilance on the part of courts of justice with a view to protecting the weaker party from abuses. Citing the case of Qua Chee Gan vs. Law Union & Rock Insurance, "The contract of insurance is one of perfect good faith (uberima fides) not for the insured alone but equally so for the insurer; in fact, it is more so for the latter, since its dominant bargaining position carries with it stricter responsibility." Landicho vs. GSIS [G.R. No. L-28866 March 17, 1972] FACTS: On June 1, 1964, the GSIS issued in favor of Flaviano Landicho, a civil engineer of the Bureau of Public Works, stationed at Mamburao, Mindoro Occidental, optional additional life insurance policy No. OG-136107 in the sum of P7,900. xxx

EH 403 Page 2

⦿ 2010-2011 ⦿ 2011-2012 ⦿

INSURANCE LAW CASE DIGESTS
Before the issuance of said policy, Landicho had filed an application, by filing and signing a printed form of the GSIS on the basis of which the policy was issued. Paragraph 7 of said application States: 7. xxx I hereby agree as follows: xxx c. That this application serves as a letter of authority to the Collecting Officer of our Office thru the GSIS to deduct from my salary the monthly premium in the amount of P33.36, beginning the month of May, 1964, and every month thereafter until notice of its discontinuance shall have beenreceived from the System; . d. That the failure to deduct from my salary the month premiums shall not make the policy lapse, however, the premium account shall be considered as indebtedness which, I bind myself to pay the System; . e. That my policy shall be made effective on the first day of the month next following the month the first premium is paid; provided, that it is not more ninety (90) days before or after the date of the medical examination, was conducted if required." While still an employee of the Bureau of Public Works, Mr. Landicho died in an airplane crash on June 29, 1966. Mrs. Landicho, in her own behalf and that of her co-plaintiffs and minor children, Rafael J. and Maria Lourdes Eugenia, filed with the GSIS a claim for P15,800, as the double indemnity due under policy No. OG-136107. GSIS denied the claim, upon the ground that the policy had never been in force because, pursuant to subdivision (e) of the above-quoted paragraph 7 of the application, the policy "shall be ... effective on the first day of the month next following the month the first premium is paid," and no premium had ever been paid on said policy. The Lower Court decided in favor of the petitioner. GSIS appealed to the Supreme Court. ISSUE: WON the insurance policy in question has ever been in force, not a single premium having been paid thereon. RULING: Lower Court decision is sustained. (T)he language, of subdivisions (c), (d) and (e) is such as to create an ambiguity that should be resolved against the party responsible therefor — defendant GSIS, as the party who prepared and furnished the application form — and in favor of the party misled thereby, the insured employee. Indeed, our Civil Code provides: The interpretation of obscure words or stipulations in a contract shall not favor the party who caused the obscurity. 2 DELA CRUZ V. CAPITAL INS. & SURETY CO., INC. DEATH RESULTING FROM BOXING IS AN ACCIDENT – SINCE DEATH IS NOT A NATURAL OR PROBABLE RESULT OF BOXING. Facts: Eduardo de la Cruz, employed as a mucker in the Itogon-Suyoc Mines, Inc. in Baguio, was the holder of an accident insurance policy underwritten by the Capital Insurance & Surety Co., Inc., In connection with the celebration of the New Year, the Itogon-Suyoc Mines, Inc. sponsored a boxing contest for general entertainment wherein the insured Eduardo de la Cruz, a non-professional boxer participated. In the course of his bout with another person, likewise a nonprofessional, of the same height, weight, and size, This is particularly true as regards insurance policies, in respect of which it is settled that the " "terms in an insurance policy, which are ambiguous, equivocal, or uncertain ... are to be construed strictly and most strongly against the insurer, and liberally in favor of the insured so as to effect the dominant purpose of indemnity or payment to the insured, especially where a forfeiture is involved" (29 Am. Jur., 181), and the reason for this rule is the "insured usually has no voice in the selection or arrangement of the words employed and that the language of the contract is selected with great care and deliberation by experts and legal advisers employed by, and acting exclusively in the interest of, the insurance company." (44 C.J.S., p. 1174.) 3. The equitable and ethical considerations justifying the foregoing view are bolstered up by two (2) factors, namely: (a) The aforementioned subdivision (c) states "that this application serves as a letter of authority to the Collecting Officer of our Office" — the Bureau of Public Works — "thru the GSIS to deduct from my salary the monthly premium in the amount of P33.36." No such deduction was made — and, consequently, not even the first premium "paid" — because the collecting officer of the Bureau of Public Works was not advised by the GSIS to make it (the deduction) pursuant to said authority. Surely, this omission of the GSIS should not inure to its benefit. . (b) The GSIS had impliedly induced the insured to believe that Policy No. OG-136107 was in force, he having been paid by the GSIS the dividends corresponding to said policy. Had the insured had the slightest inkling that the latter was not, as yet, effective for non-payment of the first premium, he would have, in all probability, caused the same to be forthwith satisfied. WHEREFORE, the decision appealed from should be, it is hereby affirmed, with costs against the defendantappellant, Government Service Insurance System. It is so ordered. .

EH 403 Page 3

⦿ 2010-2011 ⦿ 2011-2012 ⦿

INSURANCE LAW CASE DIGESTS
Eduardo slipped and was hit by his opponent on the left part of the back of the head, causing Eduardo to fall, with his head hitting the rope of the ring. He was brought to the Baguio General Hospital where he expired the the following day. The cause of death was reported as hemorrhage, intracranial, left. Simon de la Cruz, the father of the insured and who was named beneficiary under the policy, thereupon filed a claim with the insurance company for payment of the indemnity under the insurance policy Defendant insurer set up the defense that the death of the insured, caused by his participation in a boxing contest, was not accidental and, therefore, not covered by insurance. Issue: Whether the death of Eduardo was accidental since he entered in the boxing contest voluntarily. Ruling: The terms "accident" and "accidental", as used in insurance contracts, have not acquired any technical meaning, and are construed by the courts in their ordinary and common acceptation. Thus, the terms have been taken to mean that which happen by chance or fortuitously, without intention and design, and which is unexpected, unusual, and unforeseen. An accident is an event that takes place without one's foresight or expectation — an event that proceeds from an unknown cause, or is an unusual effect of a known cause and, therefore, not expected. It may be mentioned in this connection, that the tendency of court decisions in the United States in the recent years is to eliminate the fine distinction between the terms "accidental" and "accidental means" and to consider them as legally synonymous. The generally accepted rule is that, death or injury does not result from accident or accidental means within the terms of an accident-policy if it is the natural result of the insured's voluntary act, unaccompanied by anything unforeseen except the death or injury. 3 There is no accident when a deliberate act is performed unless some additional, unexpected, independent, and unforeseen happening occurs which produces or brings about the result of injury or death. 4 In other words, where the death or injury is not the natural or probable result of the insured's voluntary act which produces the injury, the resulting death is within the protection of policies insuring against the death or injury from accident. In the present case, while the participation of the insured in the boxing contest is voluntary, the injury was sustained when he slid, giving occasion to the infliction by his opponent of the blow that threw him to the ropes of the ring. Without this unfortunate incident, that is, the unintentional slipping of the deceased, perhaps he could not have received that blow in the head and would not have died. The fact that boxing is attended with some risks of external injuries does not make any injuries received in the course of the game not accidental.

-Ty New Life Enterprises vs Court of Appeals By: Yin Oliveros FACTS:  Julian Sy and Jose Sy formed a partnership under the business name of New Life Enterprises. They were holding their business in a two-storey building in Lucena City.

 Julian Sy insured the stocks in trade of New
Life Enterprises companies. INSURANCE COMPANY Western Guaranty Corporation under three insurance

Reliance Surety and Insurance Co., Inc.

TYPE OF INSURANCE Fire Insurance Policy This policy was renewed Fire Insurance Policy This policy was also renewed.

AMOUNT 350, 000. 00

300, 000. 00

- There was an
additional insurance issued in the amount of 700, 000. 00 200, 000. 00 TOTAL: 1, 550, 000. 00

Equitable Insurance Corporation

Fire Insurance Policy



The building occupied by New Life Enterprises was gutted by fire caused by a faulty electrical wiring. According to the plaintiffs, the stocks in trade were inside said building and were thus burned.

 Julian Sy, together with an agent of Reliance
Insurance, filed his claim. To support his claim, he submitted a fire clearance, the insurance policies and the inventory of stocks. He further testified that the three insurance companies are sister companies, and as a matter of fact when he was following-up his claim with Equitable Insurance, the Claims Manager told him to go first to Reliance Insurance and if said company agrees to pay, they would also pay. Ultimately, the three insurance companies denied plaintiffs' claim for payment due to BREACH OF POLICY CONDITIONS.

EH 403 Page 4

⦿ 2010-2011 ⦿ 2011-2012 ⦿

INSURANCE LAW CASE DIGESTS
 Reliance
Surety and Insurance Company claimed that plaintiff violated Policy Condition No. "3" which requires the insured to give notice of any insurance or insurances already effected covering the stocks in trade. from contracts have the force of law between the contracting parties and should be complied with in good faith.  It was incumbent upon petitioner Sy to read the insurance contracts, and this can be reasonably expected of him considering that he has been a businessperson since 1965. Furthermore, the contract concerns indemnity in case of loss in his moneymaking trade of which he should have been aware as it was precisely the reason for his procuring the insurance. doing an insurance

 The Trial Court ruled in favor of the plaintiff
that was reversed by the Court of Appeals. ISSUE:  Whether or not the plaintiff incurred a breach in the policy conditions?

RULING:  The Supreme Court ruled in favor of the insurance companies. The terms of the contract are clear and unambiguous. The insured is specifically required to disclose to the insurer any other insurance and its particulars that he may have effected on the same subject matter. The knowledge of such insurance by the insurer's agents is not the "notice" that would estop the insurers from denying the claim. Thus, it points out that while petitioner Julian Sy claimed that he had informed insurance agent Alvarez regarding the co-insurance on the property, he contradicted himself by inexplicably claiming that he had not read the terms of the policies.

v. What constitutes business



D. Perfection of the Contract of Insurance a. Offer and Acceptance/Consensuality (1) Delay in Acceptance (2) Delivery of Policy II. CONTRACT OF INSURANCE A. What may be insured (Sections 3, 4 and 5) B. Parties to the Contract (Sections 6, 7, 8 and 9) i. Who may be an insurer ii. Who may be insured iii. Rules on insurance by mortgagor or mortgagee iv. Transfer of insurance from mortgagor to mortgagee -Filipinas -Geagonio -PNB PALILEO v. COSIO [G.R. No. L-7667 November 28, 1955] FACTS: On Dec. 18, 1951, Palileo obtained from Cosio a loan in the sum of 12,000. Pursuant to their agreement, Palileo paid to Cosio as interest on the loan a total of P2,250 corresponding to 9 mos from Dec 18, 1951, on the basis of P250 a month, which is more than the maximum interest allowed by law. To secure the payment of the aforesaid loan, defendant required plaintiff to sign a a document known as “Conditional Sale of Residential Bldg” purporting to convey to defendant, with right to repurchase, a two-story building of strong materials belonging to plaintiff. This document did not express the true intention of the parties, which was merely to place said property as security for the payment of the loan. After the execution of the document, defendant insured the building against fire with the Associated Insurance & Surety Co., Inc. for the sum of P15000, the insurance policy having been issued in the name of defendant.



 Furthermore, when the words and language
of documents are clear and plain or readily understandable by an ordinary reader thereof, there is absolutely no room for interpretation or construction anymore. Courts are not allowed to make contracts for the parties; rather, they will intervene only when the terms of the policy are ambiguous, equivocal, or uncertain. The parties must abide by the terms of the contract because such terms constitute the measure of the insurer's liability and compliance therewith is a condition precedent to the insured's right of recovery from the insurer.

 While it is a cardinal principle of insurance
law that a policy or contract of insurance is to be construed liberally in favor of the insured and strictly against the insurer company, yet contracts of insurance, like other contracts, are to be construed according to the sense and meaning of the terms which the parties themselves have used. If such terms are clear and unambiguous, they must be taken and understood in their plain, ordinary and popular sense. Moreover, obligations arising

EH 403 Page 5

⦿ 2010-2011 ⦿ 2011-2012 ⦿

INSURANCE LAW CASE DIGESTS
The building was partly destroyed by fire and, after proper demand, defendant collected from the insurance company an indemnity of P13,107. Plaintiff demanded from defendant that she be credited with the necessary amount to pay her obligation out of the insurance proceeds but defendant refused to do so. Upon these facts, the trial court held that the defendant should credit the sum of P13,107 received by him from the Associated Insurance & Surety Co., Inc. to the payment of plaintiff’s obligation in the sum of P12000, thus considering the agreement fully paid and leaving a balance of P1107 from the insurance collected by the defendant; and since plaintiff had paid to defendant P2250 for 9 mos for interest which exceeds the 12 percent per annum legal interest (P1440 for one year), plaintiff overpaid P810, defendant should refund plaintiff the total of P1107 plus P810 and to pay the costs. ISSUE: WON the “trial court is justified in considering the obligation of plaintiff fully compensated by the insurance amount and in ordering defendant to refund to plaintiff the sum of P1107 representing the difference of the loan of 12K from the sum of P13107 collected by defendant from insurance notwithstanding the fact that it was proven that the insurance was taken for the benefit of the mortgagor?” HELD: SC modified the judgment of the lower court as follows: (1) the transaction had between the plaintiff and defendant was merely an equitable mortgage intended to secure the payment of the loan of 12K; (2) that the proceeds of the insurance amounting to P13, 107 was properly collected by defendant who is not required to account for it to the plaintiff; (3) that the collection of said insurance proceeds shall not be deemed to have compensated the obligation of the plaintiff to the defendant, but bars the latter from claiming its payment from the former; and (4) defendant shall pay to the plaintiff the sum of P810 representing the overpayment made by plaintiff by way of interest on the loan. The rule is that “where a mortgagee, independently of the mortgagor, insures the mortgaged property in his own name and for his own interest, he is entitled to the insurance proceeds in case of loss, but in such case, he is not allowed to retain his claim against the mortgagor, but is passed by subrogation to the insurer to the extent of the money paid”. Or, stated in another way, “the mortgagee may insure his interest in the property independently of the mortgagor. In that event, upon the destruction of the property the insurance money paid to the mortgagee will not inure to the benefit of the mortgagor, and the amount due under the mortgage debt remains unchanged. The mortgagee, however, is not allowed to retain his claim against the mortgagor, but it passes by subrogation to the insurer, to the extent of the insurance money paid”. “The general rule and the weight of authority is, that the insurer is thereupon subrogated to the rights of the mortgagee under the mortgage. This is put upon the analogy of the situation of the insurer to that of a surety.” The correct solution, contrary to the trial court’s ruling, should be that the proceeds of the insurance should be delivered to the defendant but that her claim against the plaintiff should be considered assigned to the insurance company who is deemed subrogated to the rights of the defendant to the extent of the money paid as indemnity. C. Insurable Interest Insurable Interest in life (Section 10)

i.

and

health

El Oriente Fabrica de Tabacos, Inc. vs. Juan Posadas, Collector of Internal Revenue [G.R. No. 34774, September 21, 1931] Facts: Insurer: Manufacturers Life Insurance Co., of Toronto, Canada, thru its local agent E.E. Elser Insured: A. Velhagen (manager of El Oriente) Beneficiary: El Oriente Fabrica de Tabacos, Inc. El Oriente, in order to protect itself against the loss that it might suffer by reason of the death of its manager, whose death would be a serious loss to El Oriente procured from the Insurer an insurance policy on the life of the said manager for the sum of 50,000 USD with El Oriente as the designated sole beneficiary. The insured has no interest or participation in the proceeds of said life insurance policy. El Oriente charged as expenses of its business all the said premiums and deducted the same from its gross incomes as reported in its annual income tax returns, which deductions were allowed by Posadas (Collector of Internal Revenue) upon showing by El Oriente that such premiums were legitimate expenses of the business. Upon the death of the manager, El Oriente received all the proceeds of the life insurance policy together with the interest and the dividends accruing thereon, aggregating P104,957.88. Posadas assessed and levied the sum of P3,148.74 as income tax on the proceeds of the insurance policy, which was paid by El Oriente under protest. El Oriente claiming exemption under Section 4 of the Income Tax Law. Issue: Whether or not the proceeds of insurance taken by a corporation on the life of an important official to indemnify it against loss in case of his death, are

EH 403 Page 6

⦿ 2010-2011 ⦿ 2011-2012 ⦿

INSURANCE LAW CASE DIGESTS
taxable as income under the Philippine Income Tax Law? Ruling: The Income Tax Law for the Philippines is Act No. 2833, as amended. In chapter I On Individuals, is to be found section 4 which provides that, "The following incomes shall be exempt from the provisions of this law: (a) The proceeds of life insurance policies paid to beneficiaries upon the death of the insured ... ." The Chapter on Corporations does not provide as above. It is certain that the proceeds of life insurance policies are exempt. It is not so certain that the proceeds of life insurance policies paid to corporate beneficiaries upon the death of the insured are likewise exempt. The situation will be better elucidated by a brief reference to laws on the same subject in the United States. The Income Tax Law of 1916 extended to the Philippine Legislature, when it came to enact Act No. 2833, to copy the American statute. Subsequently, the Congress of the United States enacted its Income Tax Law of 1919, in which certain doubtful subjects were clarified. Thus, as to the point before us, it was made clear, when not only in the part of the law concerning individuals were exemptions provided for beneficiaries, but also in the part concerning corporations, specific reference was made to the exemptions in favor of individuals, thereby making the same applicable to corporations. This was authoritatively pointed out and decided by the United States Supreme Court in the case of United States vs. Supplee-Biddle Hardware Co. ( [1924], 265 U.S., 189), which involved facts quite similar to those before us. To quote the exact words in the cited case of Chief Justice Taft delivering the opinion of the court: It is earnestly pressed upon us that proceeds of life insurance paid on the death of the insured are in fact capital, and cannot be taxed as income … that proceeds of a life insurance policy paid on the death of the insured are not usually classed as income. Considering, therefore, the purport of the stipulated facts, considering the uncertainty of Philippine law, and considering the lack of express legislative intention to tax the proceeds of life insurance policies paid to corporate beneficiaries, particularly when in the exemption in favor of individual beneficiaries in the chapter on this subject, the clause is inserted "exempt from the provisions of this law," we deem it reasonable to hold the proceeds of the life insurance policy in question as representing an indemnity and not taxable income. The foregoing pronouncement will result in the judgment being reversed and in another judgment being rendered in favor of El Oriente. ii. Rules on change of beneficiary (Section 11) Digested by: Anne THE INSULAR LIFE ASSURANCE COMPANY, LTD. vs. CARPONIA T. EBRADO and PASCUALA VDA. DE EBRADO [G.R. No. L-44059 October 28, 1977] Facts of the Case: On September 1, 1968, Buenaventura Cristor Ebrado was issued by The Life Assurance Co., Ltd., Policy No. 009929 on a whole-life for P5,882.00 with a, rider for Accidental Death for the same amount Buenaventura C. Ebrado designated Carpponia T. Ebrado as the revocable beneficiary in his policy. He to her as his wife. On October 21, 1969, Buenaventura C. Ebrado died when he was hit by a failing branch of a tree. As the policy was in force, The Insular Life Assurance Co., Ltd. liable to pay the coverage in the total amount of P11,745.73, representing the face value of the policy in the amount of P5,882.00 plus the additional benefits for accidental death also in the amount of P5,882.00 and the refund of P18.00 paid for the premium due November, 1969, minus the unpaid premiums and interest thereon due for January and February, 1969, in the sum of P36.27. Carponia T. Ebrado filed with the insurer a claim for the proceeds of the Policy as the designated beneficiary therein, although she admits that she and the insured Buenaventura C. Ebrado were merely living as husband and wife without the benefit of marriage. Pascuala Vda. de Ebrado also filed her claim as the widow of the deceased insured. She asserts that she is the one entitled to the insurance proceeds, not the common-law wife, Carponia T. Ebrado. In doubt as to whom the insurance proceeds shall be paid, the insurer, The Insular Life Assurance Co., Ltd. commenced an action for Interpleader before the Court of First Instance of Rizal on April 29, 1970. After the issues have been joined, a pre-trial conference was held. In the pre-trial conference the parties submits evidence and make admissions.xxx; 8) that the beneficiary designated by the insured in the policy is Carponia Ebrado and the insured made reservation to change the beneficiary but although the insured made the option to change the beneficiary, same was never changed up to the time of his death and the wife did not have any opportunity to write the company that there was reservation to change the designation of the parties it agreed that a decision be rendered based on and stipulation of facts as to who among the two claimants is entitled to the policy. On September 25, 1972, the trial court rendered judgment declaring among others, Carponia T. Ebrado disqualified from becoming beneficiary of the insured

EH 403 Page 7

⦿ 2010-2011 ⦿ 2011-2012 ⦿

INSURANCE LAW CASE DIGESTS
Buenaventura Cristor Ebrado and directing the payment of the insurance proceeds to the estate of the deceased insured. The trial court held that.It is patent from the last paragraph of Art. 739 of the Civil Code that a criminal conviction for adultery or concubinage is not essential in order to establish the disqualification mentioned therein. Neither is it also necessary that a finding of such guilt or commission of those acts be made in a separate independent action brought for the purpose. The guilt of the donee (beneficiary) may be proved by preponderance of evidence in the same proceeding (the action brought to declare the nullity of the donation). Since it is agreed in their stipulation during the pretrial that the deceased insured and defendant Carponia T. Ebrado were living together as husband and wife without being legally married and that the marriage of the insured with the other defendant Pascuala Vda. de Ebrado was valid and still existing at the time the insurance in question was purchased there is no question that defendant Carponia T. Ebrado is disqualified from becoming the beneficiary of the policy in question and as such she is not entitled to the proceeds of the insurance upon the death of the insured. Issue of the Case: Can a common-law wife named as beneficiary in the life insurance policy of a legally married man claim the proceeds thereof in case of death of the latter? Ruling: The SC affirmed the decision of the trial court. under Article 2012 of the same Code, "any person who is forbidden from receiving any donation under Article 739 cannot be named beneficiary of a fife insurance policy by the person who cannot make a donation to him. Common-law spouses are, definitely, barred from receiving donations from each other. Article 739 of the new Civil Code provides: The following donations shall be void: 1. Those made between persons who were guilty of adultery or concubinage at the time of donation; 2. Those made between persons found guilty of the same criminal offense, in consideration thereof; 3. Those made to a public officer or his wife, descendants or ascendants by reason of his office. In the case referred to in No. 1, the action for declaration of nullity may be brought by the spouse of the donor or donee; and the guilt of the donee may be proved by preponderance of evidence in the same action. The underscored clause neatly conveys that no criminal conviction for the offense is a condition precedent. In fact, it cannot even be from the aforequoted provision that a prosecution is needed. On the contrary, the law plainly states that the guilt of the party may be proved "in the same acting for declaration of nullity of donation. And, it would be sufficient if evidence preponderates upon the guilt of the consort for the offense indicated. The quantum of proof in criminal cases is not demanded. In the caw before Us, the requisite proof of commonlaw relationship between the insured and the beneficiary has been conveniently supplied by the stipulations between the parties in the pre-trial conference of the case. It case agreed upon and stipulated therein that the deceased insured Buenaventura C. Ebrado was married to Pascuala Ebrado with whom she has six legitimate children; that during his lifetime, the deceased insured was living with his common-law wife, Carponia Ebrado, with whom he has two children. These stipulations are nothing less than judicial admissions which, as a consequence, no longer require proof and cannot be contradicted. A fortiori, on the basis of these admissions, a judgment may be validly rendered without going through the rigors of a trial for the sole purpose of proving the illicit liaison between the insured and the beneficiary. In fact, in that pretrial, the parties even agreed "that a decision be rendered based on this agreement and stipulation of facts as to who among the two claimants is entitled to the policy." ACCORDINGLY, the appealed judgment of the lower court is hereby affirmed. Carponia T. Ebrado is hereby declared disqualified to be the beneficiary of the late Buenaventura C. Ebrado in his life insurance policy. As a consequence, the proceeds of the policy are hereby held payable to the estate of the deceased insured. Costs against Carponia T. Ebrado. SO ORDERED.

SOUTHERN LUZON EMPLOYEE’S ASSN. V. GOLPEO Digested by Margaret Frances Aparte Note: A common law wife of the insured who has a legal wife is disqualified as beneficiary. It is not required that there be a previous conviction for adultery or concubinage for the prohibition to apply. However, in an earlier case (such as the present case), the common-law wife designated prevailed over the legal wife because the case took place while the Old Civil Code was still applicable, under which there was no provision similar to Art.2012. FACTS: Southern Luzon Employees' Association is composed of laborers and employees of Laguna tayabas Bus Co., and Batangas Transportation Company, and one of its purposes is mutual aid of its members and their defendants in case of death. Roman A. Concepcion was a member until his death on December 13, 1950. In the form required by the

EH 403 Page 8

⦿ 2010-2011 ⦿ 2011-2012 ⦿

INSURANCE LAW CASE DIGESTS
association to be accomplished by its members, with reference to the death benefit, Roman A. Concepcion listed as his beneficiaries Aquilina Maloles, Roman M. Concepcion, Jr., Estela M. Concepcion, Rolando M. Concepcion and Robin M. Concepcion. After the death of Roman A. Concepcion, the association was able to collect voluntary contributions from its members amounting to P2,505. Three sets of claimants presented themselves, namely, (1) Juanita Golpeo, legal wife of Roman A. Concepcion, and her children; (2) Aquilina Maloles, common law wife of Roman A. Concepcion, and her children, named beneficiaries by the deceased; and (3) Elsie Hicban, another common law wife of Roman A. Concepcion, and her child. The court rendered a decision, declaring the defendants Aquiliana Malolos and her children the sole beneficiaries of the sum of P2,505.00 and ordering the plaintiff to deliver said amount to them. ISSUE: WHETHER OR NOT THE COURT COMMITED ERROR IN DESIGNATING A COMMON LAW WIFE OF AN INSURED AS THE BENEFICIARY INSTEAD OF THE LEGAL WIFE. Remember: This case took place while the Old Civil Code was still applicable. HELD: Judgment affirmed. The decision is based mainly on the theory that the contract between the plaintiff and the deceased Roman A. Concepcion partook of the nature of an insurance and that, therefore, the amount in question belonged exclusively to the beneficiaries, invoking the following pronouncements of this Court in the case of Del Val vs. Del Val, 29 Phil., 534: With the finding of the trial court that the proceeds of the life-insurance policy belongs exclusively to the defendant as his individual and separate property, we agree. That the proceeds of an insurance policy belong exclusively to the beneficiary and not to the estate of the person whose life was insured, and that such proceeds are the separate and individual property of the beneficiary, and not of the heirs of the person whose life was insured, is the doctrine in America. We believe that the same doctrine obtains in these Islands by virtue of section 428 of the Code of Commerce, which reads: "The amounts which the underwriter must deliver to the person insured, in fulfillment of the contract, shall be the property creditors of any kind whatsoever of the person who effected the insurance in favor of the formers." AS TO THE CONTENTION OF THE COUNSEL’S PLAINTIFF THAT THE PROCEEDS OF THE INSURANCE POLICY WERE DONATION OR GIFT MADE BY THE FATHER DURING HIS LIFETIME, SUCH THAT UNDER THE CIVIL CODE ARE NOT BETTERMENTS AND SHALL BE CONSIDERED AS PART OF THE LEGAL PORTION. The court disagrees with this contention. The contract of life insurance is a special contract and the destination of the proceeds thereof is determined by special laws which deal exclusively with that subject. The Civil Code has no provisions which relate directly and specifically to life-insurance contract or to the destination of life-insurance proceeds. That subject is regulate exclusively by the Code of Commerce which provides for the terms of the contract, the relations of the parties and the destination of the proceeds of the policy. Digested by: Kwin Asunto SOCIAL SECURITY SYSTEM vs. CANDELARIA D. DAVAC [G.R. No. L-21642. July 30, 1966.] Facts: This is an appeal from the resolution of the Social Security Commission declaring respondent Candelaria Davac as the person entitled to receive the death benefits payable for the death of Petronilo Davac. The late Petronilo Davac, a former employee of Lianga Bay Logging Co. Inc. became a member of the Social Security System (SSS for short) on September 1, 1957. He designated respondent Candelaria Davac as his beneficiary and indicated his relationship to her as that of "wife". When he died, each of the respondents (Candelaria Davac and Lourdes Tuplano) filed their claims for death benefit with the SSS. It appears from their respective claims and the documents submitted in support thereof, that the deceased contracted two marriages, the first, with claimant Lourdes Tuplano on August 29, 1946, who bore him a child, Romeo Davac and the second, with claimant Candelaria Davac on January 18, 1949, with whom he had a minor daughter, Elizabeth Davac. Due to their conflicting claims, the processing thereof was held in abeyance, whereupon the SSS filed this petition praying that respondents be required to interplead and litigate between themselves their conflicting claims over the death benefits in question. Issue: Whether or not the Social Security Commission acted correctly in declaring respondent Candelaria Davac as the person entitled to receive the death benefits in question. Held: yes. The benefit receivable under the Social Security Act is in the nature of a special privilege or an arrangement secured by the law, pursuant to the policy of the State to provide social security to the workingmen. The amounts that may thus be received cannot be considered as property earned by the member during

EH 403 Page 9

⦿ 2010-2011 ⦿ 2011-2012 ⦿

INSURANCE LAW CASE DIGESTS
his lifetime, and, hence, do not form part of the properties of the conjugal partnership or of the estate of the said member. They are disbursed from a public special fund created by Congress pursuant to the declared policy of the Republic "to develop, establish gradually and perfect a social security system which . . . shall provide protection against the hazards of disability, sickness, old age and death." (Section 1, Republic Act No. 1792.) Consequently, if there is a named beneficiary and the designation is not invalid, it is not the heirs of the employee who are entitled to receive the benefits, unless they are the designated beneficiaries themselves. It is only when there is no designated beneficiary or when the designation is void that the laws of succession become applicable. The Social Security Act is not a law of succession. Digested by: Rachel R. Aying RE: CLAIMS FOR BENEFITS OF THE HEIRS OF THE LATE MARIO V. CHANLIONGCO, FIDELA B CHANLIONGCO, MARIO B. CHANLIONGCO II, MA. ANGELINA C. BUENAVENTURA and MARIO C. CHANLIONGCO, JR. Facts: This matter refers to the claims for retirement benefits filed by the heirs of the late ATTY. MARIO V. CHANLIONGCO an attorney of the Court, it is in the records that at the time of his death, Atty. Chanliongco was more than 63 years of age, with more than 38 years of service in the government. He did not have any pending criminal administrative or not case against him, neither did he have any money or property accountability. The highest salary he received was P18,700.00 per annum. Aside from his widow, Dra. Fidel B. Chanliongco and an only Intimate Mario it appears that there are other deceased to namely, Mrs. Angelina C. , Jr., both born out of wedlock to Angelina R Crespo, and duly recognized by the deceased. Except Mario, Jr., who is only 17 years of age, all the claimants are of legal age. According to law, the benefits accruing to the deceased consist of: (1) retirement benefits; (2) money value of terminal leave; (3) life insurance and (4) refund of retirement premium. From the records now before US, it appears that the GSIS had already the release the life insurance proceeds; and the refund of rent to the claimants. RULING: The record also shows that the late Atty. Chanliongco died ab intestato and that he filed or over to state in his application for membership with the GSIS the beneficiary or benefits of his retirement benefits, should he die before retirement. Hence, the retirement benefits shall accrue to his estate and will be distributed among his Legal heirs in with the benefits on intestate s , as in the caw of a fife if no benefit is named in the policy (Vda. de vs. GSIS, L-28093, Jan. 30, 1971, 37 SCRA 315, 325). AQUINO, J., concurring: There may be instances, like the instant case, where in legal succession the estate is distributed according to the rules on legitime without applying the rules on intestate ion. The reason is that sometimes the estate is not even sufficient to satisfy the legitimes. The legitimes of the primary compulsory heirs, like a child or descendant, should first be satisfied. In this case the decedent's legal heirs are his legitimate child, his widow and two intimate children. His estate is partitioned among those heirs by giving them their respective time. The legitimate child gets one-half of the estate as his legitime which is regarded as his share as a legal heir Art 888, Civil Code). The widow's legitime is one-fourth of the estate. That represents also her share as a legal heir. The remaining one-fourth of the estate, which is the free portion, goes to the illegitimate children in equal shares, as their legitime, Pursuant to the provision that 'the legitimate of the illegitimate children shall be taken from the portion of the estate at the free disposal of the testator, provoked that in no case shall the total legitime of such illegitimate children exceed that free portion, and that the legitime of the surviving spouse must first be fully satisfied. The rule in Santillon vs. Miranda, L-19281, June 30, 1965, 14 SCRA 563, that when the surviving spouse concurs with only one legitimate child, the spouse is entitled to one-half of the estate and the gets the other half, t to article 996 of the Civil Code, does not apply to the case because here intimate children concur with the surviving spouse and the intimate child. In this case, to divide the estate between the surviving spouse and the ligitemate child that deprive the illegitimate children of their legitime. So, the decendent's estate is distributed in the proportion of 1/2 for the legitimate child, 1/4 for the widow and 1/8 each for the two illegitimate children. Also not of possible application to this case is the rule that the legal of an acknowledge natural child is 1/2 of the legitime of the legitimate child of that the of the spurious child is 2/5 of that of the of the intimate child or 4/5 of that of that of the acknowledged natural child. The rule be applied because the estate is not sufficient to cover legitimes of all compulsory heirs. That is one of the flaws of the law of succession. A situation as in the instant case may arise where the illegitimate children get less than their legitime. With respect to the decendant's unpaid salary and the money value of

EH 403 Page 10

⦿ 2010-2011 ⦿ 2011-2012 ⦿

INSURANCE LAW CASE DIGESTS
his leave, the same are conjugal properties because of the rule that property "obtained by the or work, or as salary of the spouses, or either of them", is conjugal in character. iii. Forfeiture of beneficiary’s interest (Section 12) iv. Insurable Interest in property (Sections 13, 14, 15, 16, 17, 18) TRADERS INSURANCE & SURETY CO vs. JUAN GOLANGCO Y OTRA [Sep 21, 1954, En Banc G.R. No. L-6442] (I really had a hard time deciphering the facts of this case as it was written in Spanish ( facts from the lower court as adopted by SC), but I assure you I have full faith that this digest is as accurate corresponding to the full text). Facts: Tomas Lianco and the Archbishop ( no name indicated) entered into a contract of lease on a parcel of land owned by church . As lessee, Lianco erected a building on the leased portion of the church’s land. Lianco later transferred ownership of this building to Kaw Eng Si, who later transferred the same to Golangco. This transfers by Lianco of his right to lease and the building’ownership were without consent of the Archbishop. The Archbishop filed an ejectment case against Lianco, who appears to be occupants of the premises building with others paying rent to Golangco. This right of Golangco to receive rent on the building was judicially recognized in a case decided between Lianco and some others occupying the premises pursuant to a compromise agreement. At the moment, the Archbishop did not exercise his option to question Golangco’s rights as lessee, as the transfer by Lianco was without the Archbishop’s consent. On April 7, 1949,Golangco applied for fire insurance with Trader’s Insurance and Surety Co. of which Golangco was issued fire insurance policy stating “that all insurance covered under said policy, includes the 'rent or other subject matter of insurance in respect of or in connection with any building or any property contained in any building'. On June 5, 1949, fire ravaged the building premises pursuant of which Golangco requested Trader’s Insurance to pay the insurance amount of 10,000 including the amount of rent P1,100 monthly. Trader’s insurance refused to pay the insurance as pertaining to the rent averring that Golangco has no insurable interest therein. Issue: WON Golangco has insurable interest ( in the property) on the rent of the building premises which may lawfully/validly be subject of insurance? Ruling: Yes, Sec. 13 of the Insurance Code provides that “Every interest in the property, whether real or personal, or any relation thereto, or liability in respect thereof of such nature that a contemplated peril might directly damnify the insured, is an insurable interest.” By virtue of the contract between Tomas B. Lianco and the Archbishop, Lianco erected the building of which the premises in question form part and became owner thereof . He transferred the ownership of the premises in question to kaw Eng Si, who in turn transferred it to plaintiff Juan Golangco .Lianco and the actual occupant of the premises acknowledged plaintiff's right to collect rentals thereon in a compromise agreement which was incorporated in a judicial judgment. Both at the time of the issuance of the policy and at the time of the fire, plaintiff Golangco was in legal possession of the premises, collecting rentals from its occupant. It seems plain that if the premises were destroyed - as they were - by fire, Golangco would be, as he was, directly damnified thereby; and hence he had an insurable interest therein (section 13, Insurance Law). It is to be noted that the policy so worded indicates that the fire insurance policy includes 'rent or other subject matter of insurance in respect of or in connection with any building or any property contained in any building'. The argument of Trader’s Insurance that a policy of insurance must specify the interest of the insured in the property insured, if he is not the absolute owner thereof, is not meritorious because it was the Trader’s, not Golangco, who prepared that policy, and it cannot take advantage of its own acts to plaintiff's detriment; and, in any case, this provision was substantially complied with by Golangco when he made a full and clear statement of his interests to Trader's manager. The contract between Lianco and the Archbishop only forbade Lianco from transferring 'his rights as LESSEE but the contracts Lianco made in favor of Kaw Eng Si and plaintiff Golangco did not transfer such rights; and hence no written consent thereto was necessary. At worst, the contract would be voidable, but not a void contract, at the option of the Archbishop; but this would not deprive Golangco of his insurable interest until such option were exercised; and it does not appear that it was ever exercised. The ejectment case filed by the Archbishop against Lianco did not remove nor destroy plaintiff's insurable interest: first, because plaintiff was not a party thereto and cannot be bound thereby; and second, because the judgment of the Municipal Court, at least as late as February 14, 1950, had not been executed so far as possession of the premises were concerned; so that, as far as plaintiff Golangco was concerned, his right to the premises and to the rentals thereon continued to exist on June 5, 1949 when the fire took place." FILIPINO MERCHANTS INSURANCE CO., INC., vs. COURT OF APPEALS and CHOA TIEK SENG [G.R. No. 85141 November 28, 1989]

EH 403 Page 11

⦿ 2010-2011 ⦿ 2011-2012 ⦿

INSURANCE LAW CASE DIGESTS
FACTS: CHOA TIEK SENG was the consignee of the shipment of fishmeal loaded on board a vessel and unloaded at the Port of Manila. The goods were described as 600 metric tons of fishmeal in new gunny bags of 90 kilos each. He insured said shipment with FILIPINO MERCHANTS insurance company from Bangkok, Thailand to Manila against all risks. 2. When the fishmeal were unloaded from the ship at Manila unto the arrastre contractor and surveyor, it was ascertained and certified that in such discharge 105 bags were in bad order condition. The extent of shortage or loss totalling 227 bags amounting to 12,148 kilos was noted in the Bad Order Certificate. 3. The consignee made a formal claim against the Insurance Company for P51,568.62. The latter refused to pay the claim so the consignee brought an action against the insurance company. 4. The trial court rendered judgment, which was affirmed by the Court of appeals, in favor of the consignee, ordering the insurance company to pay the former the sum of P51,568.62. 5. The insurance company claimed that under the "all risks" clause of the marine insurance policy it is not liable to the consignee for the partial loss of the cargo because of the absence of proof of some fortuitous event, casualty, or accidental cause to which the loss is attributable. The insurance company argues that the consignee has the burden to adduce evidence, which it failed to do, that the alleged loss to the cargo was due to a fortuitous event. To put at rest all doubts on the matter, the Court discussed on the issue of lack of insurable interest, despite being raised by the insurance company for the first time on appeal. 1. ISSUES: 1. Whether or not the consignee had an insurable interest 2. Whether or not there must be some fortuitous event for the claim to be compensable under “all risks” marine insurance policy RULING: 1. Yes, the consignee had an insurable interest in said goods which may be a valid subject of a contract of insurance. His interest over the goods is based on the perfected contract of sale. The perfected contract of sale between him and the shipper of the goods operates to vest in him an equitable title even before delivery or before be performed the conditions of the sale. Section 13 of the Insurance Code defines insurable interest in property as “every interest in property, whether real or personal, or any relation thereto, or liability in respect thereof, of such nature that a contemplated peril might directly damnify the insured. “ In principle, anyone has an insurable interest in property who derives a benefit from its existence or would suffer loss from its destruction whether he has or has not any title in, or lien upon or possession of the property. Insurable interest in property may consist in: (a) an existing interest; (b) an inchoate interest founded on an existing interest; or (c) an expectancy, coupled with an existing interest in that out of which the expectancy arises. The contract of shipment, whether under F.O.B., C.I.F., or C. & F. as in this case, is immaterial in the determination of whether the vendee has an insurable interest or not in the goods in transit. The perfected contract of sale even without delivery vests in the vendee an equitable title, an existing interest over the goods sufficient to be the subject of insurance. 2. No. An "all risks policy" should be read literally as meaning all risks whatsoever and covering all losses by an accidental cause of any kind. The very nature of the term "all risks" must be given a broad and comprehensive meaning as covering any loss other than a willful and fraudulent act of the insured. The term "all risks" cannot be given a strained technical meaning, to the effect that it extends to all damages/losses suffered by the insured cargo except: (a) loss or damage or expense proximately caused by delay, and (b) loss or damage or expense proximately caused by the inherent vice or nature of the subject matter insured. Purpose of “All Risks” insurance The very purpose of ”all risks" insurance is to give protection to the insured in those cases where difficulties of logical explanation or some mystery surround the loss or damage to property. An "all asks" policy has been evolved to grant greater protection than that afforded by the "perils clause," in order to assure that no loss can happen through the incidence of a cause neither insured against nor creating liability in the ship; it is written against all losses, that is, attributable to external causes. Burden of Proof Generally, the burden of proof is upon the insured to show that a loss arose from a covered peril, but under an "all risks" policy the burden is not on the insured to prove the precise cause of loss or damage for which it seeks compensation. The insured only has the initial burden of proving that the cargo was in good condition when the policy attached and that the cargo was damaged when

EH 403 Page 12

⦿ 2010-2011 ⦿ 2011-2012 ⦿

INSURANCE LAW CASE DIGESTS
unloaded from the vessel; thereafter, the burden then shifts to the insurer to show the exception to the coverage. The burden of the insured, therefore, is to prove merely that the goods he transported have been lost, destroyed or deteriorated. Thereafter, the burden is shifted to the insurer to prove that the loss was due to excepted perils. To impose on the insured the burden of proving the precise cause of the loss or damage would be inconsistent with the broad protective purpose of "all risks" insurance. As held in Paris-Manila Perfumery Co. vs. Phoenix Assurance Co., Ltd. “ the basic rule is that the insurance company has the burden of proving that the loss is caused by the risk excepted and for want of such proof, the company is liable.” Special type of insurance Coverage under an "all risks" provision of a marine insurance policy creates a special type of insurance which extends coverage to risks not usually contemplated and avoids putting upon the insured the burden of establishing that the loss was due to the peril falling within the policy's coverage. The insurer can avoid coverage upon demonstrating that a specific provision expressly excludes the loss from coverage. A marine insurance policy providing that the insurance was to be "against all risks" must be construed as creating a special insurance and extending to other risks than are usually contemplated, and covers all losses except such as arise from the fraud of the insured. In the present case, there being no showing that the loss was caused by any of the excepted perils i.e. delay or the inherent vice or nature of the subject matter insured, the insurer is liable under the policy. v. When insurable (Section 19) interest must exist 31, 1975, the building and the contents were totally razed by fire. Respondents, Zenith Insurance, Phil. British Assurance and SSS Accredited Group of Insurers, paid their corresponding shares of the loss. Demand was made from respondent Travellers Multi-Indemnity for its share in the loss but the same was refused. Hence, complainants demanded from the other three respondents the balance of each share in the loss but the same was also refused. Petitioner then filed a complaint in intervention claiming the proceeds of the fire insurance policy issued by respondent Travellers Multi-Indemnity but the latter alleged that it is not entitled to indemnity for lack of insurable interest before the loss of the insured premises. Issue: Whether or not petitioner has an insurable interest. Ruling: Respondent insurance company did not assail the validity of the insurance policy taken out by petitioner over the mortgaged property. Neither did it deny that the said property was totally razed by fire within the period covered by the insurance. Respondent advanced an affirmative defense of lack of insurable interest on the part of the petitioner that before the occurrence of the peril insured against the Palomos had already paid their credit due the petitioner. Respondent having admitted the material allegations in the complaint has the burden of proof to show that petitioner has no insurable interest over the insured property at the time the contingency took place. Upon that point, there is a failure of proof. Respondent exerted no effort to present any evidence to substantiate its claim, while petitioner did. For said respondent's failure, the decision must be adverse to it. However, respondent Insurance Commission absolved respondent insurance company from liability on the basis of the certification issued by the then Court of First Instance of Davao, Branch II, that in a certain civil action against the Palomos, Arsenio Lopez Chua stands as the complainant and not Tai Tong Chuache. From said evidence respondent commission inferred that the credit extended by herein petitioner to the Palomos secured by the insured property must have been paid. Such is a glaring error which cannot be sanctioned. Respondent Commission's findings are based upon a mere inference. The record of the case shows that the petitioner to support its claim for the insurance proceeds offered as evidence the contract of mortgage which has not been cancelled nor released. It has been held in a long line of cases that when the creditor is in possession of the document of credit, he need not prove non-payment for it is presumed. The validity of the insurance policy taken by petitioner was not assailed by private respondent. Moreover, petitioner's claim that the loan extended to the Palomos has not yet been paid was

Digested by: Birondo Tai Tong Chuache & Co. v. Insurance Commission Facts: Complainants acquired a parcel of land and a building they assumed the mortgage of the latter in favor of SSS, which was insured with respondent SSS Accredited Group of Insurers. On April 19, 1975, Azucena Palomo obtained a loan from petitioner Tai Tong Chuache Inc. securing it with a mortgage was executed over the land and the building in favor of petitioner. On April 25, 1975, Arsenio Chua, petitioner’s representative insured the latter's interest with Travellers Multi-Indemnity Corporation. On June 11, 1975, Pedro Palomo secured fire insurance covering the building with respondent Zenith Insurance Corporation and on July 16, 1975, another fire insurance was procured from respondent Philippine British Assurance Company, covering the same building and the contents thereof. However, on July

EH 403 Page 13

⦿ 2010-2011 ⦿ 2011-2012 ⦿

INSURANCE LAW CASE DIGESTS
corroborated by Azucena Palomo who testified that they are still indebted to herein petitioner. Public respondent argues however, that if the civil case really stemmed from the loan granted to Azucena Palomo by petitioner the same should have been brought by Tai Tong Chuache or by its representative in its own behalf. From the above premise respondent concluded that the obligation secured by the insured property must have been paid. The premise is correct but the conclusion is wrong. Citing Rule 3, Sec. 2 respondent pointed out that the action must be brought in the name of the real party in interest. Correct! However, it should be borne in mind that petitioner being a partnership may sue and be sued in its name or by its duly authorized representative. The fact that Arsenio Lopez Chua is the representative of petitioner is not questioned. Petitioner's declaration that Arsenio Lopez Chua acts as the managing partner of the partnership was corroborated by respondent insurance company. Thus Chua as the managing partner of the partnership may execute all acts of administration including the right to sue debtors of the partnership in case of their failure to pay their obligations when it became due and demandable. Or at the very least, Chua being a partner of petitioner Tai Tong Chuache & Company is an agent of the partnership. Being an agent, it is understood that he acted for and in behalf of the firm. Public respondent's allegation that the civil case filed by Arsenio Chua was in his capacity as personal creditor of spouses Palomo has no basis. The respondent insurance company having issued a policy in favor of herein petitioner which policy was of legal force and effect at the time of the fire, it is bound by its terms and conditions. Upon its failure to prove the allegation of lack of insurable interest on the part of the petitioner, respondent insurance company is and must be held liable. vi. Rules in change of interest (Sections 20, 21, 22, 23, 24) vii. Void Stipulations (Section 25) i. D. Concealment Definition (Sections 26, 27) City wrote on the corresponding form in his own handwriting. Mondragon finally type-wrote the data on the application form which was signed by private respondent Ngo Hing. The latter paid the annual premium the sum of P1,077.75 going over to the Company, but he retained the amount of P1,317.00 as his commission for being a duly authorized agent of Pacific Life. Upon the payment of the insurance premium, the binding deposit receipt was issued to private respondent Ngo Hing. Likewise, petitioner Mondragon handwrote at the bottom of the back page of the application form his strong recommendation for the approval of the insurance application. Then on April 30, 1957, Mondragon received a letter from Pacific Life disapproving the insurance application. The letter stated that the said life insurance application for 20year endowment plan is not available for minors below seven years old, but Pacific Life can consider the same under the Juvenile Triple Action Plan, and advised that if the offer is acceptable, the Juvenile Non-Medical Declaration be sent to the company. The non-acceptance of the insurance plan by Pacific Life was allegedly not communicated by petitioner Mondragon to private respondent Ngo Hing. Instead, on May 6, 1957, Mondragon wrote back Pacific Life again strongly recommending the approval of the 20year endowment insurance plan to children, pointing out that since 1954 the customers, especially the Chinese, were asking for such coverage. It was when things were in such state that on May 28, 1957 Helen Go died of influenza with complication of bronchopneumonia. Thereupon, private respondent sought the payment of the proceeds of the insurance, but having failed in his effort, he filed the action for the recovery of the same before the Court of First Instance of Cebu, which rendered the adverse decision as earlier referred to against both petitioners. ISSUE: Whether the respondent is entitled to the insurance SC: Since petitioner Pacific Life disapproved the insurance application of respondent Ngo Hing, the binding deposit receipt in question had never become in force at any time. As held by this Court, where an agreement is made between the applicant and the agent, no liability shall attach until the principal approves the risk and a receipt is given by the agent. The acceptance is merely conditional and is subordinated to the act of the company in approving or rejecting the application. Thus, in life insurance, a "binding slip" or "binding receipt" does not insure by itself It bears repeating that through the intra-company communication of April 30, 1957 Pacific Life disapproved the insurance application in question on the ground that it is not offering the twenty-year

Digested by: Lo, Justine GREAT PACIFIC LIFE ASSURANCE COMPANY vs. HONORABLE COURT OF APPEALS [G.R. No. L-31845 April 30, 1979] FACTS: It appears that on March 14, 1957, private respondent Ngo Hing filed an application with the Great Pacific Life Assurance Company (hereinafter referred to as Pacific Life) for a twenty-year endowment policy in the amount of P50,000.00 on the life of his one-year old daughter Helen Go. Said respondent supplied the essential data which petitioner Lapulapu D. Mondragon, Branch Manager of the Pacific Life in Cebu

EH 403 Page 14

⦿ 2010-2011 ⦿ 2011-2012 ⦿

INSURANCE LAW CASE DIGESTS
endowment insurance policy to children less than seven years of age. What it offered instead is another plan known as the Juvenile Triple Action, which private respondent failed to accept. In the absence of a meeting of the minds between petitioner Pacific Life and private respondent Ngo Hing over the 20-year endowment life insurance in the amount of P50,000.00 in favor of the latter's one-year old daughter, and with the non-compliance of the above quoted conditions stated in the disputed binding deposit receipt, there could have been no insurance contract duly perfected between then Acordingly, the deposit paid by private respondent shall have to be refunded by Pacific Life. We are not impressed with private respondent's contention that failure of petitioner Mondragon to communicate to him the rejection of the insurance application would not have any adverse effect on the allegedly perfected temporary contract In this first place, there was no contract perfected between the parties who had no meeting of their minds. Private respondent, being an authorized insurance agent of Pacific Life at Cebu branch office, is indubitably aware that said company does not offer the life insurance applied for. When he filed the insurance application in dispute, private respondent was, therefore, only taking the chance that Pacific Life will approve the recommendation of Mondragon for the acceptance and approval of the application in question along with his proposal that the insurance company starts to offer the 20-year endowment insurance plan for children less than seven years. Nonetheless, the record discloses that Pacific Life had rejected the proposal and recommendation. Secondly, having an insurable interest on the life of his one-year old daughter, aside from being an insurance agent and an offense associate of petitioner Mondragon, private respondent Ngo Hing must have known and followed the progress on the processing of such application and could not pretend ignorance of the Company's rejection of the 20-year endowment life insurance application. This Court is of the firm belief that private respondent had deliberately concealed the state of health and physical condition of his daughter Helen Go. Where private respondent supplied the required essential data for the insurance application form, he was fully aware that his one-year old daughter is typically a mongoloid child. Such a congenital physical defect could never be ensconced nor distinguished. Nonetheless, private respondent, in apparent bad faith, withheld the fact material to the risk to be assumed by the insurance company. As an insurance agent of Pacific Life, he ought to know, as he surely must have known. his duty and responsibility to such a material fact. Had he diamond said significant fact in the insurance application form Pacific Life would have verified the same and would have had no choice but to disapprove the application outright. Whether intentional or unintentional the concealment entitles the insurer to rescind the contract of insurance. Private respondent appears guilty thereof. We are thus constrained to hold that no insurance contract was perfected between the parties with the noncompliance of the conditions provided in the binding receipt, and concealment, as legally defined, having been combated by herein private respondent. ii. Concealment of Material Fact in bad faith (Section 28) SEGUNDINA MUSÑGI, ET AL., vs. WEST COAST LIFE INSURANCE CO. [G.R. No. L-41794 August 30, 1935] Facts: The plaintiffs, as beneficiaries, brought suit against the defendant to recover the value of two life insurance policies. Arsenio T. Garcia was insured by the defendant company in the sum of P5, 000. Arsenio T. Garcia was again insured by the defendant company in the sum of P10,000. Subsequently, Arsenio died. Even with the demand made by the plaintiffs to the defendant company to pay the two policies, defendant refused to pay It is to be noted that in both applications, the insured had to answer inquiries as to his state of health and that of his family, which he did voluntarily. In each of the said applications the following question was asked: "1. What physician or practitioner or any other person not named above have you consulted or been treated by, and for what illness, or ailment? (If none, so state.)" In the first application, the insured answered "None", and in the second, "No". These answers of the insured as well as his other statements contained in his applications were one of the causes or considerations for the issuance of the policies, and they so positively appear therein. After the death of the insured and as a result of the demand made by the beneficiaries upon the defendant to pay the value of the policies, the latter discovered that the aforementioned answers were false and fraudulent, because the truth was that the insured, before answering and signing the applications and before the issuance of the policies, had been treated in the General Hospital by a lady physician for different ailments. The defendant contends that the two policies did not create any valid obligation because they were fraudulently obtained by the insured. Issue: Whether the two answers given by the insured in his applications are false, and if they were the cause, or one of the causes, which induced the defendant to issue the policies? Ruling:

EH 403 Page 15

⦿ 2010-2011 ⦿ 2011-2012 ⦿

INSURANCE LAW CASE DIGESTS
The concealment and the false statements constituted fraud because the defendant by reason thereof accepted the risk which it would otherwise have flatly refused. When not otherwise specially provided for by the Insurance Law, the contract of life insurance is governed by the general rules of the civil law regarding contracts. Article 1261 of the Civil Code provides that there is no contract unless there should be, in addition to consent and a definite object, a consideration for the obligation established. And article 1276 provides that the statement of a false consideration shall render the contract void. The two answers being one of the considerations of the policies, and it appearing that they are false and fraudulent, it is evident that the insurance contracts were null and void and did not give rise to any right to recover their value or amount. A similar case was already decided by this court in Argente vs. West Coast Life Insurance Co. (51 Phil., 725). In discussing the legal phase of the case, this court said: One ground for the rescission of a contract of insurance under the Insurance Act is a "concealment", which in section 25 is defined as "A neglect to communicate that which a party knows and ought to communicate". In view of the foregoing, appellant's first two assignments of error are well founded, wherefore, the appealed judgment is reversed and the defendant absolved from the complaint. Digested by: Brian O. Dalanon Sunlife Assurance Company of Canada vs. CA Facts: On April 15, 1986, Robert John B. Bacani procured a life insurance contract for himself from petitioner at P100,000.00 with double indemnity in case of accidental death. The designated beneficiary was his mother, respondent Bernarda Bacani. On June 26, 1987, the insured died in a plane crash. Respondent Bernarda Bacani filed a claim with petitioner, seeking the benefits of the insurance policy taken by her son. Petitioner conducted an investigation and its findings prompted it to reject the claim. In its letter, petitioner informed respondent Bernarda Bacani, that the insured did not disclose material facts relevant to the issuance of the policy, thus rendering the contract of insurance voidable. A check representing the total premiums paid in the amount of P10,172.00 was attached to said letter. Petitioner claimed that the insured gave false statements in his application when he answered the following questions: 5. Within the past 5 years have you: a) consulted any doctor or other health practitioner? b) submitted to: EGG, X-rays, blood tests or other tests? c) attended or been admitted to any hospital or other medical facility? b) urine, kidney or bladder disorder? The deceased answered question No. 5(a) in the affirmative but limited his answer to a consultation with a certain Dr. Reinaldo D. Raymundo of the Chinese General Hospital on February 1986, for cough and flu complications. The other questions were answered in the negative. Petitioner discovered that two weeks prior to his application for insurance, the insured was examined and confined at the Lung Center of the Philippines, where he was diagnosed for renal failure. During his confinement, the deceased was subjected to urinalysis, ultra-sonography and hematology tests. Respondent Bernarda Bacani and her husband, respondent Rolando Bacani, filed an action for specific performance against petitioner with the Regional Trial Court. Petitioner filed its answer with counterclaim and a list of exhibits consisting of medical records furnished by the Lung Center of the Philippines. Petitioner filed its Request for Admissions relative to the authenticity and due execution of several documents as well as allegations regarding the health of the insured. Private respondents failed to oppose said request or reply thereto, thereby rendering an admission of the matters alleged. In ruling for private respondents, the trial court concluded that the facts concealed by the insured were made in good faith and under a belief that they need not be disclosed. Moreover, it held that the health history of the insured was immaterial since the insurance policy was "non-medical". Petitioner appealed to the Court of Appeals, which affirmed the decision of the trial court. The appellate court ruled that petitioner cannot avoid its obligation by claiming concealment because the cause of death was unrelated to the facts concealed by the insured. It also sustained the finding of the trial court that matters relating to the health history of the insured were irrelevant since petitioner waived the medical examination prior to the approval and issuance of the insurance policy. Moreover, the appellate court agreed with the trial court that the policy was "non-medical.” Issue: Whether or not the insured concealed informations in the application of his policy and whether or not the petitioner has the right to rescind the contract. Ruling: Section 26 of The Insurance Code is explicit in requiring a party to a contract of insurance to

EH 403 Page 16

⦿ 2010-2011 ⦿ 2011-2012 ⦿

INSURANCE LAW CASE DIGESTS
communicate to the other, in good faith, all facts within his knowledge which are material to the contract and as to which he makes no warranty, and which the other has no means of ascertaining. Said Section provides: A neglect to communicate that which a party knows and ought to communicate, is called concealment. The terms of the contract are clear. The insured is specifically required to disclose to the insurer matters relating to his health. The information which the insured failed to disclose were material and relevant to the approval and issuance of the insurance policy. The matters concealed would have definitely affected petitioner's action on his application, either by approving it with the corresponding adjustment for a higher premium or rejecting the same. Moreover, a disclosure may have warranted a medical examination of the insured by petitioner in order for it to reasonably assess the risk involved in accepting the application. Thus, "goad faith" is no defense in concealment. The insured's failure to disclose the fact that he was hospitalized for two weeks prior to filing his application for insurance, raises grave doubts about his bonafides. It appears that such concealment was deliberate on his part. We, therefore, rule that petitioner properly exercised its right to rescind the contract of insurance by reason of the concealment employed by the insured. iii. Concealment of Matters proving falsity of a warranty (Section 29) iv. What need not be disclosed unless required (Sections 30, 32-35) v. Test of materiality in concealment (Section 31) Digested by: Grace Jayne Dingal Vda. DE CANILANG v. COURT OF Facts: On 18 June 1982, Jaime Canilang consulted Dr. Claudio and was diagnosed as suffering from "sinus tachycardia” And was latter found to have "acute bronchitis." On next day, Jaime applied for a "nonmedical" insurance policy with respondent Great Pacific Life Assurance naming his wife, Thelma Canilang, as his beneficiary. Jaime was issued ordinary life insurance Policy effective as of 9 August 1982. On 5 August 1983, Jaime Canilang died of "congestive heart failure," "anemia," and "chronic anemia." Petitioner, widow and beneficiary of the insured, filed a claim with Great Pacific which the insurer denied upon the ground that the insured had concealed material information from it. Petitioner filed a complaint against Great Pacific with the Insurance Commission for recovery of the insurance proceeds. During the hearing called by the Insurance Commissioner, petitioner testified that she was not aware of any serious illness suffered by her late husband. The medical declaration which was set out in the application for insurance executed by Jaime Canilang read as follows: xxxx (1) I have not been confined in any hospital, sanitarium or infirmary, nor receive any medical or surgical advice/attention within the last five (5) years. (2) I have never been treated nor consulted a physician for a heart condition, high blood pressure, cancer, diabetes, lung, kidney, stomach disorder, or any other physical impairment. (3) I am, to the best of my knowledge, in good health. EXCEPTIONS: Xxx Issue: Whether or not there is a material concealment Held: There is a material concealment. On appeal by Great Pacific, the Court of Appeals reversed and set aside the decision of the Insurance Commissioner and dismissed Thelma Canilang's complaint and Great Pacific's counterclaim. The Court of Appeals found that the failure of Jaime Canilang to disclose previous medical consultation and treatment constituted material information which should have been communicated to Great Pacific to enable the latter to make proper inquiries. Canilang failed to disclose, under the caption "Exceptions," that he had twice consulted Dr. Claudio who had found him to be suffering from "sinus tachycardia" and "acute bronchitis." The provisions of P.D. No. 1460, also known as the Insurance Code of 1978 read as follows: Sec. 26. A neglect to communicate that which a party knows and ought to communicate, is called a concealment. xxx xxx xxx Sec. 28. Each party to a contract of insurance must communicate to the other, in good faith, all factors within his knowledge which are material to the contract and as to which he makes no warranty, and which the other has not the means of ascertaining. The information concealed must be information which the concealing party knew and "ought to [have] communicate[d]," that is to say, information which was "material to the contract." The test of materiality is contained in Section 31 of the Insurance Code of 1978 which reads: Sec. 31. Materially is to be determined not by the event, but solely by the probable and reasonable influence of the facts upon the party to whom the

EH 403 Page 17

⦿ 2010-2011 ⦿ 2011-2012 ⦿

INSURANCE LAW CASE DIGESTS
communication is due, in forming his estimate of the disadvantages of the proposed contract, or in making his inquiries. The information which Jaime Canilang failed to disclose was material to the ability of Great Pacific to estimate the probable risk he presented as a subject of life insurance. Had Canilang disclosed his visits to his doctor in the insurance application, it may be reasonably assumed that Great Pacific would have made further inquiries and would have probably refused to issue a non-medical insurance policy or, at the very least, required a higher premium for the same coverage. The materiality of the information withheld by Great Pacific did not depend upon the state of mind of Jaime Canilang. A man's state of mind or subjective belief is not capable of proof in our judicial process, except through proof of external acts or failure to act from which inferences as to his subjective belief may be reasonably drawn. Neither does materiality depend upon the actual or physical events which ensue. Materiality relates rather to the "probable and reasonable influence of the facts" upon the party to whom the communication should have been made, in assessing the risk involved in making or omitting to make further inquiries and in accepting the application for insurance; that "probable and reasonable influence of the facts" concealed must, of course, be determined objectively, by the judge ultimately. The insurance Great Pacific applied for was a "nonmedical" insurance policy. In Saturnino v. PhilippineAmerican Life Insurance Company, this Court held that: .. . if anything, the waiver of medical examination [in a non-medical insurance contract] renders even more material the information required of the applicant concerning previous condition of health and diseases suffered, for such information necessarily constitutes an important factor which the insurer takes into consideration in deciding whether to issue the policy or not . . . The Insurance Code of 1978 was amended by B.P. Blg. 874. This subsequent statute modified Section 27 of the Insurance Code of 1978 so as to read as follows: Sec. 27. A concealment whether intentional or unintentional entitles the injured party to rescind a contract of insurance. The Commissioner is wrong when it said that by deleting the phrase "intentional or unintentional," the Insurance Code of 1978 (prior to its amendment by B.P. Blg. 874) intended to limit the kinds of concealment which generate a right to rescind on the part of the injured party to "intentional concealments." "Intentional" and "unintentional" cancel each other out. The deletion of the phrase "whether intentional or unintentional" could not have had the effect of imposing an affirmative requirement that a concealment must be intentional if it is to entitle the injured party to rescind a contract of insurance. The restoration in 1985 by B.P. Blg. 874 of the phrase "whether intentional or unintentional" merely underscored the fact that all throughout (from 1914 to 1985), the statute did not require proof that concealment must be "intentional" in order to authorize rescission by the injured party. The nature of the facts not conveyed to the insurer was such that the failure to communicate must have been intentional rather than merely inadvertent. For Jaime Canilang could not have been unaware that his heart beat would at times rise to high and alarming levels and that he had consulted a doctor twice in the 2 months before applying for non-medical insurance. The last medical consultation took place just the day before the insurance application was filed. Jaime Canilang went to visit his doctor precisely because of the discomfort and concern brought about by his experiencing "sinus tachycardia." We find it difficult to take seriously the argument that Great Pacific had waived inquiry into the concealment by issuing the insurance policy notwithstanding Canilang's failure to set out answers to some of the questions in the insurance application. Such failure precisely constituted concealment on the part of Canilang. Petitioner's argument, if accepted, would obviously erase Section 27 from the Insurance Code of 1978. It remains only to note that the Court of Appeals finding that the parties had not agreed in the pretrial before the Insurance Commission that the relevant issue was whether or not Jaime Canilang had intentionally concealed material information from the insurer, was supported by the evidence of record, i.e., the Pre-trial Order itself dated 17 October 1984 and the Minutes of the Pre-trial Conference dated 15 October 1984, which "readily shows that the word "intentional" does not appear in the statement or definition of the issue in the said Order and Minutes." WHEREFORE, the Petition for Review is DENIED for lack of merit and the Decision of the Court of is AFFIRMED. BERNARDO ARGENTE vs. WEST COAST LIFE INSURANCE CO. Facts: On February 9, 1925, Bernardo Argente signed an application for joint insurance with his wife in the sum of P2,000. The wife, Vicenta de Ocampo, signed a like application for the same policy. Both applications, with the exception of the names and the signatures of the applicants, were written by Jose Geronimo del Rosario, an agent for the West Coast Life Insurance Co. But all the information contained in the applications was furnished the agent by Bernardo Argente. Pursuant to this application, Bernado Agente and Vicenta De Ocampo underwent a medical examination on February 10, 1925 which was recorded in the Medical Examiner’s Report.

EH 403 Page 18

⦿ 2010-2011 ⦿ 2011-2012 ⦿

INSURANCE LAW CASE DIGESTS
On May 9, 1925, Bernardo Argente and his wife submitted to the West Coast Life Insurance Co. an amended application for insurance, increasing the amount thereof to P15,000, and asked that the policy be dated May 15, 1925. The amended application was accompanied by the documents entitled "Short Form Medical Report." In both of these documents appear certain questions and answers. On November 18, 1925, Vicenta de Ocampo died of cerebral apoplexy. Thereafter Bernardo Argente presented a claim in due form to the West Coast Life Insurance Co. for the payment of the sum of P15,000 the amount of the joint life Insurance policy. In an investigation conducted by the Manger of the Manila office of the insurance company, it was found out the the answers given by the insured in their medical examinations with regard to their health and previous illness and medical attention were untrue. On May 25, 1926, wrote him to the effect that the claim was rejected because the insurance was obtained through fraud and misrepresentation. Issue: WON the alleged concealment was immaterial and insufficient to avoid the policy? Held: One ground for the rescission of a contract of insurance under the Insurance Act is "a concealment," which in section 25 is defined as "A neglect to communicate that which a party knows and ought to communicate." In an action on a life insurance policy where the evidence conclusively shows that the answers to questions concerning diseases were untrue, the truth of falsity of the answers becomes the determining factor. In the policy was procured by fraudulent representations, the contract of insurance apparently set forth therein was never legally existent. It can fairly be assumed that had the true facts been disclosed by the assured, the insurance would never have been granted. The basis of the rule vitiating the contract in case of concealment is that it misleads or deceives the insurer into accepting the risk, or accepting it at the rate of premium agreed upon. The insurer, relying upon the belief that the assured will disclose every material within his actual or presumed knowledge, is misled into a belief that the circumstance withheld does not exist, and he is thereby induced to estimate the risk upon a false basis that it does not exist. It therefore follows that the assurer in assuming a risk is entitled to know every material fact of which the assured has exclusive or peculiar knowledge, as well as all material facts which directly tend to increase the hazard or risk which are known by the assured, or which ought to be or are presumed to be known by him. And a concealment of such facts vitiates the policy. If the assured has exclusive knowledge of material facts, he should fully and fairly disclose the same, whether he believes them material or not. But notwithstanding this general rule it will not infrequently happen, especially in life risks, that the assured may have a knowledge actual or presumed of material facts, and yet entertain an honest belief that they are not material. The determination of the point whether there has or has not been a material concealment must rest largely in all cases upon the form of the questions propounded and the exact terms of the contract. IGNACIO SATURNINO vs. THE PHILIPPINE AMERICAN LIFE INSURANCE COMPANY [G.R. No. L-16163, 28 February 1963] FACTS: Estefania Saturnino obtained a 20-year endowment non-medical insurance. This kind of policy dispenses with the medical examination of the applicant usually required in ordinary life policies. However, two months prior to the issuance of the policy, Saturnino was operated for cancer, involving mastectomy of the right breast. She did not make a disclosure in her application for insurance. On the contrary, she stated that she did not have, nor had she ever had, among other ailments listed in the application, cancer or other tumors. Sometime after, Saturnino died of pneumonia, secondary to influenza. Appellants, who are her surviving husband and minor child, demanded payment of the face value of the policy. The claim was rejected and hence an action was subsequently instituted. ISSUE: Whether or not the insured made such false representations of material facts as to avoid the policy HELD: YES. The Insurance Law provides that “materiality is to be determined not by the event, but solely by the probable and reasonable influence of the facts upon the party to whom the communication is due, in forming his estimate of the proposed contract, or in making his inquiries.” The waiver of medical examination renders even more material the information required of the applicant concerning previous condition of health and diseases suffered, for such information necessarily constitutes an important factor which the insurer takes into consideration in deciding whether to issue the policy or not. It is logical to assume that if Estefania had been properly appraised of the insured’s medical history she would at least have been made to undergo medical examination in order to determine her insurability. Concealment, whether intentional or unintentional, entitles the insurer to rescind the contract of insurance, concealment being defined as “negligence to communicate that which an Insurance Law case party knows and ought to communicate”. The basis of

EH 403 Page 19

⦿ 2010-2011 ⦿ 2011-2012 ⦿

INSURANCE LAW CASE DIGESTS
the rule vitiating the contract in cases of concealment is that it misleads or deceives the insurer into accepting the risk, or accepting it at the rate of premium agreed upon. The insurer, relying upon the belief that the assured will disclose every material fact within his actual or presumed knowledge, is misled into a belief that the circumstance withheld does not exist, and he is thereby induced to estimate the risk upon a false basis that it does not exist. The judgment appealed from, dismissing the complaint and awarding the return to appellants of the premium already, paid, with interest at 6% up to January 29, 1959, affirmed, with costs against appellants. E. Representation Definition, Interpretation (Sections 3639) ii. What representation qualifies or not (Section 40) iii. Sections 41-44 i. Digested by: Gestopa, Gevina V. Insular Life Ass. Co. vs. Pineda Facts: The insured, a fisherman, in his application answered “no” to question of whether he drank beer or other intoxicants, whether he ever suffered from any ailment of the lungs and whether he had ever spat blood. Subsequently, the insured signed a statement that he did drink beer, but very seldom, that he had a chronic cough for many years and that he had experienced a “few sputums, slightly bloody. Ruling: The policy is valid. We do not consider the allegedly conflicting statements as misrepresentations of material facts. That the insured drank beer but “very seldom” is almost similar to “not at all.” We are not aware of the non-insurability of a person affected with a chronic cough. With respect to the “few sputums slightly bloody,” we consider the statement as not inconsistent with not having spat blood, since traces of blood in insured’s sputum could be due to some temporary and unimportant ailments as a bloody tooth, a sore throat or a bad cold. iv. Misrepresentation, effect (Section 45) Digested by: Roxanne A. Huyo Eguaras v. Great Eastern Life Ass. Co. Facts: On April 14, 1913, counsel for Francisca Eguaras filed a written complaint in the said Laguna court, alleging as a cause of action that about October 14, 1912, her son-in-law Dominador Albay had applied in writing to the defendant insurance company to insure his life for the sum of P5,000, naming as the beneficiary in case of his death the plaintiff Francisca Eguaras; that after compliance with the requisites and the investigation carried on by the defendant company, and it had been satisfied concerning the physical condition of the applicant, it accepted the application for insurance and on November 6, 1912, issued policy No. 5592, Exhibit A, which has been made a part of the complaint, whereby the said insurance company insured the life of the said Dominador Albay in the sum of P5,000, payable in the event of his death to Francisca Eguaras; that on December 6, 1912, said policy No. 5592 being in force, the insured Dominador Albay, died in the municipality of Santa Cruz, Laguna, and despite the fact that the beneficiary submitted satisfactory proofs of his death and that the defendant company investigated the event, still it refused and continues to refuse to pay to the plaintiff the value of the policy, Exhibit A, thereby causing damages estimated at P1,000. The court was therefore asked to render judgment against the Great Eastern Life Assurance Company, Ltd., and its general agent, West G. Smith, by sentencing them to pay to the plaintiff the sum of P5,000, the value of policy No. 5592, plus the sum of P1,000 for damages inflicted upon them, in addition to the costs of the suit. The demurrer filed to the foregoing complaint having been overruled, counsel for the insurance company and for West G. Smith replied thereto, admitting the allegations of the complaint with respect to the legal status of the parties by denying all the rest, and setting forth in special defense that the insurance policy issued in the name of Dominador [Albay] had been obtained through fraud and deceit known and consented to by the interested parties and is therefore completely illegal, void, and ineffective; wherefore he prayed that the defendants be absolved from the complaint, with the costs against the plaintiff. Issue: WON the life insurance obtained is legal and valid or whether on the contrary it was issued through fraud and deceit, and in such case, whether the defendant, The Great Eastern Life Assurance Company, Ltd., is still under obligation to pay the value thereof to the plaintiff. Ruling: It appears from the record that the insured had knowledge of the false replied contained in the two applications for insurance and knowing permitted fraud to be practised upon the insurance company, for in his acknowledgment and consent his mother-in-law was designated as the beneficiary of the insurance, despite the fact that he had children and his mother was still living. In the present case the fraud consisted in the

EH 403 Page 20

⦿ 2010-2011 ⦿ 2011-2012 ⦿

INSURANCE LAW CASE DIGESTS
fact that a healthy and robust person was substituted in place of insured invalid when Dr. Vidal made the physical examination of the one who seeking to be insured, for the real person who desired to be insured and who ought to have been examined was in bad health on and before the date of executing the insurance contract of which facts the insured Dominador Albay and the insurance agent Ponciano Remigio had full knowledge. It is therefore proven that the signatures on the insurance applications reading "Dominador Albay" are false and forged; that the person who presented himself to Dr. Vidal to be examined was not the real Dominador Albay, but another different person; that at the time of the application for insurance and the issuance of the policy which is the subject matter of this suit the real Dominador Albay was informed of all those machinations, wherefore it is plain that the insurance contract between the defendant and Dominador Albay is null and void because it is false, fraudulent and illegal. Article 1269 of the Civil Code states: There is deceit when by words or insidious machinations on the part of one of the contracting parties the other is induced to execute a contract which without them he would not have made. It is essential to the nature of the deceit, to which the foregoing article refers, that said deceit be prior to or contemporaneous with the consent that is a necessary requisite for perfecting the contract, but not that it may have occurred or happened thereafter. A contract is therefore deceitful, for the execution whereof the consent of one of the parties has been secured by means of fraud, because he was persuaded by words or insidious machinations, statements or false promises, and a defective consent wrung from him, even though such do not constitute estafa or any other criminal subject to the penal law. With this array of circumstantial evidence derived from facts duly proven as a result of the present suit, we get, if not a moral certainly, at least a full conviction that when Castor Garcia presented himself to be examined by the physician Vidal in place of Dominador Albay, serious deceit occurred in perfecting the insurance contract, for had the agent of the company not been deceived it would not have granted the insurance applied for by Albay, nor would it have executed the contract by virtue of whereof payment is claimed of the value of policy obtained through fraud; and consequently on such assumptions it is improper, nor is it permitted by the law, to order collection of the amount claimed. In a contract executed with the requisites fixed in article 1261, one of the contracting parties may have given his consent through error, violence, intimidation, or deceit, and in any of such cases the contract is void, even though, despite this nullity, no crime was committed. (Article 1265, Civil Code.) There may not have been estafa in the case at bar, but it was conclusively demonstrated by the trial that deceit entered into the insurance contract, fulfillment whereof is claimed, and therefore the conclusions reached by the court in the judgment it rendered in the criminal proceedings for estafa do not affect this suit, nor do they influence the decision proper herein, nor can they produce in the present suit, over the exception of the defendant, the force of res adjudicata. Digested by: Patricia Ko Bernardo Argente vs. West Coast Life Insurance [March 19, 1928, G.R. No. L-24899] FACTS: Bernardo Argente and his wife Vicenta de Ocampo signed an application for joint life insurance for P2,000, later amended to P15,000. All the information contained in the applications was furnished to the agent by Bernardo Argente. Pursuant to their applications, Bernardo and Vicenta were examined separately by Dr. Cesareo Sta. Ana, a medical examiner for the West Coast Life Insurance Co., The information or answers to the questions contained on the face of the Medical Examiner's Report were furnished to the doctor by the applicants, Bernardo and Vicente Argente. Later, Vicenta died of cerebral apoplexy. Bernardo presented a claim for the payment of the sum of P15,000 the amount of the joint life Insurance policy. After investigation it was discovered that the answers given by the insured in their medical examinations with regard to their health and previous illness and medical attendance were untrue. For that reason, the West Coast Life Insurance Co. refused to pay the claim of Bernardo Argente. It is admitted that it appears in the Medical Examiner's Report that Bernardo Argente, in response to the question asked by the medical examiner, "Have you ever consulted a physician for, or have you ever suffered from any ailment or disease of, the brain or nervous system?" answered "No." To the question, "Have you consulted a physician for any ailment or disease not included in your above answer," answered "Yes. Nature of Ailment, Disease or Injury. Scabies, Number of attacks 1, Date 1911. Duration 1 month, Severity Fair, results and, if within five years, name and address of every physician consulted. Dr. P. Guazon. Cured. Dr. Guazon is dead now." And to the question, "What physician or physicians, if any, not named above, have you consulted or been treated by, within the last five years and for what illness or ailment? (If none, so state)" answered "No." It is, however, not disputed that on January 10, 11, and 13, 1923, Bernardo Argente was confined in the Philippine General Hospital where he was treated by Dr. Agerico B. M. Sison for cerebral congestion and Bell's Palsy. And that Vicenta de Ocampo, in response to the question asked by the medical examiner, "How frequently, if at all, and in what quantity do you use

EH 403 Page 21

⦿ 2010-2011 ⦿ 2011-2012 ⦿

INSURANCE LAW CASE DIGESTS
beer, wine, spirits or other intoxicants?" answered "Beer only in small quantities occasionally." To the question, "Have you ever consulted a physician for or have you ever suffered from any ailment or disease of the brain or nervous system?" answered "No." To the question, "What physician or physicians, if any, not named above, have you consulted or been treated by, within the last five years and for what illness or ailment? (If none, so state)" answered "None." And to the question, "Are you in good health as far as you know and believe?" answered "Yes." It is, however, not disputed that Vicenta de Ocampo was taken by a patrolman, at the request of her husband, Bernardo Argente, on May 19, 1924, to the Meisic police station, and from there was transferred to the San Lazaro Hospital. In San Lazaro Hospital, her case was diagnosed by the admitting physician as "alcoholism," but later Doctor Domingo made a diagnosis of probable "manic-depressive psychosis," and still, later in Mary Chiles Hospital, made a final diagnosis of "phychoneurosis." ISSUE: was there false misrepresentation? RULING: Yes, there was misrepresentation giving the insurer a right to rescind the contract. The court found from the evidence that the representations made by Bernardo Argente and his wife in their applications to the defendant for life insurance were false with respect to their estate of health during the period of five years preceding the date of such applications, and that they knew the representations made by them in their applications were false. In the policy was procured by fraudulent representations, the contract of insurance apparently set forth therein was never legally existent. It can fairly be assumed that had the true facts been disclosed by the assured, the insurance would never have been granted. And it is declared that if a material fact concealed by assured it is equivalent to a false representation that it does not exist and that the essentials are the truth of the representations whether they were intended to mislead and did insurer accept them as true and act upon them to his prejudice. So it is decided that under a stipulation voiding the policy for concealment or misrepresentation of any material fact or if his interest is not truly stated or is either than the sole and unconditional ownership the facts are unimportant that insured did not intend to deceive or withhold information as to encumbrances even though no questions were asked. x x x x x x x x x The basis of the rule vitiating the contract in case of concealment is that it misleads or deceives the insurer into accepting the risk, or accepting it at the rate of premium agreed upon. The insurer, relying upon the belief that the assured will disclose every material within his actual or presumed knowledge, is misled into a belief that the circumstance withheld does not exist, and he is thereby induced to estimate the risk upon a false basis that it does not exist. The principal question, therefore, must be, Was the assurer misled or deceived into entering a contract obligation or in fixing the premium of insurance by a withholding of material information of facts within the assured's knowledge or presumed knowledge? It therefore follows that the assurer in assuming a risk is entitled to know every material fact of which the assured has exclusive or peculiar knowledge, as well as all material facts which directly tend to increase the hazard or risk which are known by the assured, or which ought to be or are presumed to be known by him. And a concealment of such facts vitiates the policy. "It does not seem to be necessary . . . that the . . . suppression of the truth should have been willful." If it were but an inadvertent omission, yet if it were material to the risk and such as the plaintiff should have known to be so, it would render the policy void. But it is held that if untrue or false answers are given in response to inquiries and they relate to material facts the policy is avoided without regard to the knowledge or fraud of assured, although under the statute statements are representations which must be fraudulent to avoid the policy. So under certain codes the important inquiries are whether the concealment was willful and related to a matter material to the risk. THE INSULAR LIFE ASSURANCE CO., LTD. v. SERAFIN D. FELICIANO ET AL. [G.R. No. L-47593 December 29, 1943] Insular Life filed this case (Motion for reconsideration) Facts: Evaristo Feliciano, who died on September 29, 1935, was suffering with advanced pulmonary tuberculosis when he signed his applications for insurance with the petitioner on October 12, 1934. On that same date Doctor Trepp, who had taken X-ray pictures of his lungs, informed the respondent Dr. Serafin D. Feliciano, brother of Evaristo, that the latter "was already in a very serious ad practically hopeless condition." Nevertheless the question contained in the application — "Have you ever suffered from any ailment or disease of the lungs, pleurisy, pneumonia or asthma?" — appears to have been answered , "No".

EH 403 Page 22

⦿ 2010-2011 ⦿ 2011-2012 ⦿

INSURANCE LAW CASE DIGESTS
False answer above referred to, as well as the others, was written by the Company's soliciting agent David, in collusion with the medical examiner Dr. Gregorio Valdez, for the purpose of securing the Company's approval of the application so that the policy to be issued thereon might be credited to said agent in connection with the inter-provincial contest which the Company was then holding among its soliciting agents to boost the sales of its policies. Agent David bribed Medical Examiner Valdez with money which the former borrowed from the applicant's mother by way of advanced payment on the premium, according to the finding of the Court of Appeals. Said court also found that before the insured signed the application he, as well as the members of his family, told the agent and the medical examiner that he had been sick and coughing for some time and that he had gone three times to the Santol Sanatorium and had X-ray pictures of his lungs taken; but that in spite of such information the agent and the medical examiner told them that the applicant was a fit subject for insurance. The policy and the application constitute the entire contract between the parties hereto. Said contract provides “xxx and the Company shall not be bound by any promise or representation heretofore or hereafter given by any person other than the above-named officials, and by them only in writing and signed conjointly as stated” Issue: Whether or not Insular Life is bound by the acts of its agents? Whether or not Feliciano is entitled to the insurance benefits? Held: NO. Sain unsurance policy is void ab initio. Insular Life is not bound by the acts of its agents, as in this case there was connivance between the medical examiner, the agent and Feliciano.From the facts of the case we cannot escape the conclusion that the insured acted in connivance with the soliciting agent and the medical examiner of the Company in accepting the policies in question." By accepting the policy Feliciano became charged with knowledge of its contents, whether he actually read it or not. The insured, therefore, had no right to rely — and we cannot believe he relied in good faith — upon the oral representation of said agent and medical examiner that he (the applicant) was a fit subject for insurance notwithstanding that he had been and was still suffering with advanced pulmonary tuberculosis. We are to conclude that the insured was a co-participant, and co-responsible with Agent David and Medical Examiner Valdez, in the fraudulent procurement of the policies in question and that by reason thereof said policies are void ab initio. SC: motion for reconsideration is sustained and the judgment of the Court of Appeals is hereby reversed. Let another judgment be entered in favor of the respondents and against the petitioner for the refund of the premiums amounting to P1,389, with legal interest thereon from the date of the complaint, and without any finding as to costs. Digested by: dlanor Saturnino vs. Philippine American Life Insurance Facts: A 20- year endowment non-medical insurance was issued to Estefania Saturnino after submission of her application to the appelle ( Philippine American Life Insurance) and payment of P339.25 as the first year’s premium. The policy dispenses with the medical examination of the applicant usually required in other ordinary life policies. Nevertheless, it required for a detailed information in the application concerning the applicant’s health medical history. It was discovered that prior to the issuance of the policy, Saturnino was operated on for cancer, involving complete removal of the right breast, including the pectoral muscles and the glands found in the right armpit. Estefania Saturnino however did not the disclose the fact of operation in the application. She even stated therein that she did not have, nor had she ever had, among other ailments listed in the application, cancer or other tumors; that she had not consulted any physician, undergone any operation or suffered any injury within the preceding five years; and that she had never been treated for nor did she ever have any illness or disease peculiar to her sex, particularly of the breast, ovaries, uterus, and menstrual disorders. The application also recites that the foregoing declarations constituted "a further basis for the issuance of the policy." Saturnino died of pneumonia, prompting the husband and minor child to demand payment of the face value of the policy. The claim was rejected and this suit was subsequently instituted. Issues: W/N the insured made such false representations of material facts to avoid the policy. Ruling: There can be no dispute that the information given by her in her application for insurance was false, namely, that she had never had cancer or tumors, or consulted any physician or undergone any operation within the preceding period of five years. The Insurance Law (Section 30) provides that "materiality is to be determined not by the event, but solely by the probable and reasonable influence of the facts upon the party to whom the communication is due, in forming his estimate of the proposed contract, or in making his inquiries." It seems to be the contention of appellants that the facts subject of the representation were not material in

EH 403 Page 23

⦿ 2010-2011 ⦿ 2011-2012 ⦿

INSURANCE LAW CASE DIGESTS
view of the "non-medical" nature of the insurance applied for, which does away with the usual requirement of medical examination before the policy is issued. The waiver of medical examination renders even more material the information required of the applicant concerning previous condition of health and diseases suffered, for such information necessarily constitutes an important factor which the insurer takes into consideration in deciding whether to issue the policy or not. Appellants also contend there was no fraudulent concealment of the truth inasmuch as the insured herself did not know, since her doctor never told her, that the disease for which she had been operated on was cancer. In the first place the concealment of the fact of the operation itself was fraudulent, as there could not have been any mistake about it, no matter what the ailment. Secondly, in order to avoid a policy it is not necessary to show actual fraud on the part of the insured. In this jurisdiction a concealment, whether intentional or unintentional, entitles the insurer to rescind the contract of insurance, concealment being defined as "negligence to communicate that which a party knows and ought to communicate". v. Materiality of representation (Section 46) vi. Rules on concealment and representation (Section 47) vii. Right of rescission/Incontestability (Section 48) Digested by: Kristine Oja Tan Chay Hong v. West Coast Life Ins Co. Facts: In 1925, defendant West Coast Life Insurance Company accepted and approved a life insurance policy for the sum of P10,000 in favor of Tan Caeng in which the plaintiff (Tan Chay Heng) was the sole beneficiary. Tan Ceang died on May 10, 1925 and in June, 1925, Tan Chay Heng submitted the proofs of the death of Tan Ceang with a claim for the payment of the policy. West Coast refused to pay and in consequence thereof Tan Chay Heng prayed before the court for judgment, with legal interest from the date of the policy, and costs. West Coast filed an answer in which it made a general and specific denial on the allegations in the complaint. West Coast further filed an amended answer, alleging special defenses that the insurance policy on the life of Tan Ceang, upon which Tan Chay Heng’s action is based, was obtained in confabulation with one Go Chulian and Dr. V. S. Locsin, thru fraud and deceit .West Coast said that it was made to appear that Tan Caeng was single, a merchant, of good health condition and not a drug user, when in fact he was married, a laborer, suffering from tuberculosis and addicted to drugs. West Coast now denies liability based on these misrepresentations and prays that it be absolved from plaintiff's complaint. To this special defense, Tan Chay Heng, claiming that it was a cross-complaint, filed a general demurrer upon the ground that it does not state facts sufficient to constitute a cause of defense. Tan Chay further contends that section 47 of the Insurance Act should be applied, and that when so applied, defendant is barred and estopped to plead and set forth the matters alleged in its special defense. That section is as follows: “ Whenever a right to rescind a contract of insurance is given to the insurer by any provision of this chapter, such right must be exercised previous to the commencement of an action on the contract.” The Trial court rendered judgment in favor of Tan Chay Heng holding that an insurer cannot avoid a policy which has been procured by fraud unless he brings an action to rescind it before he is sued thereon. Issue: WON Section 47 can be applied to the matters alleged in the special defense of Tan Chay Heng. Held: NO. Precisely, the defense of West Coast was that the policy was acquired through fraud in its execution, the policy is void ab initio, and therefore, no valid contract was ever made. Its action then cannot be for rescission because an action to rescind is founded upon and presupposes the existence of the contract. Hence, West Coast’s defense is not barred by Sec. 47. In the instant case, it will be noted that even in its prayer, the defendant does not seek to have the alleged insurance contract rescinded. It denies that it ever made any contract of insurance on the life of Tan Caeng, or that any such a contract ever existed, and that is the question which it seeks to have litigated by its special defense. In the very nature of things, if the defendant never made or entered into the contract in question, there is no contract to rescind, and, hence, section 47 upon which the lower court based its decision in sustaining the demurrer does not apply. As stated, an action to rescind a contract is founded upon and presupposes the existence of the contract which is sought to be rescinded. If all of the material matters set forth and alleged in the defendant's special plea are true, there was no valid contract of insurance, for the simple reason that the minds of the parties never met and never agreed upon the terms and conditions of the contract. We are clearly of the opinion that, if such matters are known to exist by a preponderance of the evidence, they would constitute a valid defense to plaintiff's cause of action. Upon the

EH 403 Page 24

⦿ 2010-2011 ⦿ 2011-2012 ⦿

INSURANCE LAW CASE DIGESTS
question as to whether or not they are or are not true, we do not at this time have or express any opinion, but we are clear that section 47 does not apply to the allegations made in the answer, and that the trial court erred in sustaining the demurrer. The judgment of the lower court is reversed and the case is remanded for such other and further proceedings as are not inconsistent with this opinion, with costs against the plaintiff. -Argente Considering that the insured died before the two-year period had lapsed, respondent company is not, therefore, barred from proving that the policy is void ab initio by reason of the insured's fraudulent concealment or misrepresentation. Moreover, respondent company rescinded the contract of insurance and refunded the premiums paid on September 11, 1975, previous to the commencement of this action on November 27,1975. The deceased, by affixing his signature on the application form, affirmed the correctness of all the entries and answers appearing therein. It is but to be expected that he, a businessman, would not have affixed his signature on the application form unless he clearly understood its significance. For, the presumption is that a person intends the ordinary consequence of his voluntary act and takes ordinary care of his concerns. [Sec. 5(c) and (d), Rule 131, Rules of Court]. The evidence for respondent company shows that on September 19,1972, the deceased was examined by Dr. Victoriano Lim and was found to be diabetic and hypertensive; that by January, 1973, the deceased was complaining of progressive weight loss and abdominal pain and was diagnosed to be suffering from hepatoma, (t.s.n. August 23, 1976, pp. 8-10; Exhibit 2). Another physician, Dr. Wenceslao Vitug, testified that the deceased came to see him on December 14, 1973 for consolation and claimed to have been diabetic for five years. (t.s.n., Aug. 23,1976, p. 5; Exhibit 6) Because of the concealment made by the deceased of his consultations and treatments for hypertension, diabetes and liver disorders, respondent company was thus misled into accepting the risk and approving his application as medically standard (Exhibit 5- C) and dispensing with further medical investigation and examination (Exhibit 5-A). For as long as no adverse medical history is revealed in the application form, an applicant for insurance is presumed to be healthy and physically fit and no further medical investigation or examination is conducted by respondent company F. Insurance Policy Definition (Section 49)

Author: Aldrin Montesco TAN VS. CA [174 SCRA 403] FACTS: Businessman Tan Lee Siong applied for life insurance policy with American Life Insurance Company in the amount of P80,000.00 by virtue of which he was issued Policy No. 1082467 effective November 6,1973, with his sons as designated beneficiaries thereof. On April 1975, Tan Lee Siong died and the sons subsequently filed their claim for the insurance proceeds. But in September of the same year, the company sent a letter denying petitioners' claim and rescinded the policy by reason of the alleged misrepresentation and concealment of material facts made by the deceased Tan Lee Siong in his application for insurance. The premiums paid on the policy, however, were thereupon refunded. ISSUES: Whether or not the insurance company no longer had the right to rescind the contract of insurance as rescission must allegedly be done during the lifetime of the insured within two years and prior to the commencement of the action. RULING: The so-called "incontestability clause" precludes the insurer from raising the defenses of false representations or concealment of material facts insofar as health and previous diseases are concerned if the insurance has been in force for at least two years during the insured's lifetime. The phrase "during the lifetime" found in Section 48 simply means that the policy is no longer considered in force after the insured has died. The key phrase in the second paragraph of Section 48 is "for a period of two years." As noted by the Court of Appeals, to wit: The policy was issued on November 6, 1973 and the insured died on April 26,1975. The policy was thus in force for a period of only one year and five months.

i.

Digested by: Archie Necesario Vicente Tang vs Court of Appeals, Philippine American Life Insurance Company Facts: Lee See Guat, a widow, 61 years old and an illeterate who spoke only Chinese, applied on two separate times for an insurance on her life from Philippine American Life Insurance Co. (Company), amounting to a total of P100,000. The application consisted of two parts, both in the English language. The second part of her application dealt with her state of health and because her answers indicated that she was healthy,

EH 403 Page 25

⦿ 2010-2011 ⦿ 2011-2012 ⦿

INSURANCE LAW CASE DIGESTS
the Company approved her two applications, with her nephew Vicente Tang as her benefeciary. Less than a year from her application, Lee See Guat died of lung cancer. Tang tried to claim the value of the insurance policy but the Company refused to pay on the ground that the insured was guilty of concealment and misrepresentation at the time she applied for the two policies. Both the trial court and the Court of Appeals ruled in favor of the insurance company. Issue: Whether or not Article 1332 of the Civil Code is applicable to this case. Ruling: "Art. 1332. When one of the parties is unable to read, or if the contract is in a language not understood by him, and mistake or fraud is alleged, the person enforcing the contract must show that the terms thereof have been fully explained to the former." It is the position of the petitioner that because Lee See Guat was illiterate and spoke only Chinese, she could not be held guilty of concealment of her health history because the applications for insurance were in English and the insurer has not proved that the terms thereof had been fully explained to her. It should be noted that under Art. 1332, the obligation to show that the terms of the contract had been fully explained to the party who is unable to read or understand the language of the contract, when fraud or mistake is alleged, devolves on the party seeking to enforce it. Here the insurance company is not seeking to enforce the contracts; on the contrary, it is seeking to avoid their performance. It is petitioner who is seeking to enforce them even as fraud or mistake is not alleged. Accordingly, respondent company was under no obligation to prove that the terms of the insurance contracts were fully explained to the other party. Even if we were to say that the insurer is the one seeking the performance of the contracts by avoiding paying the claim, it has to be noted as above stated that there has been no imputation of mistake or fraud by the illiterate insured whose personality is represented by her beneficiary the petitioner herein. In sum, Art. 1332 is inapplicable to the case at bar. Decision of the Court of Appeals is affirmed. On July 6, 1917, Luis Lim y Garcia of Zamboanga applied for a policy of insurance on his life (Sun Life Assurance Company of Canada) in the sum of P5,000. His wife, Pilar C. de Lim, plaintiff, was the beneficiary. The first premium of P433 was paid by Lim, and upon such payment the company issued what was called a "provisional policy." Provisional Policy: The above-mentioned life is to be assured in accordance with the terms and conditions contained or inserted by the Company in the policy which may be granted by it in this particular case for four months only from the date of the application, provided that the Company shall confirm this agreement by issuing a policy on said application when the same shall be submitted to the Head Office in Montreal. Should the Company not issue such a policy, then this agreement shall be null and void ab initio, and the Company shall be held not to have been on the risk at all, but in such case the amount herein acknowledged shall be returned. Luis Lim y Garcia died on August 23, 1917, after the issuance of the provisional policy but before approval of the application by the home office of the insurance company. The instant action is brought by the beneficiary, Pilar C. de Lim, to recover from the Sun Life Assurance Company of Canada the sum of P5,000, the amount named in the provisional policy. Court of First Instance of Zamboanga sustained the demurrer to plaintiff's complaint on the ground that it fails to state a cause of action. Issues: (1) WON the contract was consummated? (2) WON this is a form of cover notes? (note: not 100% sure about this  ) Ruling: (1) NO. The contract, to be binding from the date of the application, must have been a completed contract, one that leaves nothing to be done, nothing to be completed, nothing to be passed upon, or determined, before it shall take effect. There can be no contract of insurance unless the minds of the parties have met in agreement. As we read and understand the so-called provisional policy it amounts to nothing but an acknowledgment on behalf of the company, that it has received from the person named therein the sum of money agreed upon as the first year's premium upon a policy to be issued upon the application, if the application is accepted by the company. SC finds the 2nd rule (Joyce on Insurance, relied by appellant) to be applicable . The Rule states; “Where an agreement is made between the applicant and the agent whether by signing an application containing such condition, or otherwise, that no liability shall attach until the principal approves the risk and a

ii. Form

of insurance Policy; etc)/Contents (Sections 50-51) iii. Cover notes (Section 52)

riders,

Digested by: Sharmine M. Odchigue De Lim vs. Sun Life Ass. Co. of Canada [41 Phil 263] Facts:

EH 403 Page 26

⦿ 2010-2011 ⦿ 2011-2012 ⦿

INSURANCE LAW CASE DIGESTS
receipt is given buy the agent, such acceptance is merely conditional, and it subordinated to the act of the company in approving or rejecting; so in life insurance a "binding slip" or "binding receipt" does not insure of itself.” (2) NO. Sec 52. Cover notes may be issued to bind insurance temporarily pending the issuance of the policy. In this case there is no temporary insurance. Cases which the COURT find applicable to this case: In the case of Steinle vs. New York Life Insurance Co. ([1897], 81 Fed., 489} the facts were that the amount of the first premium had been paid to an insurance agent and a receipt given therefor. The receipt, however, expressly declared that if the application was accepted by the company, the insurance shall take effect from the date of the application but that if the application was not accepted, the money shall be returned. The trite decision of the circuit court of appeal was, "On the conceded facts of this case, there was no contract to life insurance perfected and the judgment of the circuit court must be affirmed." In the case of Cooksey vs. Mutual Life Insurance Co. ([1904], 73 Ark., 117) the person applying for the life insurance paid and amount equal to the first premium, but the application and the receipt for the money paid, stipulated that the insurance was to become effective only when the application was approved and the policy issued. The court held that the transaction did not amount to an agreement for preliminary or temporary insurance. SC did not find any error on the part of trial court in sustaining the demurrer and dismissing the case. Lastly, counsel for appellee admits the liability of the company for the return of the first premium to the estate of the deceased. Sun Life Assurance Company of Canada will immediately pay to the estate of the late Luis Lim y Garcia the of P433. SO ORDERED. Digested by: Kristine Oja Pacific Timber Export Corp. v. CA Facts: On March 13, 1963, Pacific Timber secured temporary insurance from the Workmen’s Insurance Co. for its exportation of logs to Japan. Workmen Insurance issued on said date Cover Note 1010 insuring said cargo. The regular marine policies were issued by the company in favor of Pacific Timber on April 2, 1963. The two marine policies bore the number 53H01032 and 53H01033. After the issuance of the cover note but before the issuance of the two policies, some of the logs intended to be exported were lost due to a typhoon. Pacific Timber filed its claim with the company, but the latter refused, contending that the cover note insuring the cargo is null and void for lack of valuable consideration. Issue: WON the cover note was without consideration, thus null and void. Held: It was with consideration. We uphold petitioner's submission that the Cover Note was not without consideration. The fact that no separate premium was paid on the Cover Note before the loss insured against occurred, does not militate against the validity of petitioner's contention, for no such premium could have been paid, since by the nature of the Cover Note, it did not contain, as all Cover Notes do not contain particulars of the shipment that would serve as basis for the computation of the premiums. As a logical consequence, no separate premiums are intended or required to be paid on a Cover Note. At any rate, it is not disputed that petitioner paid in full all the premiums as called for by the statement issued by private respondent after the issuance of the two regular marine insurance policies, thereby leaving no account unpaid by petitioner due on the insurance coverage, which must be deemed to include the Cover Note. If the Note is to be treated as a separate policy instead of integrating it to the regular policies subsequently issued, the purpose and function of the Cover Note would be set at naught or rendered meaningless, for it is in a real sense a contract, not a mere application for insurance which is a mere offer. iv. Application (Section 53) of insurance proceeds

Digested by: Gayle Opsima FRANCISCO DEL VAL ET AL.vs. ANDRES DEL VAL Facts: Plaintiff and defendant are siblings who are heirs at law of Gregorio Nacianceno del Val, who died intestate. The deceased, during his lifetime, took out an insurance on his life for the sum of P40,000 and made it payable to the defendant as sole beneficiary. Plaintiffs contend that the amount of the insurance policy belonged to the estate of the deceased and not to the defendant personally; that, therefore, they are entitled to a partition not only of the real and personal property, but also of the P40,000 life insurance. Issue: Whether or not the insurance policy belongs to the estate Ruling: With the finding of the trial court that the proceeds of the life-insurance policy belong exclusively to the

EH 403 Page 27

⦿ 2010-2011 ⦿ 2011-2012 ⦿

INSURANCE LAW CASE DIGESTS
defendant as his individual and separate property, we agree. That the proceeds of an insurance policy belong exclusively to the beneficiary and not to the estate of the person whose life was insured, and that such proceeds are the separate and individual property of the beneficiary, and not of the heirs of the person whose life was insured, is the doctrine in America. We believe that the same doctrine obtains in these Islands by virtue of section 428 of the Code of Commerce, which reads: "The amounts which the underwriter must deliver to the person insured, in fulfillment of the contract, shall be the property of the latter, even against the claims of the legitimate heirs or creditors of any kind whatsoever of the person who effected the insurance in favor of the former." It is claimed by the attorney for the plaintiffs that the proceeds of the insurance policy were a donation or gift made by the father during his lifetime to the defendant and that, as such, its ultimate destination is determined by those provisions of the Civil Code which relate to donations, especially article 819. This article provides that "gifts made to children which are not betterments shall be considered as part of their legal portion." We cannot agree with these contentions. The contract of life insurance is a special contract and the destination of the proceeds thereof is determined by special laws which deal exclusively with that subject. The Civil Code has no provisions which relate directly and specifically to life-insurance contracts or to the destination of life insurance proceeds. That subject is regulated exclusively by the Code of Commerce which provides for the terms of the contract, the relations of the parties and the destination of the proceeds of the policy. Digested by: Kenn Bonifacio Bros. vs Mora Facts: • • Enrique Mora mortgaged his car to the H.S. Reyes with the condition that the former would insure it with the latter as beneficiary. He insured it with the State Bonding & Insurance Co., Inc., with pertinent provisions of the policy which read: xxx xxx xxx 4. The Insured may authorize the repair of the Motor Vehicle necessitated by damage for which the Company may be liable under this Policy provided that: — (a) The estimated cost of such repair does not exceed the Authorized Repair Limit, (b) A detailed estimate of the cost is forwarded to the Company without delay, subject to the condition that "Loss, if any is payable to H.S. Reyes, Inc.," by virtue of the fact that said Oldsmobile sedan was mortgaged in favor of the said H.S. Reyes, Inc. and that under a clause in said insurance policy, any loss was made payable to the H.S. Reyes, Inc. as Mortgagee; xxx xxx xxx During the effectivity of the insurance contract, the car met with an accident. The insurance company then assigned the accident to the Bayne Adjustment Co. for appraisal of the damage. Enrique Mora, without the knowledge and consent of the H.S. Reyes, Inc., authorized the Bonifacio Bros. Inc. to furnish the labor and materials, some of which were supplied by the Ayala Auto Parts Co. The insurance company then drew a check as proceeds of the insurance policy, payable to the order of Enrique Mora or H.S. Reyes, Inc., and entrusted the check to the H.H. Bayne Adjustment Co. for disposition and delivery to the proper party. In the meantime, the car was delivered to Enrique Mora without the consent of the H.S. Reyes, Inc., and without payment to the Bonifacio Bros. Inc. and the Ayala Auto Parts Co. of the cost of repairs and materials. Upon the theory that the insurance proceeds should be paid directly to them, the Bonifacio Bros. Inc. and the Ayala Auto Parts Co. filed this complaint. Their arguments are based on paragraph 4 of the insurance contract which provides that "the insured may authorize the repair of the Motor Vehicle necessitated by damage for which the company may be liable under the policy xxx." It is stressed that the H.H. Bayne Adjustment Company's recommendation of payment of the appellants' bill for which the Insurance Company drew a check indicates that Mora and the H.H. Bayne Adjustment Co. acted for and in representation of that insurance company.









Issue: Whether or not there is privity of contract between the Bonifacio Bros. Inc. and the Ayala Auto Parts Co. on the one hand and the insurance company on the other. Who has better right over the insurance proceeds? Ruling: The appellants are not mentioned in the contract as parties thereto nor is there any clause or provision thereof from which to infer that there is an obligation on the part of the insurance company to pay the cost of repairs directly to them. It is fundamental that contracts take effect only between the parties thereto, except in some specific instances provided by law where the contract contains some stipulation in favor of a third person. Such stipulation is known as stipulation pour autrui or a provision in favor of a third person not a pay to the contract. Under this doctrine, a third person is allowed to avail himself of a benefit granted to him by the terms of the contract, provided that the contracting parties have clearly and deliberately conferred a favor upon such person. Consequently, a third person not a party to the contract has no action against the parties thereto,

EH 403 Page 28

⦿ 2010-2011 ⦿ 2011-2012 ⦿

INSURANCE LAW CASE DIGESTS
and cannot generally demand the enforcement of the same. The question of whether a third person has an enforcible interest in a contract, must be settled by determining whether the contracting parties intended to tender him such an interest by deliberately inserting terms in their agreement with the avowed purpose of conferring a favor upon such third person. In this connection, SC has laid down the rule that the fairest test to determine whether the interest of a third person in a contract is a stipulation pour autrui or merely an incidental interest, is to rely upon the intention of the parties as disclosed by their contract. In the instant case the insurance contract does not contain any words or clauses to disclose an intent to give any benefit to any repairmen or materialmen in case of repair of the car in question. The parties to the insurance contract omitted such stipulation, which is a circumstance that supports the said conclusion. On the other hand, the "loss payable" clause of the insurance policy stipulates that "Loss, if any, is payable to H.S. Reyes, Inc." indicating that it was only the H.S. Reyes, Inc. which they intended to benefit. Another cogent reason for not recognizing a right of action by the appellants against the insurance company is that "a policy of insurance is a distinct and independent contract between the insured and insurer, and third persons have no right either in a court of equity, or in a court of law, to the proceeds of it, unless there be some contract of trust, expressed or implied between the insured and third person."5 In this case, no contract of trust, expressed or implied exists. This conclusion is deducible not only from the principle governing the operation and effect of insurance contracts in general, but is clearly covered by the express provisions of section 50 of the Insurance Act which read: The insurance shall be applied exclusively to the proper interests of the person in whose name it is made unless otherwise specified in the policy. The policy in question has been so framed that "Loss, if any, is payable to H.S. Reyes, Inc.," which unmistakably shows the intention of the parties. Digested by: ra osorio Guingon vs. Del Monte FACTS: Julio Aguilar owned and operated several jeepneys. He entered into a contract with the Capital Insurance & Surety Co., Inc. insuring the operation of his jeepneys against accidents with third-party liability. The insurance policy contains the following provisions: Section II —LIABILITY TO THE PUBLIC 1. The Company, will, subject to the limits of liability, indemnify the Insured in the event of accident caused by or arising out of the use of the Motor Vehicle/s or in connection with the loading or unloading of the Motor Vehicle/s, against all sums including claimant's costs and expenses which the Insured shall become legally liable to pay in respect of: a. death of or bodily injury to any person b. damage to property Iluminado del Monte, one of the drivers of the jeepneys operated by Aguilar, bumped with the jeepney abovementioned one Gervacio Guingon who had just alighted from another jeepney and as a consequence the latter died some days thereafter. The heirs of Gervacio Guingon filed an action for damages praying that the sum of P82,771.80 be paid to them jointly and severally by the defendants, driver Iluminado del Monte, owner and operator Julio Aguilar, and the Capital Insurance & Surety Co., Inc. For failure to answer the complaint, Del Monte and Aguilar were declared in default. Capital Insurance & Surety Co., Inc. answered, alleging that the plaintiff has no cause of action against it based on the following provision: E. Action Against Company No action shall lie against the Company unless, as a condition precedent thereto, the Insured shall have fully complied with all of the terms of this Policy, nor until the amount of the Insured's obligation to pay shall have been finally determined either by judgment against the Insured after actual trial or by written agreement of the Insured, the claimant, and the Company. Any person or organization or the legal representative thereof who has secured such judgment or written agreement shall thereafter be entitled to recover under this policy to the extent of the insurance afforded by the Policy. Nothing contained in this policy shall give any person or organization any right to join the Company as a co-defendant in any action against the Insured to determine the Insured's liability. Bankruptcy or insolvency of the Insured or of the Insured's estate shall not relieve the Company of any of its obligations hereunder. Capital Insurance & Surety Co. contends that the "no action" clause in the policy closes the avenue to any third party which may be injured in an accident wherein the jeepney of the insured might have been the cause of the injury of third persons, alleging the freedom of contracts. ISSUE: WON the heirs of Guingon have a cause of action against Capital Insurance RULING: Yes. The policy in the present case is one whereby the insurer agreed to indemnify the insured "against all sums . . . which the Insured shall become legally liable to pay in respect of: a. death of or bodily injury to any person . . . ." Clearly, therefore, it is one for indemnity against liability; from the fact then that the insured is

EH 403 Page 29

⦿ 2010-2011 ⦿ 2011-2012 ⦿

INSURANCE LAW CASE DIGESTS
liable to the third person, such third person is entitled to sue the insurer The right of the person injured to sue the insurer of the party at fault (insured), depends on whether the contract of insurance is intended to benefit third persons also or only the insured. And the test applied has been this: Where the contract provides for indemnity against liability to third persons, then third persons to whom the insured is liable, can sue the insurer. The next question is on the right of the third person to sue the insurer jointly with the insured. The policy requires, as afore-stated, that suit and final judgment be first obtained against the insured; that only "thereafter" can the person injured recover on the policy; it expressly disallows suing the insurer as a codefendant of the insured in a suit to determine the latter's liability. As adverted to before, the query is which procedure to follow — that of the insurance policy or the Rules of Court. In the discussion of the points thus raised, what is paramount is the interpretation of the insurance contract with the aim in view of attaining the objectives for which the insurance was taken. The Rules of Court provide that parties may be joined either as plaintiffs or defendants, as the right to relief in respect to or arising out of the same transactions is alleged to exist (Sec. 6, Rule 3). The "no action" clause in the policy of insurance cannot prevail over the Rules of Court provision aimed at avoiding multiplicity of suits. Similarly, in the instant suit, Sec. 5 of Rule 2 on "Joinder of causes of action" and Sec. 6 of Rule 3 on "Permissive joinder of parties" cannot be superseded, at least with respect to third persons not a party to the contract, as herein, by a "no action" clause in the contract of insurance. Digested by: Mark Recto MELECIO COQUIA, MARIA ESPANUEVA and MANILA YELLOW TAXICAB CO., INC.vs.FIELDMEN'S INSURANCE CO., INC. [G.R. No. L-23276, November 29, 1968] Facts: Fieldmen’s Insurance Company issued Manila Yellow Taxicab Co. a common carrier accident insurance policy, covering the period from Dec. 1, 1961 to Dec. 1, 1962. While the policy was in force, or on February 10, 1962, a taxicab of the Insured, driven by Carlito Coquia, met a vehicular accident at Mangaldan, Pangasinan, in consequence of which Carlito died. The Insured filed therefor a claim for P5,000.00 to which the Company replied with an offer to pay P2,000.00, by way of compromise. The Insured rejected the same and made a counter-offer for P4,000.00, but the Company did not accept it. Hence, on September 18, 1962, the Insured and Carlito's parents, namely, Melecio Coquia and Maria Espanueva —the Coquias — filed a complaint against the Company to collect the proceeds of the aforementioned policy. In its answer, the Company admitted the existence thereof, but pleaded lack of cause of action on the part of the plaintiffs. The trial court rendered a decision sentencing the Company to pay to the plaintiffs the sum of P4,000.00 and the costs. Issue: Fieldmen appealed, it contended that plaintiffs have no cause of action because: 1) the Coquias have no contractual relation with the Company; and 2) the Insured has not complied with the provisions of the policy concerning arbitration. (FIRST ISSUE IS the MAIN TOPIC) Ruling: As regards the first defense, it should be noted that, although, in general, only parties to a contract may bring an action based thereon, this rule is subject to exceptions, one of which is found in the second paragraph of Article 1311 of the Civil Code of the Philippines, reading: If a contract should contain some stipulation in favor of a third person, he may demand its fulfillment provided he communicated his acceptance to the obligor before its revocation. A mere incidental benefit or interest of a person is not sufficient. The contracting parties must have clearly and deliberately conferred a favor upon a third person. In the stipulations, the Company "will indemnify any authorized Driver who is driving the Motor Vehicle" of the Insured and, in the event of death of said driver, the Company shall, likewise, "indemnify his personal representatives." In fact, the Company "may, at its option, make indemnity payable directly to the claimants or heirs of claimants ... it being the true intention of this Policy to protect ... the liabilities of the Insured towards the passengers of the Motor Vehicle and the Public" — in other words, third parties. Thus, the policy under consideration is typical of contracts pour autrui, this character being made more manifest by the fact that the deceased driver paid fifty percent (50%) of the corresponding premiums, which were deducted from his weekly commissions. Under these conditions, it is clear that the Coquias — who, admittedly, are the sole heirs of the deceased — have a direct cause of action against the Company, and, since they could have maintained this action by themselves, without the assistance of the Insured, it goes without saying that they could and did properly join the latter in filing the complaint herein. The second defense set up by the Company is based upon Section 17 of the policy reading: If any difference or dispute shall arise with respect to the amount of the Company's liability under this

EH 403 Page 30

⦿ 2010-2011 ⦿ 2011-2012 ⦿

INSURANCE LAW CASE DIGESTS
Policy, the same shall be referred to the decision of a single arbitrator to be agreed upon by both parties or failing such agreement of a single arbitrator, to the decision of two arbitrators, one to be appointed in writing by each of the parties within one calendar month after having been required in writing so to do by either of the parties and in case of disagreement between the arbitrators, to the decision of an umpire who shall have been appointed in writing by the arbitrators before entering on the reference and the costs of and incident to the reference shall be dealt with in the Award. And it is hereby expressly stipulated and declared that it shall be a condition precedent to any right of action or suit upon this Policy that the award by such arbitrator, arbitrators or umpire of the amount of the Company's liability hereunder if disputed shall be first obtained. The record shows, however, that none of the parties to the contract invoked this section, or made any reference to arbitration, during the negotiations preceding the institution of the present case. In fact, counsel for both parties stipulated, in the trial court, that none of them had, at any time during said negotiations, even suggested the settlement of the issue between them by arbitration, as provided in said section. Their aforementioned acts or omissions had the effect of a waiver of their respective right to demand an arbitration. In the decision of the Supreme Court of Minnesota in Independent School Dist. No. 35, St. Louis County vs. A. Hedenberg & Co., Inc.7 from which we quote: The test for determining whether there has been a waiver in a particular case is stated by the author of an exhaustive annotation in 117 A.L.R. p. 304, as follows: "Any conduct of the parties inconsistent with the notion that they treated the arbitration provision as in effect, or any conduct which might be reasonably construed as showing that they did not intend to avail themselves of such provision, may amount to a waiver thereof and estop the party charged with such conduct from claiming its benefits". xxx xxx xxx The decisive facts here are that both parties from the inception of their dispute proceeded in entire disregard of the provisions of the contract relating to arbitration and that neither at any stage of such dispute, either before or after commencement of the action, demanded arbitration, either by oral or written demand, pleading, or otherwise. Their conduct was as effective a rejection of the right to arbitrate as if, in the best Coolidge tradition, they had said, "We do not choose to arbitrate". As arbitration under the express provisions of article 40 was "at the choice of either party," and was chosen by neither, a waiver by both of the right to arbitration followed as a matter of law. WHEREFORE, the decision appealed from should be as it is hereby affirmed in toto, with costs against the herein defendant-appellant, Fieldmen's Insurance Co., Inc. It is so ordered. v. Sections 54-59 vi. Open Policy (Section 60) Digested by: George Development Insurance Corporation v.s. IAC and PHILIPPINE UNION REALTY DEVELOPMENT CORPORATION [G.R. No. 71360 July 16, 1986] Facts: A fire broke out in the building of the private respondent and it sued for recovery of damages from the petitioner on the basis of an insurance contract between them. The private respondent's building was insured by petitioner against fire for P2,500,000.00 under an open policy, Policy No. RY/F-082. Petitioner refused to pay advancing the argument that only the building and not the elevators were insured. Note: Actually, the petitioner in this case already lost through default. The SC considered its delay inexcusable, however, it proceeded to discuss the merits of the case. Issue: WON the respondent is entitled to indemnity. Ruling: The petitioner's claim that the insurance covered only the building and not the elevators is absurd, to say the least. This Court has little patience with puerile arguments that affront common sense, let alone basic legal principles with which even law students are familiar. The circumstance that the building insured is seven stories high and so had to be provided with elevators-a legal requirement known to the petitioner as an insurance company-makes its contention all the more ridiculous. No less preposterous is the petitioner's claim that the elevators were insured after the occurrence of the fire, a case of shutting the barn door after the horse had escaped, so to speak. 4 This pretense merits scant attention. Equally undeserving of serious consideration is its submission that the elevators were not damaged by the fire, against the report of The arson investigators of the INP 5 and, indeed, its own expressed admission in its answer 6 where it affirmed that the fire "damaged or destroyed a portion of the 7th floor of the insured building and more particularly a Hitachi elevator control panel." There is no reason to disturb the factual findings of the lower court, as affirmed by the Intermediate Appellate Court, that the heat and moisture caused by the fire damaged although they did not actually burn the

EH 403 Page 31

⦿ 2010-2011 ⦿ 2011-2012 ⦿

INSURANCE LAW CASE DIGESTS
elevators. Neither is this Court justified in reversing their determination, also factual, of the value of the loss sustained by the private respondent in the amount of P508,867.00. The only remaining question to be settled is the amount of the indemnity due to the private respondent under its insurance contract with the petitioner. This will require an examination of this contract, Policy No. RY/F-082, as renewed, by virtue of which the petitioner insured the private respondent's building against fire for P2,500,000.00. The petitioner argues that since at the time of the fire the building insured was worth P5,800,000.00, the private respondent should be considered its own insurer for the difference between that amount and the face value of the policy and should share pro rata in the loss sustained. Accordingly, the private respondent is entitled to an indemnity of only P67,629.31, the rest of the loss to be shouldered by it alone. In support of this contention, the petitioner cites Condition 17 of the policy, which provides: If the property hereby insured shall, at the breaking out of any fire, be collectively of greater value than the sum insured thereon then the insured shall be considered as being his own insurer for the difference, and shall bear a ratable proportion of the loss accordingly. Every item, if more than one, of the policy shall be separately subject to this condition. However, there is no evidence on record that the building was worth P5,800,000.00 at the time of the loss; only the petitioner says so and it does not back up its self-serving estimate with any independent corroboration. On the contrary, the building was insured at P2,500,000.00, and this must be considered, by agreement of the insurer and the insured, the actual value of the property insured on the day the fire occurred. This valuation becomes even more believable if it is remembered that at the time the building was burned it was still under construction and not yet completed. The Court notes that Policy RY/F-082 is an open policy and is subject to the express condition that: Open Policy This is an open policy as defined in Section 57 of the Insurance Act. In the event of loss, whether total or partial, it is understood that the amount of the loss shall be subject to appraisal and the liability of the company, if established, shall be limited to the actual loss, subject to the applicable terms, conditions, warranties and clauses of this Policy, and in no case shall exceed the amount of the policy. As defined in the aforestated provision, which is now Section 60 of the Insurance Code, "an open policy is one in which the value of the thing insured is not agreed upon but is left to be ascertained in case of loss. " This means that the actual loss, as determined, will represent the total indemnity due the insured from the insurer except only that the total indemnity shall not exceed the face value of the policy. The actual loss has been ascertained in this case and, to repeat, this Court will respect such factual determination in the absence of proof that it was arrived at arbitrarily. There is no such showing. Hence, applying the open policy clause as expressly agreed upon by the parties in their contract, we hold that the private respondent is entitled to the payment of indemnity under the said contract in the total amount of P508,867.00. vii. Valued Policy (Section 61) Digested by: Mark Rossel Salomon Harding vs. Commercial Union Ass. Co. Facts: Henry Harding bought an automobile which was subsequently given to her wife as a gift. The said automobile was repaired and repainted at the Luneta Garage. Luneta Garage, acting as agent for Smith, Bell & Company (limited), solicited of the plaintiff Mrs. Harding the insurance of said automobile by the Commercial Union Assurance Company. A proposal was made, followed by an inspection as to thevakue of the automobile and thereafter Commercial Union Assurance Company issued a policy of insurance upon said proposal., in which policy the said automobile was described as of the 'present value' of P3,000. Subsequently, the destroyed by fire. Issue: W/N there was a proper valuation in the insurance policy. Ruling: There was a proper valuation. The ordinary test of the value of property is the price it will commend in the market if offered for sale. But that test cannot, in the very nature of the case, be applied at the time application is made for insurance. Men may honestly differ about the value of property, or as to what it will bring in the market; and such differences are often very marked among those whose special business it is to buy and sell property of all kinds. The assured could do no more than estimate such value; and that, it seems, was all that he was required to do in this case. His duty was to deal fairly with the Company in making such estimate. The special finding shows that he discharged that duty and observed good faith. We shall not presume that the Company, after requiring the assured in his application to give the 'estimated value,' and then to covenant that he had stated all material facts in regard to such value, so far as known to him, and after carrying that covenant, by express words, into the written contract, intended to said automobile was totally

EH 403 Page 32

⦿ 2010-2011 ⦿ 2011-2012 ⦿

INSURANCE LAW CASE DIGESTS
abandon the theory upon which it sought the contract, and make the absolute correctness of such estimated value a condition precedent to any insurance whatever. The application, with its covenant and stipulations, having been made a part of the policy, that presumption cannot be indulged without imputing to the Company a purpose, by studied intricacy or an ingenious framing of the policy, to entrap the assured into incurring obligations which, perhaps, he had no thought of assuming." Section 163 of the Insurance Law (Act No. 2427) provides that "the effect of a valuation in a policy of fire insurance is the same as in a policy of marine insurance." By the terms of section 149 of the Act cited, the valuation in a policy of marine insurance is conclusive if the insured had an insurable interest and was not guilty of fraud. viii. Running Policy (Section 62) ix. Limitation to commence action (Section 63) Digested by: Jamee S. Singco Eagle Star Ins. vs. Chia Yu Facts: Atkin, Kroll & Co., loaded on the S. S. Roeph Silverlight owned and operated by Leigh Hoegh & Co., A/S, of San Francisco California, 14 bales of assorted underwear valued at P8,085.23 consigned to Chia Yu in the City of Manila. The shipment was insured against all risks by Eagle Star Ins. Co. of San Francisco, California, under a policy issued to the shipper and by the latter assigned to the consignee. When the vessel started discharging its cargo into the custody of the Manila Terminal Co., Inc., which was then operating the arrastre service for the Bureau of Customs the 14 bales consigned to Chia Yu, only 10 were delivered to him as the remaining 3 could not be found. Three of those delivered were also found damaged to the extent of 50 per cent. Chia Yu claimed indemnity for the missing and damaged bales. But the claim was declined, first, by the carrier and afterward by the insurer, whereupon Chia Yu brought the present action against both, including their respective agents in the Philippines. More than two years after delivery of the damaged bales and the date when the missing bales should have been delivered, the action was resisted by the defendants principally on the ground of prescription. The trial court found for plaintiff and rendered judgment in his favor for the sum claimed plus legal interest and costs. The judgment was affirmed by the Court of Appeals. Issue: whether plaintiff's action has prescribed Ruling: On the part of the carrier the defense of prescription is made to rest on the following stipulation of the bill of lading: In any event the carrier and the ship shall be discharged from all liability in respect of loss or damage unless suit is brought within one year after the delivery of the goods or the date when the goods should have been delivered. The stipulation is but a repetition of a provision contained in section 3 (6) of the United States Carriage of Goods by Sea, Act of 1936, which was adopted and made applicable to the Philippines by Commonwealth Act 65 and by express agreement incorporated by reference in the bill of lading. The case for the insurer stands on a different footing, for its claim of prescription is founded upon the terms of the policy and not upon the bill of lading. Under our law the time limit for bringing a civil action upon a written contract is ten years after the right of action accrues. (Sec. 43, Act 190; Art. 1144, New Civil Code.) But counsel for the insurer claim that this statutory in the policy: No suit action on this Policy, for the recovery of any claim, shall be sustainable in any Court of law or equity unless the insured shall have fully complied with all the terms and conditions of this Policy nor unless commenced with twelve (12) months next after the happening of the loss . . . To this the court cannot agree. Examining the policy sued upon in the present case, we find that its prescriptive clause, if given effect in accordance with the terms of the policy, would reduce the period allowed the insured for bringing his action to less than one year. This is so because the said clause makes the prescriptive period begin from the happening of the loss and at the same time provides that the no suit on the policy shall be sustainable in any court unless the insured shall have first fully complied with all the terms and conditions of the policy, among them that which requires that, as so as the loss is determined, written claim therefor be filed with the carrier and that the letter to the carrier and the latter's reply should be attached to the claim papers to be sent to the insurer. It is obvious that compliance with this condition precedent will necessarily consume time and thus shorten the period for bringing suit to less than one year if the period is to begin, as stated in the policy, from "the happening of the loss." Being contrary to the law of the forum, such stipulation cannot be given effect. When did the cause of action accrue? On that question we agree with the court below that plaintiff's cause of action did not accrue until his claim was finally rejected by the insurance company. This is because, before such final rejection, there was no real necessity for bringing suit. As the policy provides that the insured should file his claim, first, with the carrier and then

EH 403 Page 33

⦿ 2010-2011 ⦿ 2011-2012 ⦿

INSURANCE LAW CASE DIGESTS
with the insurer, he had a right to wait for his claim to be finally decided before going to court. The law does not encourages unnecessary litigation. In concluding, we may state that contractual limitations contained in insurance policies are regarded with extreme jealousy by courts and will be strictly construed against the insurer and should not be permitted to prevent a recovery when their just and honest application would not produce that result. Wherefore, the judgment appealed from is reversed with respect to the carrier and its agents but affirmed with respect to the insurance company and its agents, with costs against the latter. Digested by: Patrina T. Soco Ang vs. Fulton Fire Insurance Co. FACTS: On September 9, 1953, defendant Fulton Fire Insurance Company issued a policy No. F-4730340, in favor of P. & S Department Store (Sally C. Ang) over stocks of general merchandise, consisting principally of dry goods, contained in a building occupied by the plaintiffs at Laoag, Ilocos Norte. The insurance was issued for one year, but the same was renewed for another year on September 31, 1954. On December 17, 1954, the store containing the goods insured was destroyed by fire. On December 30, following, plaintiffs executed the first claim form. On April 6, 1956, the Fulton Fire Insurance Company wrote the plaintiffs that their claim was denied. This denial of the claim was received by the plaintiffs on April 19, 1956. Defendant claims that under paragraph 13 of the policy, if the loss or damage is occasioned by the willful act of the insured, or if the claim is made and rejected but no action is commenced within 12 months after such rejection, all benefits under the policy would be forfeited, and that since the claim of the plaintiffs was denied and plaintiffs received notice of denial on April 18, 1956, and they brought the action only on May 5, 1958, all the benefits under the policy have been forfeited. The court below held that the bringing of the action in the Court of First Instance of Manila on May 11, 1956 to assert their claim, tolled the running of the 12 month period within which the action must be filed. It said that even if the case was filed against the agent rather than the defendant, it was merely a procedural mistake. The complaint was dismissed by the Court without prejudice on September 3, 1957, and motion for reconsideration dated September 21, 1957. The instant complaint was filed on May 8, 1958 therefore it is still within the one-year prescriptive period. ISSUE: Whether the filing of the previous suit tolled or suspended the running of the prescriptive period. RULING: The condition contained in the insurance policy that claims must be presented within one year after rejection is not merely a procedural requirement. The condition is an important matter, essential to a prompt settlement of claims against insurance companies, as it demands that insurance suits be brought by the insured while the evidence as to the origin and cause of destruction have not yet disappeared. It is in the nature of a condition precedent to the liability of the insurer, or in other terms, a resolutory cause, the purpose of which is to terminate all liabilities in case the action is not filed by the insured within the period stipulated. The bringing of the action against the Paramount Surety & Insurance Company, the agent of the defendant Company cannot have any legal effect except that of notifying the agent of the claim. Beyond such notification, the filing of the action can serve no other purpose. There is no law giving any effect to such action upon the principal. Besides, there is no condition in the policy that the action must be filed against the agent, and this Court cannot by interpretation, extend the clear scope of the agreement beyond what is agreed upon by the parties. Decision of RTC reversed. Sun Insurance vs CA [G.R. No. 89741 March 13, 1991] Facts: Herein private respondent Emilio Tan took from petitioner, Sun Insurance, a P300,000.00 property insurance policy to cover his interest in the electrical supply store of his brother housed in a building in Iloilo City. Four (4) days after the issuance of the policy, the building was burned including the insured store. On August 20, 1983, Tan filed his claim for fire loss with petitioner, but on February 29, 1984, petitioner wrote Tan denying the latter's claim. On April 3, 1984, Tan wrote petitioner, seeking reconsideration of the denial of his claim. On September 3, 1985, Tan's counsel wrote the Insurer inquiring about the status of his April 3, 1984 request for reconsideration. Petitioner answered the letter on October 11, 1985, advising Tan's counsel that the Insurer's denial of Tan's claim remained unchanged, enclosing copies of petitioners' letters of February 29, 1984 and May 17, 1985 (response to petition for reconsideration). On November 20, 1985 filed a case with the RTC of Iloilo, but petitioner filed a motion to dismiss on the alleged ground that the action had already prescribed. Said motion was denied; and petitioner's motion for reconsideration was also denied. CA also denied his appeal. Thus the instant petition. Issue: WON the RTC and CA is correct in ruling against Sun Insurance when it filed a motion to dismiss Emilio

EH 403 Page 34

⦿ 2010-2011 ⦿ 2011-2012 ⦿

INSURANCE LAW CASE DIGESTS
Tan’s complaint on the ground that petitioner’s action has already prescribed. Corollary issue: WON that MOTION FOR RECONSIDERATION filed by Tan on April 3, 1984 INTERRUPTS THE TWELVE (12) MONTHS PRESCRIPTIVE PERIOD TO CONTEST THE DENIAL OF THE INSURANCE CLAIM. Held: The petition was considered and the CA decision, which affirms RTC decision, was reversed and set aside. SC ruled that the MR filed by Tan on April 1984 did not interrupt the twelve months prescriptive period. While it is a cardinal principle of insurance law that a policy or contract of insurance is to be construed liberally in favor of the insured and strictly against the insurer company, yet, contracts of insurance, like other contracts, are to be construed according to the sense and meaning of the terms which the parties themselves have used. If such terms are clear and unambiguous, they must be taken and understood in their plain, ordinary and popular sense. Condition 27 of the Insurance Policy, which is the subject of the conflicting contentions of the parties, reads: 27. Action or suit clause — If a claim be made and rejected and an action or suit be not commenced either in the Insurance Commission or in any court of competent jurisdiction within twelve (12) months from receipt of notice of such rejection, or in case of arbitration taking place as provided herein, within twelve (12) months after due notice of the award made by the arbitrator or arbitrators or umpire, then the claim shall for all purposes be deemed to have been abandoned and shall not thereafter be recoverable hereunder. Respondent Tan, in his letter addressed to the petitioner insurance company dated April 3, 1984 (Rollo, pp. 50-52), admitted that he received a copy of the letter of rejection on April 2, 1984. Thus, the 12month prescriptive period started to run from the said date of April 2, 1984, for such is the plain meaning and intention of Section 27 of the insurance policy. The condition contained in an insurance policy that claims must be presented within one year after rejection is not merely a procedural requirement but an important matter essential to a prompt settlement of claims against insurance companies as it demands that insurance suits be brought by the insured while the evidence as to the origin and cause of destruction have not yet disappeared. It is apparent that Section 27 of the insurance policy was stipulated pursuant to Section 63 of the Insurance Code, which states that: Sec. 63. A condition, stipulation or agreement in any policy of insurance, limiting the time for commencing an action thereunder to a period of less than one year from the time when the cause of action accrues, is void. The insured's cause of action or his right to file a claim either in the Insurance Commission or in a court of competent jurisdiction commences from the time of the denial of his claim by the Insurer, either expressly or impliedly. But as pointed out by the petitioner insurance company, the rejection referred to should be construed as the rejection, in the first instance, for if what is being referred to is a reiterated rejection conveyed in a resolution of a petition for reconsideration, such should have been expressly stipulated. Thus, to allow the filing of a motion for reconsideration to suspend the running of the prescriptive period of twelve months, a whole new body of rules on the matter should be promulgated so as to avoid any conflict that may be brought by it. This Court uses the phrase "final rejection", the same cannot be taken to mean the rejection of a petition for reconsideration as insisted by respondents. Such was clearly not the meaning contemplated by this Court. The Insurance policy in said case provides that the insured should file his claim, first, with the carrier and then with the insurer. The "final rejection" being referred to in said case is the rejection by the insurance company. Digested by: Ugbinar, Krystel Marie ACCFA vs. Alpha Insurance and Surety Co. Inc. [G.R. No. L-24566 July 29, 1968] Facts: According to the allegations of the complaint, in order to guarantee the Asingan Farmers' Cooperative Marketing Association, Inc. (FACOMA) against loss on account of "personal dishonesty, amounting to larceny or estafa of its Secretary-Treasurer, Ricardo A. Ladines, the appellee, Alpha Insurance & Surety Company had issued, on 14 February 1958, its bond, No. P-FID-15-58, for the sum of Five Thousand Pesos (P5,000.00) with said Ricardo Ladines as principal and the appellee as solidary surety. On the same date, the Asingan FACOMA assigned its rights to the appellant, Agricultural Credit Cooperative and Financing Administration (ACCFA for short), with approval of the principal and the surety. During the effectivity of the bond, Ricardo Ladines converted and misappropriated, to his personal benefit, some P11,513.22 of the FACOMA funds, of which P6,307.33 belonged to the ACCFA. Upon discovery of the loss, ACCFA immediately notified in writing the survey company on 10 October 1958, and presented the proof of loss within the period fixed in the bond; but despite repeated demands the surety company refused and failed to pay. Whereupon, ACCFA filed suit against appellee on 30 May 1960.

EH 403 Page 35

⦿ 2010-2011 ⦿ 2011-2012 ⦿

INSURANCE LAW CASE DIGESTS
Defendant Alpha Insurance & Surety Co., Inc., (now appellee) moved to dismiss the complaint for failure to state a cause of action, giving as reason that (1) the same was filed more than one year after plaintiff made claim for loss, contrary to the eighth condition of the bond, providing as follows: . EIGHT LIMITATION OF ACTION No action, suit or proceeding shall be had or maintained upon this Bond unless the same be commenced within one year from the time of making claim for the loss upon which such action, suit or proceeding, is based, in accordance with the fourth section hereof. (2) the complaint failed to show that plaintiff had filed civil or criminal action against Ladines, as required by conditions 4 and 11 of the bond; and (3) that Ladines was a necessary and indispensable party but had not been joined as such. At first, the Court of First Instance denied dismissal; but, upon reconsideration, the court reversed its original stand, and dismissed the complaint on the ground that the action was filed beyond the contractual limitation period Issue: whether or not the provision of a fidelity bond that no action shall be had or maintained thereon unless commenced within one year from the making of a claim for the loss upon which the action is based, is valid or void, in view of Section 61-A of the Insurance Act invalidating stipulations limiting the time for commencing an action thereon to less than one year from the time the cause of action accrues. Ruling: We find the appeal meritorious. A fidelity bond is, in effect, in the nature of a contract of insurance against loss from misconduct, and is governed by the same principles of interpretation: Mechanics Savings Bank & Trust Co. vs. Guarantee Company, 68 Fed. 459; Pao Chan Wei vs. Nemorosa, 103 Phil. 57. Consequently, the condition of the bond in question, limiting the period for bringing action thereon, is subject to the provisions of Section 61-A of the Insurance Act (No. 2427), as amended by Act 4101 of the pre-Commonwealth Philippine Legislature, prescribing that: SEC. 61-A — A condition, stipulation or agreement in any policy of insurance, limiting the time for commencing an action thereunder to a period of less than one year from the time when the cause of action accrues is void. Since a "cause of action" requires, as essential elements, not only a legal right of the plaintiff and a correlative obligation of the defendant but also "an act or omission of the defendant in violation of said legal right" (Maao Sugar Central vs. Barrios, 79 Phil. 666), the cause of action does not accrue until the party obligated refuses, expressly or impliedly, to comply with its duty (in this case, to pay the amount of the bond). The year for instituting action in court must be reckoned, therefore, from the time of appellee's refusal to comply with its bond; it can not be counted from the creditor's filing of the claim of loss, for that does not import that the surety company will refuse to pay. In so far, therefore, as condition eight of the bond requires action to be filed within one year from the filing of the claim for loss, such stipulation contradicts the public policy expressed in Section 61-A of the Philippine Insurance Act. Condition eight of the bond, therefore, is null and void, and the appellant is not bound to comply with its provisions. As a consequence of the foregoing, condition eight of the Alpha bond is null and void, and action may be brought within the statutory period of limitation for written contracts (New Civil Code, Article 1144). The case of Ang vs. Fulton Fire Insurance Co., 2 S.C.R.A. 945 (31 July 1961), relied upon by the Court a quo, is no authority against the views herein expressed, since the effect of Section 61-A of the Insurance Law on the terms of the Policy or contract was not there considered. The condition of previous conviction (paragraph b, clause 4, of the contract) having been deleted by express agreement and the surety having assumed solidary liability, the other grounds of the motion to dismiss are equally untenable. A creditor may proceed against any one of the solidary debtors, or some or all of them simultaneously (Article 1216, New Civil Code). WHEREFORE, the appealed order granting the motion to dismiss is reversed and set aside, and the records are remanded to the Court of First Instance, with instructions to require defendant to answer and thereafter proceed in conformity with the law and the Rules of Court. Costs against appellee. So ordered. Digested by: Sheiryl G. Vinas Mayer Steel Pipe Corp vs. CA Facts: Hongkong Govt. contracted Mayer Steel to manufacture steel pipes. The pipes covered by 3 invoices were insured with South Sea Insurance for $212K, while the others under another 3 separate invoices where insured with Charter Insurance for $149K. They were issued an “all risks insurance policies”. When the pipes reached Hongkong, it was damaged, though it was certified in good condition before it was loaded in the ship. Mayer filed a claim with Charter but they were only given HK$49K and so they filed a case for the remainder of the claim in court. RTC found for Mayer, which was reversed by CA, on the ground of prescription. Issue:

EH 403 Page 36

⦿ 2010-2011 ⦿ 2011-2012 ⦿

INSURANCE LAW CASE DIGESTS
WON COGSA would apply, and therefore the cause of action has prescribed. Ruling: No. COGSA, does not apply and the cause of action will prescribe only after 10 years. COGSA applies to a claim against the carrier under the contract of carriage, which must be filed within 2 years from delivery of the goods. This is a claim against the insurer under the insurance contract and therefore will prescribe in 10 years. Respondent court erred in applying Section 3(6) of the Carriage of Goods by Sea Act. Section 3(6) of the Carriage of Goods by Sea Act states that the carrier and the ship shall be discharged from all liability for loss or damage to the goods if no suit is filed within one year after delivery of the goods or the date when they should have been delivered. Under this provision, only the carrier's liability is extinguished if no suit is brought within one year. But the liability of the insurer is not extinguished because the insurer's liability is based not on the contract of carriage but on the contract of insurance. The Filipino Merchants case is different from the case at bar. In Filipino Merchants, it was the insurer which filed a claim against the carrier for reimbursement of the amount it paid to the shipper. The ruling in Filipino Merchants should apply only to suits against the carrier filed either by the shipper, the consignee or the insurer. When the court said in Filipino Merchants that Section 3(6) of the Carriage of Goods by Sea Act applies to the insurer, it meant that the insurer, like the shipper, may no longer file a claim against the carrier beyond the one-year period provided in the law. But it does not mean that the shipper may no longer file a claim against the insurer because the basis of the insurer's liability is the insurance contract. An insurance contract is a contract whereby one party, for a consideration known as the premium, agrees to indemnify another for loss or damage which he may suffer from a specified peril . An "all risks" insurance policy covers all kinds of loss other than those due to willful and fraudulent act of the insured. 12 Thus, when private respondents issued the "all risks" policies to petitioner Mayer, they bound themselves to indemnify the latter in case of loss or damage to the goods insured. Such obligation prescribes in ten years, in accordance with Article 1144 of the New Civil Code. Granted. CA reversed. FILIPINO MERCHANTS INSURANCE CO., Inc. v. CA GR No. 85141 Nov. 28, 1989 Second Division [Regalado] Nature: Petition for review on certiorari challenging a CA decision finding FilMerchant liable to pay plaintiff. Facts: (oops.) This is the story of your mom’s life.

This is a story about a consignee/buyer who bought fishmeal products from Bangkok and had it delivered to the port of Manila. He entered into an insurance contract with defendant insurance company (FilMerchant) under policy no. M-2678 for P267,653.59 and for goods described as 600 metric tons of fishmeal in new gunny bags of 90 kilos each. What was actually imported was 59.940mtons in 666 gunny bags. Upon arrival at Manila, arrastre and defendant’s surveyor found 227 bags in bad order condition. Because of this loss, buyer formally claimed from FilMerchant but the said insurance company refused to pay. He brought suit. Trial court ruled for him and against FilMerchant, CA affirmed trial court hence this petition. FilMerchant argues: (1) CA erred in the interpretation and application of the “all risk” clause of maritime insurance policy. It says it should not be held liable for partial loss notwithstanding the clear absence of proof of some fortuitous event, casualty, or accidental cause to which the loss is attributable. (2) Respondent had no insurable interest in the subject cargo. The shipment reveals that it is a “C & F” contract of shipment. The seller, not the consignee, paid for the shipment. As there was yet no delivery to the consignee, ownership (and interest) does not yet pass to him. Issues: W/N CA was correct in its interpretation of the “all risk” clause in the maritime insurance contract. W/N the insured had insurable interest over the property insured. Ruling: a) “All risks policy” has no technical meaning. The clause in question reproduced: “5. This insurance is against all risks of loss or damage to the subject-matter insured but shall in no case be deemed to extend to cover loss, damage, or expense proximately caused by delay or inherent vice or nature of the subjectmatter insured. Claims recoverable hereunder shall be payable irrespective of percentage “

*In no event shall the publisher be held liable for any consequential damages arising out of or related to the use or inability to use this case brief material. INSURANCE LAW barcrammer_jed limitation to commence action/insurable interest in property

EH 403 Page 37

⦿ 2010-2011 ⦿ 2011-2012 ⦿

INSURANCE LAW CASE DIGESTS
An "all risks policy" should be read literally as meaning all risks whatsoever and covering all losses by an accidental cause of any kind. The very nature of the term "all risks" must be given a broad and comprehensive meaning as covering any loss other than a willful and fraudulent act of the insured. 7 This is pursuant to the very purpose of an "all risks" insurance to give protection to the insured in those cases where difficulties of logical explanation or some mystery surround the loss or damage to property. Burden of proof shifts to the insurer. Generally, the burden of proof is upon the insured to show that a loss arose from a covered peril, but under an "all risks" policy the burden is not on the insured to prove the precise cause of loss or damage for which it seeks compensation. The insured under an "all risks insurance policy" has the initial burden of proving that the cargo was in good condition when the policy attached and that the cargo was damaged when unloaded from the vessel; thereafter, the burden then shifts to the insurer to show the exception to the coverage. b) Vendee/Consignee has insurable interest SC: The shipment contract being that of cost and freight (C&F) is immaterial in the determination of insurable interest. The perfected contract of sale vests in the vendee an equitable title, an interest sufficient enough to be insurable. Further, Art. 1523 NCC provides that where, in pursuance of a contract of sale, the seller is authorized or required to send the goods to the buyer, delivery of the goods to a carrier, whether named by the buyer or not, for, the purpose of transmission to the buyer is deemed to be a delivery of the goods to the buyer, the exceptions to said rule not obtaining in the present case. The Court has heretofore ruled that the delivery of the goods on board the carrying vessels partake of the nature of actual delivery since, from that time, the foreign buyers assumed the risks of loss of the goods and paid the insurance premium covering them. The plaintiff did not return the policy nor de4manded for the return of the proportionate premium and neither did the defendant offer to breturn the premium. The property covered by the policy was destroyed by fire on aug. 15, 1952. The defendant refused to pay plaintiff’s claim on the ground that the policy was cancelled as of May 10, 1952. Plaintiff contends in this appeal that her letter, dated April 30, 1952, was a mere request orb offer to cancel the policy and did not terminate the same since it was not accompanied by the surrender of the policy for cancellation, Issue: Whether the policy as effectively cancelled or not. Held: This case hinges on the interpretation of paragraph 10 of the policy, reading: “This insurance may be terminated anytime at the request of the Insured, in which case the Company will retain the customary period rate for the time the policy has been enforced. This insurance may also at any time be terminated at the option of the Company, on notice to that effect being given to the insured, in which case the Company shall be liable to repay on demand a ratable proportion of the premium for the expired term from the date of cancellment” Pursuant to this stipulation,the contract in question could be terminated “at any time” upon the unilateral act of either party. Whichever party exercised the “option” did not need the approval. Consent nor concurrence of the other thereto, That consent was given at the making of the contract. Moreover, pursuant to her letter, plaintiff considered the contract terminated upon receipt of said letter by the defendant (deade el recibo de la presente’0, Decision of the lower court dismissing the4 action to recover the amount pf fire insurance policy is affirmed. (note: this ruling is deemed modigfied by Sec. 65} Saura Import & Export Co. vs. Phil. InterNational Surety Co., Inc. and Philippine National Bank [G.R. No. L-15184, May 31, 1963] Facts: Saura Import & Export Co Inc., mortgaged to the Phil. National Bank Davao, a parcel of land to secure the payment of promissory note of P27,000.00 with a condition from PNB that Saura shall insure the mortgaged property at all times against fire and earthquake for an amount satisfactory to PNB, indorsing to the latter policies of such insurance.

WHEREFORE, the instant petition is DENIED and the assailed decision of the respondent Court of Appeals is AFFIRMED in toto. x. Cancellation of Policy (Sections 64-65) Digested by: Vicente Ynclino Paulino vs Capital Ins. & Surety Co. [105 Phil. 1315, May 15, 1959] Facts: Plaintiff Paulino secured a fire insurance policy issued by defendant Capital Insurance on Feb. 8, 1952. On April 30, 1952 the Plaintiff wrote the defendant requesting cancellation of the policy, which the latter received on May 10, 1952.

EH 403 Page 38

⦿ 2010-2011 ⦿ 2011-2012 ⦿

INSURANCE LAW CASE DIGESTS
Pursuant to the requirement, Saura insured the building together with its contents with the Philippine International Surety, an insurance firm acceptable to PNB for P29,000.00, against fire for the period of 1 year from October 2, 1954 and as further required the insurance policy was endorsed to PNB. 13 days after the issuance of the fire insurance policy, insurer cancelled it, effective as of the date of issue. Notice of cancellation was given to PNB in writing, sent via Registered Mail, personally addressed to Fortunato Domingo, PNB Branch Manager. The latter received it on Nov. 8, 1954. On April 6, 1955, the building and its contents worth P40,685.69 were burned. Saura filed a claim and upon presentation of notice of loss with PNB, Saura learned for the 1st time that the policy had previously been cancelled. Insurer refused to pay Saura. Issue: Whether or not the notice of cancellation of the policy to the mortgagee (PNB) was already a substantial compliance of the insurer’s duty to notify the insured of the policy cancellation? Ruling: Fire insurance policies and other contracts of insurance upon property, in addition to the common provision for cancellation of the policy upon request of the insured, generally provide for cancellation by the insurer by notice to the insured for a prescribed period, which is usually 5 days, and the return of the unearned portion of the premium paid by the insured, such provision for cancellation upon notice being authorized by statutes in some jurisdiction, either specifically or as a provision of an adopted standard form of policy. The purpose of provisions or stipulations for notice to the insured, is to prevent the cancellation of the policy, without allowing the insured ample opportunity to negotiate for other insurance in its stead. The form and sufficiency of a notice of cancellation is determined by policy provisions. In order to form the basis for the cancellation of a policy, notice to the insured need not be in any particular form, in the absence of a statute or policy provision prescribing such form, and it is sufficient, so long as it positively and unequivocally indicates to the insured, that it is the intention of the company that the policy shall cease to be binding. Where the policy contains no provisions that a certain number of days notice shall be given, a reasonable notice and opportunity to obtain other insurance must be given. Actual personal notice to the insured is essential to a cancellation under a provision for cancellation by notice. The actual receipt by the insured of a notice of cancellation is universally recognized as a condition precedent to a cancellation of the policy by the insurer, and consequently a letter containing notice of cancellation which is mailed by the insurer but not received by the insured, is ineffective as cancellation. The notice should be personal to the insured and not to and/or through any unauthorized person by the policy. In the case at bar, the defendant insurance company, must have realized the paramount importance of sending a notice of cancellation, when it sent the notice of cancellation of the policy to the bank (as mortgagee), but not to the insured with which it (insurance company) had direct dealing. It was the primary duty of the insurance company to notify the insured, but it did not. Thus, liability attached principally to the insurance company, for its failure to give notice of the cancellation of the policy to Saura. Malayan Insurance vs. Cruz Facts: Coronacion Pinca has been sustained by the Insurance Commission in her claim for compensation for her burned house. The petitioner now disputes the decision on the ground that there was no valid insurance contract at the time of the loss because they have already cancelled the policy for non-payment of premium. Issue: WON Section 77 of the insurance Code is applicable in this case. Ruling: Section 77 is not applicable because payment of premium was in fact eventually made. There was a premium invoice which was issued to Pinca given by the agent of the petitioner who received the payment in behalf of Malayan Insurance. According to Section 65 of the Insurance Code a valid cancellation of policy requires the concurrence of the following conditions 1) Prior notice of cancellation to insured; 2) Notice must be based on the occurrence after effective date of policy of one of more of the grounds mentioned; 3) Must be in writing, mailed or delivered to the insured at the address shown on the policy; 4) Must state the grounds relied upon provided in Section 64 of the Insurance Code and upon request of the insured, to furnish facts on which cancellation is based. However, there was no proof of notice, assuming it complied with the other requisites mentioned above, was actually mailed to and received by Pinca. All Malayan Insurance offers to show that the cancellation was communicated to the insured is its employee’s testimony that the said cancellation was sent “by mail through our mailing section” without more.

EH 403 Page 39

⦿ 2010-2011 ⦿ 2011-2012 ⦿

INSURANCE LAW CASE DIGESTS
xi. Renewal of Policy (Section 66) G. Warranties (Sections 67-76) Definition, kinds and form of warranty (Section 67-69) ii. Express warranty (Sections 70-71) iii. Promissory warranty (Sections 72-73) iv. Violation of material warranty, effect (Section 74) i. Digested by Margaret Frances Aparte E. M. BACHRACH, vs. BRITISH AMERICAN ASSURANCE COMPANY, [G.R. No. L-5715 December 20, 1910] FACTS: Bachrach insured his building against fire with the British-American Assurance Company. After the effectivity of the policy, the insured stored gasoline, paints and varnishes within the premises insured. The building was burned and the insurer refused to pay the loss on the ground that the risk of fire was increased by the storage of gasoline, paints and varnishes. The insurer also claimed that the plaintiff transferred his interest in and to the property covered by the policy to H. W. Peabody & Co. to secure certain indebtedness due and owing to said company, and also that the plaintiff had transferred his interest in certain of the goods covered by the said policy to one Macke, to secure certain obligations assumed by the said Macke for and on behalf of the insured. Such execution of a chattel mortgage on the insured property without consent to the insurer violated what is known as the "alienation clause,". ISSUE: I. Whether the use of the building as a paint and varnish shop annulled the policy insurance. II. Whether the execution of the chattel mortgages without the knowledge and consent of the insurance company annulled the policy insurance. HELD: The court ruled in negative for both issues. I. The property insured consisted mainly of household furniture kept for the purpose of sale. The preservation of the furniture in a salable condition by retouching or otherwise was incidental to the business. The evidence offered by the plaintiff is to the effect that alcohol was used in preparing varnish for the purpose of retouching, though he also says that the alcohol was kept in store and not in the bodega where the furniture was. It is well settled that the keeping of inflammable oils on the premises, though prohibited by the policy, does not void it if such keeping is incidental to the business. Thus, where a furniture factory keeps benzine for the purposes of operation (Davis vs. Pioneer Furniture Company, 78 N. W. Rep., 596; Faust vs. American Fire Insurance Company, 91 Wis., 158), or where it is used for the cleaning machinery (Mears vs. Humboldt Insurance Company, 92 Pa. St., 15; 37 Am. Rep., 647), the insurer can not on that ground avoid payment of loss, though the keeping of the benzine on the premises is expressly prohibited It may be added that there was no provision in the policy prohibiting the keeping of paints and varnishes upon the premises where the insured property was stored. If the company intended to rely upon a condition of that character, it ought to have been plainly expressed in the policy. II. Upon reading the policy of insurance issued by the defendant to the plaintiff, it will be noted that there is no provision in said policy prohibiting the plaintiff from placing a mortgage upon the property insured, but, admitting that such a provision was intended, we think the lower court has completely answered this contention of the defendant. He said, in passing upon this question as it was presented: It is claimed that the execution of a chattel mortgage on the insured property violated what is known as the "alienation clause," which is now found in most policies, and which is expressed in the policies involved in cases 6496 and 6497 by a purchase imposing forfeiture if the interest in the property pass from the insured. (Cases 6496 and 6497, in which are involved other action against other insurance companies for the same loss as in the present action.) This clause has been the subject of a vast number of judicial decisions (13 Am. & Eng. Encyc. of Law, 2d ed., pp. 239 et seq.), and it is held by the great weight of authority that the interest in property insured does not pass by the mere execution of a chattel mortgage and that while a chattel mortgage is a conditional sale, there is no alienation within the meaning of the insurance law until the mortgage acquires a right to take possession by default under the terms of the mortgage. No such right is claimed to have accrued in the case at bar, and the alienation clause is therefore inapplicable Digested by: Kwin Asunto Young vs. Midland Textile insurance company [G.R. No. 9370. March 31, 1915.] Facts: The purpose of the present action is to recover the sum of P3,000 upon an insurance policy. The lower

EH 403 Page 40

⦿ 2010-2011 ⦿ 2011-2012 ⦿

INSURANCE LAW CASE DIGESTS
court rendered a judgment in favor of the plaintiff and against the defendant for the sum of P2,708.78, and costs. From that judgment the defendant appealed to this court. The undisputed facts upon which said action is based are as follows: The plaintiff occupied a building at '321 Calle Claveria, as a residence and bodega (storehouse). On the 29th of May, 1912, the defendant, in consideration of the payment of a premium of P60, entered into a contract of insurance with the plaintiff promising to pay to the plaintiff the sum of P3,000, in case said residence and bodega and contents should be destroyed by fire. One of the conditions of said contract was that no hazardous goods be stored or kept in the building. On the 4th or 5th of February, 1913, the plaintiff placed in said residence and bodega three boxes which belonged to him and which were filled with fireworks for the celebration of the Chinese new year. On the 18th day of March, 1913, said residence and bodega and the contents thereof were partially destroyed. Fireworks were found in a part of the building not destroyed by the fire; that they in no way contributed to the fire, or to the loss occasioned thereby. Issue: Whether or not the placing of said fireworks in the building insured, under the conditions above enumerated, they being "hazardous goods," is a violation of the terms of the contract of insurance. Held: Yes. The word "stored" has been defined to be a deposit in a store or warehouse for preservation or safe keeping; to put away for future use, especially for future consumption; to place in a warehouse or other place of deposit for safe keeping. Said definition does not include a deposit in a store, in small quantities, for daily use. "Daily use" precludes the idea of deposit for preservation or safe keeping, as well as a deposit for future consumption or safe keeping. A violation of the terms of a contract of insurance, by either party, will constitute the basis for a termination of the contractual relations, at the election of the other. The right to terminate the contractual relations exists even though the violation was not the direct cause of the loss. In the present case, the deposit of the "hazardous goods," in the building insured, was a violation of the terms of the contract. Although the hazardous goods did not contribute to the loss, the insurer, at his election, was relieved from liability Said deposit created a new risk, not included in the terms of the contract. The insurer had neither been paid, nor had he entered into a contract, to cover the increased risk. Contracts of insurance are contracts of indemnity, upon the terms and conditions specified therein. Parties have a right to impose such reasonable conditions at the time of the making of the contract as they deem wise and necessary. The rate of premium is measured by the character of the risk assumed. The insurer, for a comparatively small consideration, undertakes to guarantee the insured against loss or damage, upon the terms and conditions agreed upon, and upon no other. When the insurer is called upon to pay, in case of loss, he may justly insist upon a fulfillment of the terms of the contract. If the insured cannot bring himself within the terms and conditions of the contract, he is not entitled to recover for any loss suffered. The terms of the contract constitute the measure of the insurer's liability. If the contract has been terminated, by a violation of its terms on the part of the insured, there can be no recovery. Compliance with the terms of the contract is a condition precedent to the right of recovery. Courts cannot make contracts for the parties. While contracts of insurance are construed most favorably to the insured yet they must be construed according to the sense and meaning of the terms which the parties themselves have used. Astute and subtle distinctions should not be permitted, when the language of the contract is plain and unambiguous. Such distinctions tend to bring the law itself into disrepute. The judgment of the lower court is revoked and the defendant is relieved from any responsibility under said complaint, and, without any finding as to costs. Digested by: Rachel R. Aying UNION MANUFACTURING CO., INC. and the REPUBLIC BANK vs. PHILIPPINE GUARANTY CO., INC Facts: (1) Union Manufacturing Co., Inc. obtained certain loans, overdrafts and other credit accommodations from the Republic Bank for P415,000.00 with interest at 9% per annum from said date and to secure the payment, Union Manufacturing executed a real and chattel mortgages on certain properties. (2) As additional condition of the mortgage contract, Union Manufacturing undertook to secure insurance coverage over the mortgaged properties for the same amount of P415,000.00. (3) As Union Manufacturing failed to secure insurance coverage on the mortgaged properties, Republic Bank procured from the Philippine Guaranty Co., Inc. an insurance coverage on loss against fire for P500,000.00 over the properties of the Union Manufacturing with the annotation that loss or damage, if any, under is payable to Republic Bank as

EH 403 Page 41

⦿ 2010-2011 ⦿ 2011-2012 ⦿

INSURANCE LAW CASE DIGESTS
its interest may appear, subject however to the printed conditions of the Fire Insurance Policy Form; (4) Fire Insurance Policy No. 43170 was issued for the sum of P500,000.00 in favor of the assured, Union Manufacturing for which the corresponding premium of was paid by the Republic Bank to Philippine Guaranty (5) Upon the expiration of said fire policy, the same was renewed by the Republic Bank upon payment of the corresponding premium, (7) A fire occurred in the premises of the Union Manufacturing, (8) Union Manufacturing filed its fire claim with Philippine Guaranty but was denied on the following grounds: (a.)When Philippine Guaranty issued Fire Insurance Policy No. 43170 ... in the sum of P500,000.00 to cover the properties of the Union Manufacturing the same properties were already covered by Fire Policy of the Sincere Insurance Company and by insurance policies of the Oceanic Insurance Agency and (b) when Fire Insurance Policy No. 43170 was already in full force and effect, Union Manufacturing without the consent of Philippine Guaranty Co., Inc., obtained other insurance policies totalling P305,000.00 over the same properties prior to the fire. Issue: whether or not Republic Bank can recover its interest (as mortgagee) from the Fire Insurance Policy with Philippine Guaranty. Ruling: In as much as the Union Manufacturing has violated the condition of the policy to the effect that it did not reveal the existence of other insurance policies over the same properties, as required by the warranty appearing on the face of the policy and that said Union Manufacturing Co., Inc. represented that there were no other insurance policies at the time of the issuance of said defendant's policy, and it appearing furthermore that while the policy of the defendant was in full force and effect the Union Manufacturing Co., Inc. secured other fire insurance policies without the written consent of the defendant endorsed on the policy, the conclusion is inevitable that both the Republic Bank and Union Manufacturing Co., Inc. cannot recover from the same policy of the defendant because the same is null and void. It is to Santa Ana v. Commercial Union Assurance Co., that one turns to for the first explicit formulation as to the controlling principle. As was made clear in the opinion of this Court, penned by Justice Villa-Real: "Without deciding whether notice of other insurance upon the same property must be given in writing, or whether a verbal notice is sufficient to render an insurance valid which requires such notice, whether oral or written, we hold that in the absolute absence of such notice when it is one of the conditions specified in the fire insurance policy, the policy is null and void." The next year, in Ang Giok Chip v. Springfield Fire & Marine Ins. Co., the conformity of the insured to the terms of the policy, implied from the failure to express any disagreement with what is provided for, was stressed in these words of the ponente, Justice Malcolm: "It is admitted that the policy before us was accepted by the plaintiff. The receipt of this policy by the insured without objection binds both the acceptor and the insured to the terms thereof. The insured may not thereafter be heard to say that he did not read the policy or know its terms, since it is his duty to read his policy and it will be assumed that he did so." As far back as 1915, in Young v. Midland Textile Insurance Company, it was categorically set forth that as a condition precedent to the right of recovery, there must be compliance on the part of the insured with the terms of the policy. As stated in the opinion of the Court through Justice Johnson: "If the insured has violated or failed to perform the conditions of the contract, and such a violation or want of performance has not been waived by the insurer, then the insured cannot recover. Courts are not permitted to make contracts for the parties. The function and duty of the courts consist simply in enforcing and carrying out the contracts actually made. While it is true, as a general rule, that contracts of insurance are construed most favorably to the insured, yet contracts of insurance, like other contracts, are to be construed according to the sense and meaning of the terms which the parties themselves have used. If such terms are clear and unambiguous they must be taken and understood in their plain, ordinary and popular sense." More specifically, there was a reiteration of this Santa Ana ruling in a decision by the then Justice, later Chief Justice, Bengzon, in General Insurance & Surety Corp. v. Ng Hua. Thus: "The annotation then, must be deemed to be a warranty that the property was not insured by any other policy. Violation thereof entitles the insurer to rescind. (Sec. 69, Insurance Act) Such misrepresentation is fatal in the light of our views in Santa Ana v. Commercial Union Assurance Company, Ltd. ... . The materiality of non-disclosure of other insurance policies is not open to doubt." As a matter of fact, in a 1966 decision, Misamis Lumber Corp. v. Capital Ins. & Surety Co., Inc., Justice J.B.L. Reyes, for this Court, made manifest anew its adherence to such a principle in the face of an assertion that thereby a highly unfavorable provision for the insured would be accorded recognition. This is the language used: "The insurance contract may be rather onerous ('one sided', as the lower court put it), but that in itself does not justify the abrogation of its express terms, terms which the insured accepted or adhered to and which is the law between the contracting parties." H. Premium (Sections 77-82) Definition, when insured entitled to it (Section 77)

i.

Philippine Phoenix and Insurance Company vs. Woodworks Inc.

EH 403 Page 42

⦿ 2010-2011 ⦿ 2011-2012 ⦿

INSURANCE LAW CASE DIGESTS
[G.R. No. L-25317 August 6, 1979] Facts: Philippine Phoenix and Insurance Company ( Phil. Phoenix for short) issued a fire insurance policy in favor of Woodworks, Inc. upon application of the latter insuring Woodwork’s building and equipments against loss by fire for a one year term from from July 21, 1960 to July 21, 1961. After issuance of the policy, Woodworks did not pay the premium as stipulated. On April 19, 1961, Phil. Phoenix notified Woodworks of the cancellation of the policy at the same time claiming earned premiums still unpaid by Woodworks from July 21, 1960 to April 19, 1961 worth P7,483.11 for 271 days). In said letter, Phil. Phoenix credited the remaining balance ( or from April 19, 1961 to July 21, 1961 equivalent to 3,110.25 for 94 days) to Woodwork’s account. Woodworks naturally refused to pay Phil. Phoenix ‘s alleged “earned premiums” averring that failure to pay the premium after the issuance of the policy rendered the insurance policy unenforceable thereby prompting Phil. Phoenix legal action for recovery of premiums before the lower court ( CFI) of which Phoenix won. Now on appeal before the SC on pure questions of law. Issue: 1. WON non-payment or failure as the case may be of premiums after issuance of the fire insurance policy rendered such policy invalid or of no effect 2. Even if the premium is unpaid after issuance of the policy by Phil. Phoenix, granting aguendo that indeed Phil. Phoenix gave Woodworks credit extensions for purposes of giving effect of the policy, was there acceptance of such credit extended in this case? Ruling: 1. Insurance policy became ineffective by nonpayment of premiums after issuance of policy. Section 77 of the Insurance Code (Presidential Decree No. 612, promulgated on December 18, 1974), provides that no contract of insurance issued by an insurance company is valid and binding unless and until the premium thereof has been paid, notwithstanding any agreement to the contrary. The insurance policy provides: THE COMPANY HEREBY AGREES with the Insured ... that if the Property above described, or any part thereof, shall be destroyed or damaged by Fire or Lightning after payment of Premium, at any time between 4:00 o'clock in the afternoon of the TWENTY FIRST day of JULY One Thousand Nine Hundred and SIXTY and 4:00 o'clock in the afternoon of the TWENTY FIRST day of JULY One Thousand Nine Hundred and SIXTY ONE. ... (Emphasis supplied) Insurance is "a contract whereby one undertakes for a consideration to indemnify another against loss, damage or liability arising from an unknown or contingent event." The consideration is the "premium". "The premium must be paid at the time and in the way and manner specified in the policy and, if not so paid, the policy will lapse and be forfeited by its own terms." Since the premium had not been paid, the policy must be deemed to have lapsed. The non-payment of premiums does not merely suspend but put, an end to an insurance contract, since the time of the payment is peculiarly of the essence of the contract. ... the rule is that under policy provisions that upon the failure to make a payment of a premium or assessment at the time provided for, the policy shall become void or forfeited, or the obligation of the insurer shall cease, or words to like effect, because the contract so prescribes and because such a stipulation is a material and essential part of the contract. This is true, for instance, in the case of life, health and accident, fire and hail insurance policies. In fact, if the peril insured against had occurred, plaintiff, as insurer, would have had a valid defense against recovery under the Policy it had issued. Explicit in the Policy itself is plaintiff's agreement to indemnify defendant for loss by fire only "after payment of premium," supra. Compliance by the insured with the terms of the contract is a condition precedent to the right of recovery. The burden is on an insured to keep a policy in force by the payment of premiums, rather than on the insurer to exert every effort to prevent the insured from allowing a policy to elapse through a failure to make premium payments. The continuance of the insurer's obligation is conditional upon the payment of premiums, so that no recovery can be had upon a lapsed policy, the contractual relation between the parties having ceased. Moreover, "an insurer cannot treat a contract as valid for the purpose of collecting premiums and invalid for the purpose of indemnity." 2. No express acceptance of credit extensions, if any, by Woodworks. The Policy provides for pre-payment of premium. Accordingly; "when the policy is tendered the insured must pay the premium unless credit is given or there is a waiver, or some agreement obviating the necessity for prepayment." To constitute an extension of credit there must be a clear and express agreement therefor." An acceptance of an offer to allow credit, if one was made, is as essential to make a valid agreement for credit, to change a conditional delivery of an insurance policy to an unconditional delivery, as it is to make any other contract. Such an acceptance could not be merely a mental act or state of mind, but would require a promise to pay made known in some manner to defendant.

EH 403 Page 43

⦿ 2010-2011 ⦿ 2011-2012 ⦿

INSURANCE LAW CASE DIGESTS
Digested by: Emmanuel Servicewide Specialists Inc. vs. CA [1996 May 8] Facts: Respondent spouses Ricardo and Elisa Trinidad purchased a 1983 Isuzu Gemini car from Autoworld Sales Corporation. The price was P98,156.00 payable in 24 equal monthly installments of P4,089.00 each month. To secure payment thereof, the Trinidads executed on the same date a promissory note and a deed of chattel mortgage on the subject car in favor of Autoworld. Autoworld assigned its interests on the promissory note and chattel mortgage to Filinvest. These assignments were made with due notice to private respondents who delivered 17 checks to Filinvest in full payment of the car. On November 8, 1985, Filinvest assigned all its rights and interests on the promissory note and chattel mortgage in favor of Servicewide. On November 18, 1985, Ricardo Trinidad received a demand letter from Servicewide stating that an assignment of credit had been made by Filinvest in its favor and that the Trinidads had not paid two successive installments on the car which had matured on July 15 and August 15, 1985. No mention was made in the letter that Filinvest had paid insurance premiums to Perla Compania de Seguros to insure the car against loss and damage corresponding to two years. Trinidads were also never informed by Filinvest that their installment payments on the car were converted to premium payments on the insurance. After informing private respondents that they failed to pay the last two consecutive monthly installments, petitioner demanded that either they pay the whole remaining balance of P6,977.67, including accrued interest, or return possession of the car to petitioner. When the Trinidads refused to pay the amount demanded or to return the car, petitioner filed an action for replevin and damages. Issue: WON petitioner should have applied the installment payments made by private respondents for the payment of the car to the payment of the insurance premiums without prior notice to private respondents. Ruling: The petition is unmeritorious. The provision in the Chattel Mortgage subject of the controversy states: "The said MORTGAGOR covenants and agrees that he will cause the property/ies herein above mortgaged to be insured against loss or damage by accident, theft and fire for a period of one year from date thereof and every year thereafter until the mortgage obligation is fully paid with an insurance company, or companies acceptable to the MORTGAGEE in an amount not less than the outstanding balance of the mortgage obligation; that he will make all loss, if any, under such policy or policies, payable to the MORTGAGEE or its assigns as its interest may appear and forewith deliver such policy or policies to the MORTGAGEE, the said MORTGAGOR further covenants and agrees that in default of his affecting or renewing such insurance and delivering the policies so endorsed to the MORTGAGEE within five (5) days after the execution of this mortgage or the expiry date of the insurance the MORTGAGEE, may, at his option, but without any obligation to do so effect such insurance or obtain such renewal for the account of the MORTGAGOR and that any money so disbursed by the MORTGAGEE shall be added to the principal indebtedness hereby secured and shall become due and payable at the time for the payment of the immediately coming or following installment to be due under the note aforesaid after the date of such insurance renewal and shall bear interest at the same rate as the principal indebtedness.” Petitioner contends that the matter about the notice is deemed waived by private respondents because the car should be fully covered at all times. Petitioner claims that if, as stated in the Chattel Mortgage, private respondents failed to renew the insurance, petitioner is entitled to renew the same for the account of private respondents without any notice to them. The petition is unmeritorious. While it is true that the Chattel Mortgage does not say that notice to the mortgagor of the renewal of the insurance premium by the mortgagee is necessary, at the same time, there is no provision that authorizes petitioner to apply the payments made to it for the payment of the chattel to the payment of the said premiums. From the records of the case, it is clear that private respondents had fully paid for the car. This fact was never rebutted by petitioner; it was the insurance premiums pertaining to the two-year period from July 29, 1984 to July 29, 1986 that petitioner claims were not paid. Both the Regional Trial Court and the Court of Appeals found that before the mortgagee (petitioner) may effect the renewal of insurance, two conditions must be met: (1) Default by the mortgagor (private respondents) in effecting renewal of the insurance and (2) failure to deliver the policy with endorsement to petitioner. The Court notes an additional element of the provisions regarding the renewal of the insurance; specifically, that petitioner was under no obligation to effect the same. In other words, petitioner as mortgagee was not duty-bound to renew the insurance

EH 403 Page 44

⦿ 2010-2011 ⦿ 2011-2012 ⦿

INSURANCE LAW CASE DIGESTS
in the event that private respondents failed to do so; it was merely optional on its part. The question now arises whether private respondents were in default for failing to have the car covered by insurance for the period in question. Private respondents claim that the car was duly covered and the Court finds no evidence on record showing this assertion to be false. Petitioner has averred, however, that the insurance taken by private respondents was only for third-party liability and not the comprehensive insurance required. If petitioner was aware that the insurance coverage was inadequate, why did it not inform private respondent about it? After all, since petitioner was under no obligation to effect renewal thereof, it is but logical that it should relay to private respondents any defect of the insurance coverage before itself assuming the same. Furthermore, even if the car were not covered with the proper insurance, there is nothing in the provisions of the Chattel Mortgage that authorizes petitioner to apply previous payments for the car to the insurance. What is stated is: "x x x that any money so disbursed by the mortgagee shall be added to the principal indebtedness hereby secured x x x" talics supplied). Clear is it that petitioner is not obligated to convert any of the installments made by private respondents for the car to the payment for the renewal of the insurance. Should it decide to do so, it has to send notice to private respondents who had already paid in full the principal indebtedness in question. When petitioner wrote private respondents the November 8, 1985 demand letter regarding nonpayment of the installments, no mention was made of unpaid insurance premiums. Thus, private respondents were quite justified in ignoring the same since, to the best of their knowledge, they had already paid for the car in full. ii. Effects of non-payment/partial payment Digested by: Birondo ACME Shoe Rubber & Plastic Corp. vs. CA Facts: Petitioner ACME had been insuring its properties yearly against fire with Domestic Insurance Company (INSURER). It continued the insurance for the period of May 15, 1962 up to May 15, 1963 for the amount of P200,000. On January 8, 1964, ACME only paid P3,331.26, which the INSURER applied as renewal premium for the period May 15, 1963 to May 15, 1964 and the issued renewal receipt having a stamp thereon that it is subject to Receipt of Payment Clause and Credit Agreement – both attached as riders. The former clause declared that “the insurance will be deemed valid and binding upon the company only when the premium and documentary stamps have actually been paid”; the latter provided that “the premium corresponding to the first 90 days of the policy’s term or any renewal is considered paid for the purpose of only making said policy valid and binding during said portion of the term and it shall automatically become void and ineffective (without prejudice to the obligation of the insured to pay the corresponding short premium for the said 90 days) unless prior to the expiration of said period the INSURED shall have actually paid to the Company the total premium and the documentary stamps. Then, ACME, through its President, signed a promissory note on May 26, 1964 promising to pay the INSURER within 90 days from the effective date of the Policy (May 15, 1964) the premium and documentary stamps (P3,331.26) and failure to pay when due, it is agreed that said policy should stand automatically cancelled. A fire completely destroyed ACME’s properties on October13, 1964 prompting it to file a claim against the INSURER which however disclaimed liability on the ground that as of the date of loss, the properties burned were no covered by insurance. ACME has obtained a favorable decision in the trial court when it sued for collection of insurance proceeds and damages however the same was reversed by the CA – affirming the INSURER’s contention and thus it could not be held liable for any indemnity. Issue: Whether or not there was an insurance contract at the moment of the loss. Ruling: CA is sustained. ACME, through its President, was fully aware that the policy would automatically be cancelled if it will not pay the premium before August 13, 1964 (90th day from May 14, 1964). In fact, reminders for payment being sent were unheeded. Not having paid the 1964-1965 premium within the extension granted, and pursuant to RA No. 3450, the policy was automatically cancelled and there was no insurance coverage to speak of as of the date of the fire. It was also ACME’s contention that the INSURER – which accepted the one-year premium on January 8, 1964, has no right to apply it to the payment of a period of coverage prior thereto for the policy was void under RA 3450 (Sec. 72. xxx. No policy issued by an insurance company is valid and binding unless and until the premium thereof has been paid). However, since said Act was approved only on June 20, 1963 and was put into effect only on October 1, 1963, it could not retroactively affect the renewal of the insurance policy on May 15, 1963. So, ACME’s premium payment of January 8, 1964 was properly applied to the 1963-1964 premium. Lastly, ACME’s claim that the INSURER would unjustly enrich itself if it were to be allowed to apply the oneyear premium it received to a past period when the policy was void and the INSURER had incurred no risk

EH 403 Page 45

⦿ 2010-2011 ⦿ 2011-2012 ⦿

INSURANCE LAW CASE DIGESTS
is flawed for the reason already stated that the renewal receipts were issued before RA 3450 was approved and implemented. It is axiomatic that laws have no retroactive effect unless the contrary is provided. What became automatically cancelled by RA 3450 was the 1964-1965 policy for ACME’s failure to pay the premium within the 90-day extension granted and in accordance with the express terms of the promissory note it had signed. Judgment affirmed. Digested by: CANTILLAS, IRISH ARCE vs. THE CAPITAL INSURANCE & SURETY CO., INC., FACTS: In Civil Case No. 66466 of the Court of First Instance of Manila, the Capital Insurance and Surety Co., Inc., (COMPANY) was ordered to pay Pedro Arce (INSURED) the proceeds of a fire insurance policy. Not satisfied with the decision, the company appealed to this Court on questions of law. The INSURED(Pedro ARce) was the owner of a residential house in Tondo, Manila, which had been insured with the COMPANY(Capital insurance) since 1961 under Fire Policy No. 24204. On November 27, 1965, the COMPANY sent to the INSURED Renewal Certificate No. 47302 to cover the period December 5, 1965 to December 5, 1966. The COMPANY also requested payment of the corresponding premium in the amount of P 38.10. Anticipating that the premium could not be paid on time, the INSURED, thru his wife, promised to pay it on January 4, 1966. The COMPANY accepted the promise but the premium was not paid on January 4, 1966. On January 8, 1966, the house of the INSURED was totally destroyed by fire. On January 10, 1966, INSURED's wife presented a claim for indemnity to the COMPANY. She was told that no indemnity was due because the premium on the policy was not paid. Nonetheless the COMPANY tendered a check for P300.00 as financial aid which was received by the INSURED's daughter, Evelina R. Arce. The COMPANY reiterated that the check was given "not as an obligation, but as a concession" because the renewal premium had not been paid, The INSURED cashed the check but then sued the COMPANY on the policy. ISSUE: Whether the petitioners are entitled to claim from their policy despite non-payment of their premium. RULING: SEC. 72. An insurer is entitled to payment of premium as soon as the thing insured is exposed to the perils insured against, unless there is clear agreement to grant credit extension for the premium due. No policy issued by an insurance company is valid and binding unless and until the premium thereof has been paid " (Italics supplied.) (p. 11, Appellant's Brief.) Morever, the parties in this case had stipulated: IT IS HEREBY DECLARED AND AGREED that not. withstanding anything to the contrary contained in the within policy, this insurance will be deemed valid and binding upon the Company only when the premium and documentary stamps therefor have actually been paid in full and duly acknowledged in an official receipt signed by an authorized official/representative of the Company, " (pp. 45-46, Record on Appeal.) It is obvious from both the Insurance Act, as amended, and the stipulation of the parties that time is of the essence in respect of the payment of the insurance premium so that if it is not paid the contract does not take effect unless there is still another stipulation to the contrary. In the instant case, the INSURED was given a grace period to pay the premium but the period having expired with no payment made, he cannot insist that the COMPANY is nonetheless obligated to him. THE CAPITAL INSURANCE & SURETY CO., INC vs. PLASTIC ERA CO., INC., AND COURT OF APPEALS Digested by: marccarillo Facts: On Dec. 7, 1960, Capital Insurance & Surety Co., Inc (Capital Insurance) delivered to Plastic Era Manufacturing Co., Inc (Plastic Era) its Open Fire Policy No.22760 wherein the former undertook to insure the latter’s building, equipments, raw materials, products and accessories located at Sheridan Street, Mandaluyong, Rizal. The policy expressly provides that if the property insured would be destroyed or damaged by fire after the payment of the premiums, at anytime between the Dec. 15 1960 and one o'clock in the afternoon of the Dec. 15, 1961, the insurance company shall make good all such loss or damage in an amount not exceeding P100k. Plastic Era failed to pay its premium and instead executed an acknowledgment receipt promising to pay 30 days after date. On Jan.8,1961, Plastic Era delivered a check as partial payment of the insurance premium worth 1k. However, it was dishonored by the Bank of America for lack of funds on Feb.20, 1961. Records revealed that Plastic Era had a balance of P1,193.41 as early as Jan.19, 1961. (Note: premium due date was on Jan.16,1961) Not unexpectedly, the property insured by Plastic Era was destroyed by fire on Jan.18, 1961 (that’s 2 days after the premium became due). Plastic Era filed a claim for indemnity. Capital Insurance wanted to become part of Philippine Jurisprudence so it denied

EH 403 Page 46

⦿ 2010-2011 ⦿ 2011-2012 ⦿

INSURANCE LAW CASE DIGESTS
the claim on the ground that the premium was not paid. Pissed, Plastic Era then filed a case in the lower court which renderred judgment in its favor. The appelate court affirmed it and so the issue is brought before the SC. Issue: W/N a contract of insurance has been duly perfected between the petitioner, Capital Insurance, and respondent Plastic Era. Ruling: Yup, it has been perfected. The Insurnace Policy states that: “THE COMPANY HEREBY AGREES with the Insured but subject to the terms and conditions endorsed or otherwise expressed hereon, which are to be taken as part of this Policy), that if the Property described, or any part thereof, shall be destroyed or damaged by Fire or Lightning after payment of the Premiums, at anytime between...xxx” In clear and unequivocal terms the insurance policy provides that it is only upon payment of the premiums by Plastic Era that Capital Insurance agrees to insure the properties of the former against loss or damage in an amount not exceeding P100,000.00. It appears on record that on the day the insurance policy was delivered, Plastic Era did not pay the Capital Insurance, but instead executed an acknowledgment receipt. Q:Could not this have been considered a valid payment of the insurance premium? Article 1249 NCC: The delivery of promissory notes payable to order, or bills of exchange or other mercantile documents shall produce the effect of payment only when they have been cashed, or when through the fault of the creditor they have been impaired. Under this provision the mere delivery of a bill of exchange in payment of a debt does not immediately effect payment. It simply suspends the action arising from the original obligation in satisfaction of which it was delivered, until payment is accomplished either actually or presumptively. But wait... Acceptance by Capital Insurance considered as waiver Significantly, in the case before Us the Capital Insurance accepted the promise of Plastic Era to pay the insurance premium within thirty (30) days from the effective date of policy. By so doing, it has implicitly agreed to modify the tenor of the insurance policy and in effect, waived the provision therein that it would only pay for the loss or damage in case the same occurs after the payment of the premium. Considering that the insurance policy is silent as to the mode of payment, Capital Insurance is deemed to have accepted the promissory note in payment of the premium. Citing a US Case, our Supreme Court sad that “that although one of conditions of an insurance policy is that "it shall not be valid or binding until the first premium is paid", if it is silent as to the mode of payment, promissory notes received by the company must be deemed to have been accepted in payment of the premium. In other words, a requirement for the payment of the first or initial premium in advance or actual cash may be waived by acceptance of a promissory note ...” The fact that the check issued by Plastic Era in partial payment of the promissory note was later on dishonored did not in any way operate as a forfeiture of its rights under the policy, there being no express stipulation therein to that effect. This is how the story ends. :D Digested by: Brian O. Dalanon Velasco vs. Apostol Facts: That on November 27, 1973, at about 2:30 p.m. plaintiffs were riding in their Mercury car driven by their driver Restitute Guarra, along Quezon Boulevard when an N/S taxicab driven by defendant Dominador Santos registered in the name of defendants Alice Artuz, c/o Norberto Santos, crossed the center island towards their direction, and finally collided with their car at the left front part. The said taxicab tried to return to its original lane, but was unable to climb the island, and instead, backtracked, hitting again plaintiffs' car in the left near portion, causing the latter's back portion to turn toward the center hitting a jeepney on its right. Originally sued as defendants were Dominador Santos, Alice Artuz, and Norberto Santos. After an answer was filed by said defendants, private respondent Maharlika Insurance Co., Inc. was impleaded as a defendant in an amended complaint filed by the petitioner with an allegation that the N/S taxicab was insured against third party liability for P20,000 with private respondent at the time of the accident. In its answer to the amended complaint, respondent Maharlika Insurance Co., Inc. claimed that there was no cause of action against it because at the time of the accident, the alleged insurance policy was not in force due to non-payment of the premium. It further averred that even if the taxicab had been insured, the complaint would still be premature since the policy provides that the insurer would be liable only when the insured becomes legally liable.

EH 403 Page 47

⦿ 2010-2011 ⦿ 2011-2012 ⦿

INSURANCE LAW CASE DIGESTS
The trial court rendered judgment in favor of the plaintiff finding that the evidence on the negligence of defendant Dominador Santos was uncontroverted and the proximate cause of the accident was his negligence. 5 Defendants Dominador Santos, Alice Artuz, and Norberto Santos were adjudged jointly and severally liable to petitioners. However, Maharlika Insurance Co. was exonerated on the ground that the policy was not in force for failure of the therein defendants to pay the initial premium and for their concealment of a material fact. Petitioners fault the respondent-judge for considering private respondent's defense of late payment of premium when, according to them, "the same was waived at the pre-trial. Petiotioners theorize that what was stipulated in the pre-trail order “does not include the issue on whether defendant Maharlika Insurance Co., Inc. is liable under the insurance policy, even as the premium was paid after the accident in question.” The accident for which respondent insurance company is sought to be held liable occurred on November 27, 1973 while the initial premium was paid only on December 11, 1973. Petitioners maintain that in spite of this late payment, the policy is nevertheless binding because there was an implied agreement to grant a credit extension so as to make the policy effective. To them, the subsequent acceptance of the premium and delivery of the policy estops the respondent company from asserting that the policy is ineffective. Issue: Whether defendant Maharlika Insurance Co. Inc. is liable under the insurance policy on account of the negligence of defendant Dominador Santos. Ruling: Petitioners' position is bereft of merit Although there is no express statement as to the fact of late payment, this is necessarily deemed included in or ineluctably inferred from the issue of whether the company is liable under the insurance policy it had allegedly issued for the vehicle involved and on which petitioners seek to recover. Issues that are impliedly included therein or may be inferable therefrom by necessary implication are as much integral parts of the pre-trial order as those that are expressly stipulated. In fact, it would be absurd and inexplicable for the respondent company to knowingly disregard or deliberately abandon the issue of non-payment of the premium on the policy considering that it is the very core of its defense. The delivery of the policy made on March 28, 1974 and only because the premium was had been paid, in fact, more than three months before such delivery. The said payment was accepted by the insurer without any knowledge that the risk insured against had already occurred since such fact was concealed by the insured and was not revealed to the insurer. The fact withheld could not in any event have influenced the respondent company in entering into the supposed contract or in estimating the character of the risk or in fixing the rate premium, for the simple reason that no such contract existed between the defendants and the company at the time of the accident. What should be apparent from such actuations of therein defendants, however, is the presence of bad faith on their part, a reprehensible disregard of the principle that insurance contracts are uberrimae fidae and demand the most abundant good faith. Digested by: Grace Jayne Dingal VALENZUELA v. COURT OF APPEALS, ARAGON et al. Facts: Arturo Valenzuela is a General Agent of Philippine American General Insurance (Philamgen) since 1965. He was authorized to solicit and sell in behalf of Philamgen all kinds of non-life insurance, and in consideration of services rendered was entitled to receive the full agent's commission of 32.5% from Philamgen under the scheduled commission rates. From 1973 to 1975, Valenzuela solicited marine insurance from one of his clients, the Delta Motors in the amount of P4.4 Million from which he was entitled to a commission of 32%. However, Valenzuela did not receive his full commission which amounted to P1.6 Million from the P4.4 Million insurance coverage of the Delta Motors. In 1977, Philamgen started to become interested in and expressed its intent to share in the commission due Valenzuela on a fifty-fifty basis. Because of the refusal of Valenzuela, Philamgen terminated the General Agency Agreement of Valenzuela. Issue: Whether or not Philamgen could continue to hold Valenzuela jointly and severally liable with the insured for unpaid premiums Held: NO. The principal cause of the termination of Valenzuela as General Agent of Philamgen arose from his refusal to share his Delta commission. The apparent bad faith of the private respondents in terminating the General Agency Agreement of petitioners. The agency involving petitioner and private respondent is one "coupled with

EH 403 Page 48

⦿ 2010-2011 ⦿ 2011-2012 ⦿

INSURANCE LAW CASE DIGESTS
an interest," and, therefore, should not be freely revocable at the unilateral will of the latter. With the termination of the General Agency Agreement, Valenzuela would no longer be entitled to commission on the renewal of insurance policies of clients sourced from his agency. Despite the termination of the agency, Philamgen continued to hold Valenzuela jointly and severally liable with the insured for unpaid premiums. Valenzuela had an interest in the continuation of the agency when it was unceremoniously terminated not only because of the commissions he should continue to receive from the insurance business he has solicited and procured but also for the fact that by the very acts of the respondents, he was made liable to Philamgen in the event the insured fail to pay the premiums due. They are estopped by their own positive averments and claims for damages. Therefore, the respondents cannot state that the agency relationship between Valenzuela and Philamgen is not coupled with interest. There is an exception to the principle that an agency is revocable at will and that is when the agency has been given not only for the interest of the principal but for the interest of third persons or for the mutual interest of the principal and the agent. In these cases, it is evident that the agency ceases to be freely revocable by the sole will of the principal. The factor rendering Philamgen and the private respondents liable in damages is that the termination by them of the General Agency Agreement was tainted with bad faith. If a principal acts in bad faith and with abuse of right in terminating the agency, then he is liable in damages. Valenzuela is not liable to Philamgen for the unpaid and uncollected premiums. Under Section 77 of the Insurance Code, the remedy for the non-payment of premiums is to put an end to and render the insurance policy not binding — Sec. 77 ... [N]otwithstanding any agreement to the contrary, no policy or contract of insurance is valid and binding unless and until the premiums thereof have been paid except in the case of a life or industrial life policy whenever the grace period provision applies… In Philippine Phoenix Surety v. Woodworks, we held that the non-payment of premium does not merely suspend but puts an end to an insurance contract since the time of the payment is peculiarly of the essence of the contract. And in Arce v. The Capital Insurance and Surety Co. Inc. (117 SCRA 63, [1982]), we reiterated the rule that unless premium is paid, an insurance contract does not take effect. Thus: It is to be noted that Delgado (Capital Insurance & Surety Co., Inc. v. Delgado, 9 SCRA 177 [1963] was decided in the light of the Insurance Act before Sec. 72 was amended by the underscored portion. Supra. Prior to the Amendment, an insurance contract was effective even if the premium had not been paid so that an insurer was obligated to pay indemnity in case of loss and correlatively he had also the right to sue for payment of the premium. But the amendment to Sec. 72 has radically changed the legal regime in that unless the premium is paid there is no insurance.” In Philippine Phoenix Surety case, we held: Moreover, an insurer cannot treat a contract as valid for the purpose of collecting premiums and invalid for the purpose of indemnity. No contract of Insurance by an insurance company is valid and binding unless and until the premium thereof has been paid, notwithstanding any agreement to the contrary Since admittedly the premiums have not been paid, the policies issued have lapsed. The insurance coverage did not go into effect or did not continue and the obligation of Philamgen as insurer ceased. Hence, for Philamgen which had no more liability under the lapsed and inexistent policies to demand, much less sue Valenzuela for the unpaid premiums would be the height of injustice and unfair dealing. In this instance, with the lapsing of the policies through the nonpayment of premiums by the insured there were no more insurance contracts to speak of. As this Court held in the Philippine Phoenix Surety case, supra "the non-payment of premiums does not merely suspend but puts an end to an insurance contract since the time of the payment is peculiarly of the essence of the contract." The circumstances of the case, however, require that the contractual relationship between the parties shall be terminated upon the satisfaction of the judgment. No more claims arising from or as a result of the agency shall be entertained by the courts after that date. ACCORDINGLY, the petition is GRANTED.

UCPB General Insurance vs. Masagana Telamart Inc. , [GR 137172] Facts: On 15 April 1991, UCPB General Insurance Co. Inc. (UCPBGen) issued 5 insurance policies covering Masagana Telamart, Inc.'s various property described therein against fire, for the period from 22 May 1991 to 22 May 1992. In March 1992, UCPBGen evaluated the policies and decided not to renew them upon expiration of their terms on 22 May 1992. UCPBGen advised Masagana's broker, Zuellig Insurance Brokers, Inc. of its intention not to renew the policies. On 6 April 1992, UCPBGen gave written notice to Masagana of the non-renewal of the policies at the address stated in the policies. On 13 June 1992, fire razed Masagana's property covered by three of the insurance policies UCPBGen issued. On 13 July 1992, Masagana presented to UCPBGen's cashier at its head office 5 manager's checks in the total amount of P225,753.95, representing premium for the renewal of the policies from 22 May 1992 to 22 May 1993. No notice of loss

EH 403 Page 49

⦿ 2010-2011 ⦿ 2011-2012 ⦿

INSURANCE LAW CASE DIGESTS
was filed by Masagana under the policies prior to 14 July 1992. On 14 July 1992, Masagana filed with UCPBGen its formal claim for indemnification of the insured property razed by fire. On the same day, 14 July 1992, UCPBGen returned to Masagana the 5 manager's checks that it tendered, and at the same time rejected Masagana's claim for the reasons (a) that the policies had expired and were not renewed, and (b) that the fire occurred on 13 June 1992, before Masagana's tender of premium payment. On 21 July 1992, Masagana filed with the Regional Trial Court, Branch 58, Makati City, a civil complaint against UCPBGen for recovery of P18,645,000.00, representing the face value of the policies covering Masagana's insured property razed by fire, and for attorney's fees. On 23 October 1992, after its motion to dismiss had been denied, UCPBGen filed an answer to the complaint. It alleged that the complaint "fails to state a cause of action"; that UCPBGen was not liable to Masagana for insurance proceeds under the policies because at the time of the loss of Masagana's property due to fire, the policies had long expired and were not renewed. After due trial, on 10 March 1993, the Regional Trial Court, Branch 58, Makati, rendered decision, (1) authorizing and allowing Masagana to consign/deposit with this Court the sum of P225,753.95 (refused by UCPBGen) as full payment of the corresponding premiums for the replacementrenewal policies; (2) declaring Masagana to have fully complied with its obligation to pay the premium thereby rendering the replacement-renewal policy effective and binding for the duration 22 May 1992 until 22 May 1993; and, ordering UCPBGen to deliver forthwith to Masagana the said replacement-renewal policies; (3) declaring two of the policies in force from 22 August 1991 up to 23 August 1992 and 9 August 1991 to 9 August 1992, respectively; and (4) ordering UCPBGen to pay Masagana the sums of: (a) P18,645,000.00 representing the latter's claim for indemnity under three policies and/or its replacementrenewal policies; (b) 25% of the total amount due as and for attorney's fees; (c) P25,000.00 as necessary litigation expenses; and, (d) the costs of suit. In due time, UCPBGen appealed to the Court of Appeals. On 7 September 1998, the Court of Appeals promulgated its decision affirming that of the Regional Trial Court with the modification that item 3 of the dispositive portion was deleted, and the award of attorney's fees was reduced to 10% of the total amount due. The Court of Appeals held that following previous practise, Masagana was allowed a 60 to 90 day credit term for the renewal of its policies, and that the acceptance of the late premium payment suggested an understanding that payment could be made later. UCPBGen appealed. Issue: Whether the fire insurance policies issued by UCPBGen to the Masagana covering the period 22 May 1991 to 22 May 1992, had expired on the latter date or had been extended or renewed by an implied credit arrangement though actual payment of premium was tendered on a latter date after the occurrence of the risk (fire) insured against. Held: The answer is easily found in the Insurance Code. No, an insurance policy, other than life, issued originally or on renewal, is not valid and binding until actual payment of the premium. Any agreement to the contrary is void. The parties may not agree expressly or impliedly on the extension of credit or time to pay the premium and consider the policy binding before actual payment. The case of Malayan Insurance Co., Inc. vs. Cruz-Arnaldo is not applicable. In that case, payment of the premium was in fact actually made on 24 December 1981, and the fire occurred on 18 January 1982. Here, the payment of the premium for renewal of the policies was tendered on 13 July 1992, a month after the fire occurred on 13 June 1992. The assured did not even give the insurer a notice of loss within a reasonable time after occurrence of the fire. Hence, the Supreme Court reversed and set aside the decision of the Court of Appeals in CA-GR CV 42321. In lieu thereof, the Court rendered judgment dismissing Masagana's complaint and UCPBGen's counterclaims thereto filed with the Regional Trial Court, Branch 58, Makati City, in Civil Case 92-2023, without costs. PHILIPPINE PHOENIX SURETY & INSURANCE, INC. vs.WOODWORKS, INC. [G.R. No. L-22684 August 31, 1967] FACTS: That on April 1, 1960, plaintiff issued to defendant Fire Policy No. 9652 for the amount of P300,000.00. The premiums of said policy amounted to P6, 051.95. The defendant paid P3,000.00 on September 22, 1960 and the plaintiff made several demands on defendant to pay the amount of P3,522.09. Appellee Philippine Phoenix Surety & Insurance Co., Inc. commenced this action in the Municipal Court of Manila to recover from appellant Woodworks, Inc. the sum of P3,522.09, representing the unpaid balance of the premiums on a fire insurance policy issued by appellee in favor of appellant for a term of one year from April 1, 1960 to April 1, 1961. From an adverse decision of said court, Woodworks, Inc. appealed to the Court of First Instance of Manila. Appeal upon a question of law taken by Woodworks, Inc. from the judgment of the Court of First Instance of Manila "ordering the defendant, Woodworks, Inc. to pay to the plaintiff, Philippine Phoenix Surety & Insurance, Inc., the sum of P3,522.09 with interest thereon at the legal rate of 6% per annum from the date of the filing of the complaint until fully paid, and costs of the suit." Hence, this petition. Issue:

EH 403 Page 50

⦿ 2010-2011 ⦿ 2011-2012 ⦿

INSURANCE LAW CASE DIGESTS
Whether or not the non-payment of premium does not cancel the policy in a perfected contract of insurance Whether or not a partial payment of the premium made the policy effective during the whole period of the policy Ruling: The petition is lack of merit. There is, consequently, no doubt at all that, as between the insurer and the insured, there was not only a perfected contract of insurance but a partially performed one as far as the payment of the agreed premium was concerned. Thereafter the obligation of the insurer to pay the insured the amount for which the policy was issued in case the conditions therefor had been complied with, arose and became binding upon it, while the obligation of the insured to pay the remainder of the total amount of the premium due became demandable. The court did not agree with appellant's theory that non-payment by it of the premium due, produced the cancellation of the contract of insurance. Such theory would place exclusively in the hands of one of the contracting parties the right to decide whether the contract should stand or not. Rather the correct view would seem to be this: as the contract had become perfected, the parties could demand from each other the performance of whatever obligations they had assumed. In the case of the insurer, it is obvious that it had the right to demand from the insured the completion of the payment of the premium due or sue for the rescission of the contract. As it chose to demand specific performance of the insured's obligation to pay the balance of the premium, the latter's duty to pay is indeed indubitable. The appealed decision being in accordance with law and the evidence, the same is affirmed. Digested by: Gestopa, Gevina Makati Tuscany Condominium Corp. vs. Court of Appeals Facts: Sometime in early 1982, private respondent American Home Assurance Co. (AHAC), represented by American International Underwriters (Phils.), Inc., issued in favor of petitioner Makati Tuscany Condominium Corporation (TUSCANY) Insurance Policy on the latter's building and premises, for a period beginning March 1982 and ending March 1983, with a total premium of P466,103.05. The premium was paid on installments, all of which were accepted by private respondent. In February 1983, private respondent issued to petitioner another Insurance Policy, which replaced and renewed the previous policy, for a term covering 1 March 1983 to 1 March 1984. The premium in the amount of P466,103.05 was again paid on installments. All payments were likewise accepted by private respondent. In January 1984, the policy was again renewed and private respondent issued to petitioner Insurance Policy for the period March 1984 to March 1985. On this renewed policy, petitioner made two installment payments, both accepted by private respondent. Thereafter, petitioner refused to pay the balance of the premium. Consequently, AHAC filed an action to recover the unpaid balance of P314,103.05 for Insurance policy. In its answer with counterclaim, Tuscany admitted the issuance of Insurance Policy. It explained that it discontinued the payment of premiums because the policy did not contain a credit clause in its favor and the receipts for the installment payments covering the policy for 1984-85, as well as the two (2) previous policies, stated the following reservations: (2) Acceptance of this payment shall not waive any of the company rights to deny liability on any claim under the policy arising before such payments or after the expiration of the credit clause of the policy; and (3) Subject to no loss prior to premium payment. If there be any loss such is not covered. Tuscany further claimed that the policy was never binding and valid, and no risk attached to the policy. It then pleaded a counterclaim for P152,000.00 for the premiums already paid for 1984-85, and in its answer with amended counterclaim, sought the refund of P924,206.10 representing the premium payments for 1982-85. Issue: Whether payment by installment of the premiums due on an insurance policy invalidates the contract of insurance. Ruling: No. The subject policies are valid even if the premiums were paid on installments. The records clearly show that Tuscany and AHAC intended subject insurance policies to be binding and effective notwithstanding the staggered payment of the premiums. The initial insurance contract entered into in 1982 was renewed in 1983, then in 1984. In those 3 years, the insurer accepted all the installment payments. Such acceptance of payments speaks loudly of the insurer's intention to honor the policies it issued to Tuscany. Certainly, basic principles of equity and fairness would not allow the insurer to continue collecting and accepting the premiums, although paid on installments, and later deny liability on the lame excuse that the premiums were not prepaid in full. Thus, while the import of Section 77 is that prepayment of premiums is strictly required as a condition to the validity of the contract, the Court was not prepared to rule that the request to make installment payments duly approved by the insurer, would prevent the entire contract of insurance from going into effect despite payment and acceptance of

EH 403 Page 51

⦿ 2010-2011 ⦿ 2011-2012 ⦿

INSURANCE LAW CASE DIGESTS
the initial premium or first installment. Section 78 of the Insurance Code in effect allows waiver by the insurer of the condition of prepayment by making an acknowledgment in the insurance policy of receipt of premium as conclusive evidence of payment so far as to make the policy binding despite the fact that premium is actually unpaid. Section 77 merely precludes the parties from stipulating that the policy is valid even if premiums are not paid, but does not expressly prohibit an agreement granting credit extension, and such an agreement is not contrary to morals, good customs, public order or public policy. So is an understanding to allow insured to pay premiums in installments not so proscribed. At the very least, both parties should be deemed in estoppel to question the arrangement they have voluntarily accepted. It appearing from the peculiar circumstances that the parties actually intended to make the three (3) insurance contracts valid, effective and binding, Tuscany may not be allowed to renege on its obligation to pay the balance of the premium after the expiration of the whole term of the third policy (AH-CPP9210651) in March 1985. Moreover, where the risk is entire and the contract is indivisible, the insured is not entitled to a refund of the premiums paid if the insurer was exposed to the risk insured for any period, however brief or momentary. Digested by: Roxanne A. Huyo Tibay v. CA Facts: On 22 January 1987 private respondent Fortune Life and General Insurance Co., Inc. (FORTUNE) issued Fire Insurance Policy No. 136171 in favor of Violeta R. Tibay and/or Nicolas Roraldo on their two-storey residential building located at 5855 Zobel Street, Makati City, together with all their personal effects therein. The insurance was for P600,000.00 covering the period from 23 January 1987 to 23 January 1988. On 23 January 1987, of the total premium of P2,983.50, petitioner Violeta Tibay only paid P600.00 thus leaving a considerable balance unpaid. On 8 March 1987 the insured building was completely destroyed by fire. On 10 March 1987 Violeta Tibay paid the balance of the premium. On the same day, she filed with FORTUNE a claim on the fire insurance policy. Her claim was accordingly referred to its adjuster, Goodwill Adjustment Services, Inc. (GASI). Petitioner forthwith complied and signed a non-waiver agreement. In a letter dated 11 June 1987, FORTUNE denied the claim of Violeta for violation of Policy Condition No. 2 and of Sec. 77 of the Insurance Code. Efforts to settle the case before the Insurance Commission proved futile. On 19 July 1990 the trial court ruled for petitioners and adjudged FORTUNE liable. On 24 March 1995 the Court of Appeals reversed the court a quo by declaring FORTUNE not to be liable to plaintiff-appellees therein but ordering defendant-appellant to return to the former the premium of P2,983.50 plus 12% interest from 10 March 1987 until full payment. Hence this petition for review… Issue: WON a fire insurance policy is valid, binding and enforceable upon mere partial payment of premium Ruling: The SC finds no merit in the petition; hence, it affirms the Court of Appeals. Insurance is a contract whereby one undertakes for a consideration to indemnify another against loss, damage or liability arising from an unknown or contingent event. The consideration is the premium, which must be paid at the time and in the way and manner specified in the policy, and if not so paid, the policy will lapse and be forfeited by its own terms. The pertinent provisions in the Policy on premium read – THIS POLICY OF INSURANCE WITNESSETH, THAT only after payment to the Company in accordance with Policy Condition No. 2 of the total premiums by the insured as stipulated above for the period aforementioned for insuring against Loss or Damage by Fire or Lightning as herein appears, the Property herein described x x x 2. This policy including any renewal thereof and/or any endorsement thereon is not in force until the premium has been fully paid to and duly receipted by the Company in the manner provided herein. Any supplementary agreement seeking to amend this condition prepared by agent, broker or Company official, shall be deemed invalid and of no effect. xxx xxx xxx Except only in those specific cases where corresponding rules and regulations which are or may hereafter be in force provide for the payment of the stipulated premiums in periodic installments at fixed percentage, it is hereby declared, agreed and warranted that this policy shall be deemed effective, valid and binding upon the Company only when the premiums therefor have actually been paid in full and duly acknowledged in a receipt signed by any authorized official or representative/agent of the Company in such manner as provided herein. Clearly the Policy provides for payment of premium in full. Accordingly, where the premium has only been partially paid and the balance paid only after the peril insured against has occurred, the insurance contract did not take effect and the insured cannot collect at all on the policy. This is fully supported by Sec. 77 of the Insurance Code which provides –

EH 403 Page 52

⦿ 2010-2011 ⦿ 2011-2012 ⦿

INSURANCE LAW CASE DIGESTS
SEC. 77. An insurer is entitled to payment of the premium as soon as the thing insured is exposed to the peril insured against. Notwithstanding any agreement to the contrary, no policy or contract of insurance issued by an insurance company is valid and binding unless and until the premium thereof has been paid, except in the case of a life or an industrial life policy whenever the grace period provision applies. Apparently the crux of the controversy lies in the phrase “unless and until the premium thereof has been paid.” In Escosura v. San Miguel Brewery, Inc., the Court through Mr. Justice Jesus G. Barrera, interpreting the phrase “with pay” used in connection with leaves of absence with pay granted to employees, ruled - x x x the legislative practice seems to be that when the intention is to distinguish between full and partial payment, the modifying term is used x x x. Furthermore, the two (2) cases, Phoenix and Tuscany, adequately demonstrate the waiver, either express or implied, of prepayment in full by the insurer: impliedly, by suing for the balance of the premium as in Phoenix, and expressly, by agreeing to make premiums payable in installments as in Tuscany. But contrary to the stance taken by petitioners, there is no waiver express or implied in the case at bench. Conformably with the aforesaid stipulations explicitly worded and taken in conjunction with Sec. 77 of the Insurance Code the payment of partial premium by the assured in this particular instance should not be considered the payment required by the law and the stipulation of the parties. Rather, it must be taken in the concept of a deposit to be held in trust by the insurer until such time that the full amount has been tendered and duly receipted for. In other words, as expressly agreed upon in the contract, full payment must be made before the risk occurs for the policy to be considered effective and in force. Thus, no vinculum juris whereby the insurer bound itself to indemnify the assured according to law ever resulted from the fractional payment of premium. The insurance contract itself expressly provided that the policy would be effective only when the premium was paid in full. It would have been altogether different were it not so stipulated. Ergo, petitioners had absolute freedom of choice whether or not to be insured by FORTUNE under the terms of its policy and they freely opted to adhere thereto. Applying further the rules of statutory construction, the case of South Sea Surety and Insurance Company, Inc. v. Court of Appeals, speaks only of two (2) statutory exceptions to the requirement of payment of the entire premium as a prerequisite to the validity of the insurance contract. These exceptions are: (a) in case the insurance coverage relates to life or industrial life (health) insurance when a grace period applies, and (b) when the insurer makes a written acknowledgment of the receipt of premium, this acknowledgment being declared by law to, be then conclusive evidence of the premium payment. The express mention of exceptions operates to exclude other exceptions; conversely, those which are not within the enumerated exceptions are deemed included in the general rule. Thus, under Sec. 77, as well as Sec. 78, until the premium is paid, and the law has not expressly excepted partial payments, there is no valid and binding contract. Hence, in the absence of clear waiver of prepayment in full by the insurer, the insured cannot collect on the proceeds of the policy. iii. Non-Default Options in Life Insurance iv. Reinstatement of a Lapsed Policy of Life Insurance v. Acknowledgement in policy of receipt of premium is conclusive (Section 78) Digested by: Patricia Ko American Home Assurance vs. Chua [G.R. No. 130421. June 28, 1999] FACTS: Petitioner, American Home, is a domestic corporation engaged in the insurance business. Sometime in 1990, respondent Chua obtained from petitioner a fire insurance covering the stock-in-trade of his business, Moonlight Enterprises, located at Valencia, Bukidnon. The insurance was due to expire on 25 March 1990. On 5 April 1990 respondent issued PCIBank Check No. 352123 in the amount of P2,983.50 to petitioner’s agent, James Uy, as payment for the renewal of the policy. In turn, the latter delivered Renewal Certificate No. 00099047 to respondent. The check was drawn against a Manila bank and deposited in petitioner’s bank account in Cagayan de Oro City. The corresponding official receipt was issued on 10 April. Subsequently, a new insurance policy, Policy No. 206-4234498-7, was issued, whereby petitioner undertook to indemnify respondent for any damage or loss arising from fire up toP200,000 for the period 25 March 1990 to 25 March 1991. On 6 April 1990 Moonlight Enterprises was completely razed by fire. Total loss was estimated between P4,000,000 and P5,000,000. Respondent filed an insurance claim with petitioner and four other co-insurers, namely, Pioneer Insurance and Surety Corporation, Prudential Guarantee and Assurance, Inc., Filipino Merchants Insurance Co. and Domestic Insurance Company of the Philippines. Petitioner refused to honor the claim notwithstanding several demands by respondent, thus, the latter filed an action against petitioner before the trial court. In its defense, petitioner claimed there was no existing insurance contract when the fire occurred since respondent did not pay the premium. It also alleged that even assuming there was a contract, respondent violated several conditions of the policy,

EH 403 Page 53

⦿ 2010-2011 ⦿ 2011-2012 ⦿

INSURANCE LAW CASE DIGESTS
particularly: (1) his submission of fraudulent income tax return and financial statements; (2) his failure to establish the actual loss, which petitioner assessed at P70,000; and (3) his failure to notify to petitioner of any insurance already effected to cover the insured goods. These violations, petitioner insisted, justified the denial of the claim. Petitioner emphasizes that when the fire occurred on 6 April 1990 the insurance contract was not yet subsisting pursuant to Article 1249[3] of the Civil Code, which recognizes that a check can only effect payment once it has been cashed. Although respondent testified that he gave the check on 5 April to a certain James Uy, the check, drawn against a Manila bank and deposited in a Cagayan de Oro City bank, could not have been cleared by 6 April, the date of the fire. In fact, the official receipt issued for respondent’s check payment was dated 10 April 1990, four days after the fire occurred. Citing jurisprudence,[4] petitioner also contends that respondent’s non-disclosure of the other insurance contracts rendered the policy void. Respondent refutes the reason for petitioner’s denial of his claim. To bolster his argument, respondent cites Section 66 of the Insurance Code,[5] which requires the insurer to give a notice to the insured of its intention to terminate the policy forty-five days before the policy period ends. In the instant case, petitioner opted not to terminate the policy. Instead, it renewed the policy by sending its agent to respondent, who was issued a renewal certificate upon delivery of his check payment for the renewal of premium. At this precise moment the contract of insurance was executed and already in effect. Respondent also claims that it is standard operating procedure in the provinces to pay insurance premiums by check when collected by insurance agents. ISSUES: 1. whether there was a valid payment of premium, considering that respondent’s check was cashed after the occurrence of the fire; 2. whether respondent violated the policy by his submission of fraudulent documents and nondisclosure of the other existing insurance contracts; RULING: On the payment of premium through check (Relevant provision: Sec. 78) The general rule in insurance laws is that unless the premium is paid the insurance policy is not valid and binding. The only exceptions are life and industrial life insurance.[6] Whether payment was indeed made is a question of fact which is best determined by the trial court. The trial court found, as affirmed by the Court of Appeals, that there was a valid check payment by respondent to petitioner. Well-settled is the rule that the factual findings and conclusions of the trial court and the Court of Appeals are entitled to great weight and respect, and will not be disturbed on appeal in the absence of any clear showing that the trial court overlooked certain facts or circumstances which would substantially affect the disposition of the case.[7] We see no reason to depart from this ruling. According to the trial court the renewal certificate issued to respondent contained the acknowledgment that premium had been paid. It is not disputed that the check drawn by respondent in favor of petitioner and delivered to its agent was honored when presented and petitioner forthwith issued its official receipt to respondent on 10 April 1990. Section 306 of the Insurance Code provides that any insurance company which delivers a policy or contract of insurance to an insurance agent or insurance broker shall be deemed to have authorized such agent or broker to receive on its behalf payment of any premium which is due on such policy or contract of insurance at the time of its issuance or delivery or which becomes due thereon.[8] In the instant case, the best evidence of such authority is the fact that petitioner accepted the check and issued the official receipt for the payment. It is, as well, bound by its agent’s acknowledgment of receipt of payment. Section 78 of the Insurance Code explicitly provides: An acknowledgment in a policy or contract of insurance of the receipt of premium is conclusive evidence of its payment, so far as to make the policy binding, notwithstanding any stipulation therein that it shall not be binding until the premium is actually paid. This Section establishes a legal fiction of payment and should be interpreted as an exception to Section 77. On whether there were conditions stipulated violations of the policy

Is respondent guilty of the policy violations imputed against him? We are not convinced by petitioner’s arguments. Ordinarily, where the insurance policy specifies as a condition the disclosure of existing coinsurers, non-disclosure thereof is a violation that entitles the insurer to avoid the policy. This condition is common in fire insurance policies and is known as the “other insurance clause.” The purpose for the inclusion of this clause is to prevent an increase in the moral hazard. We have ruled on its validity and the case of Geagonia v. Court of Appeals[10] clearly illustrates such principle. However, we see an exception in the instant case. Citing Section 29[11] of the Insurance Code, the trial court reasoned that respondent’s failure to disclose was not intentional and fraudulent. The application of Section 29 is misplaced. Section 29 concerns concealment which is intentional. The relevant provision is Section 75, which provides that: A policy may declare that a violation of specified provisions thereof shall avoid it, otherwise the breach of an immaterial provision does not avoid the policy.

EH 403 Page 54

⦿ 2010-2011 ⦿ 2011-2012 ⦿

INSURANCE LAW CASE DIGESTS
To constitute a violation the other existing insurance contracts must be upon the same subject matter and with the same interest and risk.[12] Indeed, respondent acquired several co-insurers and he failed to disclose this information to petitioner. Nonetheless, petitioner is estopped from invoking this argument. The trial court cited the testimony of petitioner’s loss adjuster who admitted previous knowledge of the co-insurers. Indubitably, it cannot be said that petitioner was deceived by respondent by the latter’s non-disclosure of the other insurance contracts when petitioner actually had prior knowledge thereof. Petitioner’s loss adjuster had known all along of the other existing insurance contracts, yet, he did not use that as basis for his recommendation of denial. The loss adjuster, being an employee of petitioner, is deemed a representative of the latter whose awareness of the other insurance contracts binds petitioner. We, therefore, hold that there was no violation of the “other insurance” clause by respondent. vi. Return of premium (Sections 79-82) GREAT PACIFIC LIFE ASSURANCE CORP. vs. COURT OF APPEALS AND MEDARDA V. LEUTERIO FACTS This is a petition for review under Rule 45 of the Rules of Court, assailing the decision and resolution of the Court of Appeals dated May 17, 1994 and January 4, 1994, respectively, in CA G.R. CV No. 18341. The appellate court affirmed in toto the judgment of the Regional Trial Court of Misamis Oriental in an insurance claim filed by private respondent against Great Pacific Life Assurance Co. The Supreme Court found the petition not meritorious. Contrary to petitioner's allegations, there was no sufficient proof that the insured had suffered from hypertension. Aside from the statement of the insured's widow who was not even sure if the medicines taken by Dr. Leuterio were for hypertension, the petitioner had not proven nor produced any witness who could attest to Dr. Leuterio's medical history. Clearly, it had failed to establish that there was concealment made by the insured, hence it cannot refuse payment of the claim. ISSUES 1. Whether the Court of Appeals erred in holding petitioner liable to DBP as beneficiary in a group life insurance contract from a complaint filed by the widow of the decedent/mortgagor? NO 2. Whether Dr. Leuterio concealed that he had hypertension, which would vitiate the insurance contract? NO HELD 1. The rationale of a group insurance policy of mortgagors, otherwise known as the "mortgage redemption insurance," is a device for the protection of both the mortgagee and the mortgagor. On the part of the mortgagee, it has to enter into such form of contract so that in the event of the unexpected demise of the mortgagor during the subsistence of the mortgage contract, the proceeds from such insurance will be applied to the payment of the mortgage debt, thereby relieving the heirs of the mortgagor from paying the obligation. In a similar vein, ample protection is given to the mortgagor under such a concept so that in the event of death; the mortgage obligation will be extinguished by the application of the insurance proceeds to the mortgage indebtedness. Consequently, where the mortgagor pays the insurance premium under the group insurance policy, making the loss payable to the mortgagee, the insurance is on the mortgagor's interest, and the mortgagor continues to be a party to the contract. In this type of policy insurance, the mortgagee is simply an appointee of the insurance fund, such loss-payable clause does not make the mortgagee a party to the contract. The insured private respondent did not cede to the mortgagee all his rights or interests in the insurance, the policy stating that: "In the event of the debtor's death before his indebtedness with the Creditor [DBP] shall have been fully paid, an amount to pay the outstanding indebtedness shall first be paid to the creditor and the balance of sum assured, if there is any, shall then be paid to the beneficiary/ies designated by the debtor." When DBP submitted the insurance claim against petitioner, the latter denied payment thereof, interposing the defense of concealment committed by the insured. Thereafter, DBP collected the debt from the mortgagor and took the necessary action of foreclosure on the residential lot of private respondent. In Gonzales La O vs. Yek Tong Lin Fire & Marine Ins. Co. we held: "Insured, being the person with whom the contract was made, is primarily the proper person to bring suit thereon. . . . Subject to some exceptions, insured may thus sue, although the policy is taken wholly or in part for the benefit of another person named or unnamed, and although it is expressly made payable to another as his interest may appear or otherwise. . . . Although a policy issued to a mortgagor is taken out for the benefit of the mortgagee and is made payable to him, yet the mortgagor may sue thereon in his own name, especially where the mortgagee's interest is less than the full amount recoverable under the policy, . . . 'And in volume 33, page 82, of the same work, we read the following: `Insured may be regarded as the real party in interest, although he has assigned the policy for the purpose of collection, or has assigned as collateral security any judgment he may obtain." And since a policy of insurance upon life or health may pass by transfer, will or succession to any person, whether he has an insurable interest or not, and such person may recover it whatever the insured might have recovered, the widow of the decedent Dr. Leuterio may file the suit against the insurer, Grepalife.

EH 403 Page 55

⦿ 2010-2011 ⦿ 2011-2012 ⦿

INSURANCE LAW CASE DIGESTS
2. The question of whether there was concealment was aptly answered by the appellate court, thus: "The insured, Dr. Leuterio, had answered in his insurance application that he was in good health and that he had not consulted a doctor for any of the enumerated ailments, including hypertension; when he died the attending physician had certified in the death certificate that the former died of cerebral hemorrhage, probably secondary to hypertension. From this report, the appellant insurance company refused to pay the insurance claim. Appellant alleged that the insured had concealed the fact that he had hypertension. Contrary to appellant's allegations, there was no sufficient proof that the insured had suffered from hypertension. Aside from the statement of the insured's widow who was not even sure if the medicines taken by Dr. Leuterio were for hypertension, the appellant had not proven nor produced any witness who could attest to Dr. Leuterio's medical history . . . Appellant insurance company had failed to establish that there was concealment made by the insured, hence, it cannot refuse payment of the claim." The fraudulent intent on the part of the insured must be established to entitle the insurer to rescind the contract. Misrepresentation as a defense of the insurer to avoid liability is an affirmative defense and the duty to establish such defense by satisfactory and convincing evidence rests upon the insurer. In the case at bar, the petitioner failed to clearly and satisfactorily establish its defense, and is therefore liable to pay the proceeds of the insurance. I.Loss (Sections 83-87) Digested by: dlanor Paris-Manila Perfume Co. vs. Phoenix Assurance Co. Facts: On May 22, 1924, defendant (Phoenix Assurance Co.) issued to plaintiff (Paris-Manila Perfume Co.) its fire insurance policy No. 841163 in the sum of P13,000 upon the property of the plaintiff at No. 1 Calle Cisneros, Cavite, insuring plaintiff's property against fire for that amount. That on July 4, 1924, the property covered by the insurance was completely destroyed by fire for the total loss to the plaintiff of P38.025.56; that under its policy with the defendant, it promptly presented its claim but the defendant wrongfully and unjustly refused to pay it. Defendant contended among others that the policy of insurance did not cover any loss or damage occasioned by explosion, and the loss was occasioned by an explosion, and was not covered by the policy. The lower court rendered judgment in favor of the plaintiff. Issues: W/N Whether or not the loss of the property was caused mainly by explosion and thus not covered by the insurance policy. Ruling: The real cause of the fire is more or less a matter of conjecture, upon which there is little, if any, evidence. But the fact remains that there was a fire, and that the plaintiffs property was destroyed. It is true that it may be that the explosion was the primary cause of the fire, but that is only a matter of conjecture, and upon that point, the burden of proof was upon the defendant. Defendant relies upon section 6 of the policy, as follows: 6. Unless otherwise expressly stated in the policy the insurance does not cover — (h) Loss or damage occasioned by the explosion; but loss or damage by explosion of gas for illuminating or domestic purposes in a building in which gas is not generated and which does not form a part of any gas works, will be deemed to be loss by fire within the meaning of this policy. In answer to that, plaintiff relies upon section 5, which is as follows: 5. The insurance does not cover — (d) Loss or damage occasioned directly or indirectly, approximately or remotely by or through or in consequence of: (1) Earthquake, hurricane, volcanic eruption or other convulsion of nature, and the company shall not be liable for loss or damage arising during or within a reasonable time after any of the said occurrences, unless it be proved by the insured to the satisfaction of the company that such loss or damage was not in any way occasioned by or through or in consequence of any of the said occurrences Examination of the above provisions would reveal that section 5 excludes not only the damages which may immediately result from an earthquake, but also any damage which may follow the earthquake, and that section 6 excludes only the damages which are the direct result of the explosion itself, and that it does not except damages which occurred from the fire occuring after the explosion, even though the explosion may have been the primary cause of the fire. Moreover, there was no competent evidence as to whether the explosion caused the fire or the fire caused the explosion. -sagabalDigested by: Ichelle S. Malabuyoc

EH 403 Page 56

⦿ 2010-2011 ⦿ 2011-2012 ⦿

INSURANCE LAW CASE DIGESTS
THE EAST FURNITURE INC vs. THE GLOBE & RUTGERS FIRE INSURANCE CO. OF NEW YORK [G.R. No. L-35848 November 22, 1932] Facts: Plaintiff is a duly registered partnership engaged in the sale of furniture; that the defendant is a company engaged in the insurance business. Plaintiff insured against fire the articles existing in its establishment. On March 2, 1929, a fire broke out in plaintiff's establishment, as a result of which the insured articles therein found were destroyed by the fire. Defendants defenses are:(1) that the fire in question was of intentional origin; (2) that the claims of loss presented by the plaintiff were false and fraudulent; (3) that the furniture in question had been mortgaged by the plaintiff to the Manila Finance and Discount Corporation, so that at the time of the fire the plaintiff was not the only party interested therein, contrary to the representations made in its claims of loss; and (4) that the plaintiff violated one of the conditions of the policies by refusing to furnish the defendants with a physical inventory of the contents of its store at the time of the fire. By agreement of the parties the three cases were tried jointly who after the trial found that the claims presented by the plaintiff were notoriously fraudulent, and, accordingly, sustained defendant's second special defense and dismissed the complaint in each of the three cases. Hence, this petition. Issue: W/N fire was of intentional origin W/N the claim of loss were fraudulent Ruling: Fire of Intentional Origin We are thus led to the conclusion that defendants' first special defense is well founded — that the fire in question was of intentional origin and was caused with the connivance of the plaintiff. Neither the interest of the justice nor public policy would be promoted by an omission of the courts to expose and condemn incendiarism once the same is established by competent evidence. It would tend to encourage rather than suppress that great public menace if the courts do not expose the crime to public condemnation when the evidence in a case like the present shows that it has really been committed. Fraudulent claim of loss We may also consider the damage caused by the fire in relation with defendant's second special defense that plaintiff's claims of loss were false and fraudulent. To each of the proofs of loss which the plaintiff presented to the respective insurance companies four days after the fire was attached an inventory of the furniture claimed to have been in the building at the time of the fire. This inventory contains 506 pieces of furniture and 3,700 board feet of lumber of the alleged total value of P52,061.99. This amount was the total loss claimed to have been suffered by the plaintiff, although we note that in its complaints in these cases amended it is conceded that some furniture of the value of about P5,000 was saved. Regardless of any difference of opinion as to the value of the insured furniture and the extent of the damage caused thereto by the fire in question, the fact that the insured only had approximately 202 pieces of furniture in the building at the time of the fire and sought to compel the insurance companies to pay for 506 pieces conclusively shows that its claim was not honestly conceived. The trial court's conclusion that said claim is notoriously fraudulent, is correct. Condition 12 of each of the insurance policies sued upon provides that "if the claim be in any respect fraudulent, or if any false declaration be made or used in support thereof, or if any fraudulent means or devices are used by the Insured or anyone acting on his behalf to obtain any benefit under this policy; or, if the loss or damage be occasioned by the wilful act, or with the connivance of the Insured, — all benefit under this policy shall be forfeited." J.Notice and Proof of Loss (Sections 88-92) Sharruf & Co. vs Baloise Fire Ins. Co., [GR No. 44119, March 30, 1937] Facts: Salomon Sharruf and Eskenazi were doing business under the firm name of Sharuff & Co. They applied for insurance the merchandise they had in stock. The insurance companies (Baloise Fire Ins. Co., Sun Insurance Office, Springfield Insurance Co.) sent their representative to asses and examine the goods. As a result, the insurance companies issued insurance policies with the total amount of 40,000.00. They then entered into a partnership, substituting the name Sharruf & Co. with the Sharruf & Eskanzi. About 12.41 o’clock on the morning of September 22, 1933, a fire broke out, burning and damaging the merchandise insured by Sharruf and Eskanzi. Issue: Whether or not the claim of loss filed by the plaintiffs is fraudulent. Ruling: With respect to the question whether or not the claim of loss filed by the plaintiffs is fraudulent, it is alleged by them that the total value of the textiles contained in cases deposited inside the building when the partnership Sharruf & Eskenazi was formed was P12,000; that of the fancy jewelry with imitation stones from P15,000 to P17,000, and that of the

EH 403 Page 57

⦿ 2010-2011 ⦿ 2011-2012 ⦿

INSURANCE LAW CASE DIGESTS
kitchen utensils and tableware made of aluminum, bronze and glass P10,676. If, as said plaintiffs claim, they had already sold articles, mostly textiles, valued at P8,000, a small quantity of cloth must have been left at the time the fire occured. In their claim, however, the textiles allegedly consumed by fire and damaged by water are assessed by them at P12,000. The claim of P12,000 is certainly not attributable to a mere mistake in estimate and counting because if they had textiles worth only P12,000 before the fire and they sold goods, mostly textiles, worth P8,000, surely textiles in the same amount of P12,000 could not have been burned and damaged after the fire. Of the kitchen utensils and tableware made of aluminum, bronze and glass, of which, according to the evidence for the plaintiffs, they had a stock valued at P10,676, there were found after the fire articles worth only P1,248.80. Therefore, utensils valued at P9,427.20 were lacking. A considerable amount of kitchen utensils made of noninflammable and fire-proof material could not, by the very nature of things have been totally consumed by the fire. At most, said articles would have been damaged, as the rest, and would have left traces of their existence. The same may be said of the fancy jewels with imitation stones, and others of which the fancy jewels with imitation stones, and others of which the plaintiffs claim to have had a stock worth from P15,000 to P17,000 at the time of the fire, of which only a few valued at P3,471.16, were left after the fire. According to said plaintiffs, all the articles, for the alleged loss of which indemnity is sought, were contained in about 40 showcases and wardrobes. According to the testimony of the fire station chiefs, corrobarated by the photographs of record, the flames caused more damage in the upper part of the rooms than in the lower part thereof; since, of the ten or eleven cases found inside the building after the fire, only a few were partially burned and others scorched judging from their appearance, the goods were damaged more by water than by fire. According to the inventory made by White & Page, adjusters of the insurance companies, in the presence of the plaintiffs themselves and according to data supplied by the latter, the total value thereof, aside, from the articles not included in the inventories Exhibits B, C, and C-1, assessed at P744.50, amounts to only P8,077.35. If the plaintiffs' claim that at time of the fire there were about 40 cases inside the burnt building were true, a ten or eleven of them were found after the fire, traces of the thirty or twenty-nine cases allegedly burnt would be found, since experience has shown that during the burning of a building all the cases deposited therein are not so reduced to ashes that the least vestige thereof cannot be found. In the case of Go Lu vs. Yorkshire Insurance Co. (43 Phil., 633), this court laid down the following doctrine: This court will legally presume that in an ordinary fire fifty bales or boxes of bolt goods of cloth cannot be wholly consumed or totally destroyed, and that in the very nature of things some trace or evidence will be left remaining of their loss or destruction. The plaintiffs, upon whom devolve the legal obligation to prove the existence, at the time of the fire, of the articles and merchandise for the destruction of which they claim indemnity from the defendant companies, have not complied with their duty because they have failed to prove by a preponderance of evidence that when the fire took place there where in the burnt building articles and merchandise in the total amount of the insurance policies or that the textiles and other damaged and undamaged goods found in the building after the fire were worth P40,000. On the contrary, their own witness, Robles, testified that up to the month of September, 1933, there were about 39 or 40 cases belonging to the plaintiffs in his garage on Mabini Street, indicating thereby that the cases of merchandise examined by the agent of the insurance companies on July 25 and August 15, 1933, and for which the insurance policies were issued, were taken from the burned building where they were found. So great is the difference between the amount of articles insured, which the plaintiffs claim to have been in the building before the fire, and the amount thereof shown by the vestige of the fire to have been therein, that the most liberal human judgment can not attribute such difference to a mere innocent error in estimate or counting but to a deliberate intent to demand of the insurance companies payment of an indemnity for goods not existing at the time of the fire, thereby constituting the so-called "fraudulent claim" which, by express agreement between the insurers and the insured, is a ground for exemption of the insurers from civil liability. Therefore, as the herein plaintiffs-appellees have acted in bad faith in presenting a fraudulent claim, they are not entitled to the indemnity claimed by them by virtue of the insurance policies issued by the defendantappellant companies in their favor. Author: Aldrin Montesco Malayan Ins. Co. Vs. Cruz Arnaldo [154 SCRA 672] FACTS: On June 7, 1981, the petitioner (hereinafter called (MICO) issued to the private respondent, Coronacion Pinca, Fire Insurance Policy No. F-001-17212 on her property for the amount of P14,000.00 effective July 22, 1981, until July 22, 1982 On October 15,1981, MICO allegedly cancelled the policy for non-payment, of the premium and sent the corresponding notice to Pinca. On December 24, 1981, payment of the premium for Pinca was received by Domingo Adora, agent of MICO. On January 15, 1982, Adora remitted this payment to MICO,together with other payments. On January 18, 1982, Pinca's property was completely burned.

EH 403 Page 58

⦿ 2010-2011 ⦿ 2011-2012 ⦿

INSURANCE LAW CASE DIGESTS
On February 5, 1982, Pinca's payment was returned by MICO to Adora on the ground that her policy had been cancelled earlier. But Adora refused to accept it. In due time, Pinca made the requisite demands for payment, which MICO rejected. She then went to the Insurance Commission. It is because she was ultimately sustained by the public respondent that the petitioner has come to us for relief. ISSUE: Whether or not the cancellation of the policy by the insurance company was valid RULING: The Supreme Court ruled in favor of the insured. The above provision (Sec. 77 as invoked by the insurance company) is not applicable because payment of the premium was in fact eventually made in this case. Notably, the premium invoice issued to Pinca at the time of the delivery of the policy on June 7, 1981 was stamped "Payment Received" of the amount of P930.60 on "12-24-81" by Domingo Adora. This is important because it suggests an understanding between MICO and the insured that such payment could be made later, as agent Adora had assured Pinca. In any event, it is not denied that this payment was actually made by Pinca to Adora, who remitted the same to MICO. It is not disputed that the premium was actually paid by Pinca to Adora on December 24, 1981, who received it on behalf of MICO, to which it was remitted on January 15, 1982. What is questioned is the validity of Pinca's payment and of Adora's authority to receive it. MICO's acknowledgment of Adora as its agent defeats its contention that he was not authorized to receive the premium payment on its behalf. It is clearly provided in Section 306 of the Insurance Code. And it is a well-known principle under the law of agency that: Payment to an agent having authority to receive or collect payment is equivalent to payment to the principal himself; such payment is complete when the money delivered is into the agent's hands and is a discharge of the indebtedness owing to the principal. The policy could be cancelled on any of the supervening grounds enumerated in Article 64 (except "non-payment of premium") provided the cancellation was made in accordance therewith and with Article 65. Section 64 reads as follows: SEC. 64. No policy of insurance other than life shall be cancelled by the insurer except upon prior notice thereof to the insured, and no notice of cancellation shall be effective unless it is based on the occurrence, after the effective date of the policy, of one or more of the following: (a) non-payment of premium; (b) conviction of a crime arising out of acts increasing the hazard insured against; (c) discovery of fraud or material misrepresentation; (d) discovery of willful, or reckless acts or commissions increasing the hazard insured against; (e) physical changes in the property insured which result in the property becoming uninsurable; or (f) a determination by the Commissioner that the continuation of the policy would violate or would place the insurer in violation of this Code. As for the method of cancellation, Section 65 provides as follows: SEC. 65. All notices of cancellation mentioned in the preceding section shall be in writing, mailed or delivered to the named insured at the address shown in the policy, and shall state (a) which of the grounds set forth in section sixty-four is relied upon and (b) that, upon written request of the named insured, the insurer will furnish the facts on which the cancellation is based. A valid cancellation must, therefore, require concurrence of the following conditions: (1) There must be prior notice of cancellation to the insured; (2) The notice must be based on the occurrence, after the effective date of the policy, of one or more of the grounds mentioned; (3) The notice must be (a) in writing, (b) mailed, or delivered to the named insured, (c) at the address shown in the policy; (4) It must state (a) which of the grounds mentioned in Section 64 is relied upon and (b) that upon written request of the insured, the insurer will furnish the facts on which the cancellation is based. Thus, it behooved MICO's to make sure that the cancellation was actually sent to and received by the insured. No such proof in support thereto was presented by the insurance company. As it has not been shown that there was a valid cancellation of the policy, there was consequently no need to renew it but to pay the premium thereon. Payment was thus legally made on the original transaction and it could be, and was, validly received on behalf of the insurer by its agent Adora. Adora, incidentally, had not been informed of the cancellation either and saw no reason not to accept the said payment.

EH 403 Page 59

⦿ 2010-2011 ⦿ 2011-2012 ⦿

INSURANCE LAW CASE DIGESTS
Lastly, Loss and its amount may be determined on the basis of such proof as may be offered by the insured, which need not be of such persuasiveness as is required in judicial proceedings. 25 If, as in this case, the insured files notice and preliminary proof of loss and the insurer fails to specify to the former all the defects thereof and without unnecessary delay, all objections to notice and proof of loss are deemed waived under Section 90 of the Insurance Code. Guidelines on Claims Settlement a. Unfair Claims Settlement; Sanctions b. Prescription of Action c. Subrogation L. Double Insurance (Sections 93-94) i. Double Insurance (Section 93) ii. Over-Insurance (Section 94) M. Reinsurance (Sections 95-98) Digested by: Archie Necesario Phil American Life Insurance vs Auditor General Facts: On January 1, 1950, Philippine American Life Insurance Company (Philamlife), a domestic life insurance corporation, and American International Reinsurance Company (AIRCO) a corporation organized under the laws of the Republic of Panama, entered into a reinsurance treaty. On their agreement, “Philamlife agrees to reinsure with AIRCO the entire first excess of such life insurance on the lives of persons as may be written by the Philamlife under direct application over and above its maximum limit of retention for life insurance, and AIRCO binds itself to accept such reinsurances on the same terms. It is also stipulated that when new policies are applied for and issued Philamlife can cede automatically any amount, within the limits.” On July 16, 1959, the Philippine government enacted the Margin Law which provides for the payment of margin fee for all sales of foreign exchange. Pursuant to the law, the Central Bank of the Philippines collected the sum of P268,747.48 as foreign exchange margin on Philamlife remittances to Airco. Philamlife subsequently filed with the Central Bank a claim for the refund of the above sum of P268,747.48. The ground therefor was that the reinsurance premiums so remitted were paid pursuant to the reinsurance treaty, and, therefore, were pre-existing obligations expressly exempt by the Margin Law from the margin fee. Auditor General ruled that "[r]emittance of premia on insurance policies issued or renewed on or after July 16, 1959, or even if issued or renewed before the said date, but their reinsurance was effected, only thereafter, are not exempt from the margin fee, even if the reinsurance treaty under which they are K. reinsured was approved by the Central Bank before July 16, 1959. Issue: Whether or not Philamlife is exempt from paying the margin fee by virtue of the pre-existing obligation made by the reinsurance treaty. Ruling: There should not be any misapprehension as to the distinction between a reinsurance treaty, on the one hand, and a reinsurance policy or a reinsurance cession, on the other. The concept of one and the other is well expressed thus: . . . A reinsurance policy is thus a contract of indemnity one insurer makes with another to protect the first insurer from a risk it has already assumed. . . . In contradistinction a reinsurance treaty is merely an agreement between two insurance companies whereby one agrees to cede and the other to accept reinsurance business pursuant to provisions specified in the treaty. The practice of issuing policies by insurance companies includes, among other things, the issuance of reinsurance policies on standard risks and also on substandard risks under special arrangements. The lumping of the different agreements under a contract has resulted in the term known to the insurance world as "treaties." Such a treaty is, in fact, an agreement between insurance companies to cover the different situations described. Reinsurance treaties and reinsurance policies are not synonymous. Treaties are contracts for insurance; reinsurance policies or cessions . . . are contracts of insurance. Nothing in that treaty, however, obligates Philamlife to remit to AIRCO a fixed, certain, and obligatory sum by way of reinsurance premiums. All that the reinsurance treaty provides on this point is that Philamlife "agrees to reinsure." The treaty speaks of a probability; not a reality. For, without reinsurance, no premium is due. Of course, the reinsurance treaty lays down the duty to remit premiums — if any reinsurance is effected upon the covenants in that treaty written. So it is that the reinsurance treaty per se cannot give rise to a contractual obligation calling for the payment of foreign exchange "issued, approved and outstanding as of the date this Act [Republic Act 2609] takes effect." For an exemption to come into play, there must be a reinsurance policy or, as in the reinsurance treaty provided, a "reinsurance cession" which may be automatic or facultative. Philamlife's obligation to remit reinsurance premiums becomes fixed and definite upon the execution of the reinsurance cession. Because, for every life insurance policy ceded to Airco, Philamlife agrees to pay premium. 12It is only after a reinsurance cession is made that payment of reinsurance premium may be

EH 403 Page 60

⦿ 2010-2011 ⦿ 2011-2012 ⦿

INSURANCE LAW CASE DIGESTS
exacted, as it is only after Philamlife seeks to remit that reinsurance premium that the obligation to pay the margin fee arises. Upon the premise that the margin fee of P268,747.48 was collected on remittances made on reinsurance effected on or after the Margin Law took effect, refund thereof does not come within the coverage of the exemption circumscribed in Section 3 of the said law. The petition for review is hereby denied, and the ruling of the Auditor General denying refund is hereby affirmed. Digested by: Sharmine M. Odchigue Fieldmen’s Ins. Co. v. Asian Surety and Ins. Co. [34 SCRA 36] Facts: Between April 11, 1960 and January 9, 1961 — the Asian Surety & Insurance Company, Inc. (ceding company) and the Fieldmen's insurance Company, Inc. (reinsuring company) entered into seven (7) reinsurance agreements or treaties 1 under the general terms of which the former, undertook to cede to the latter, a specified portion of the amount of insurance underwritten by ASIAN upon payment to FIELDMEN'S of a proportionate share of the gross rate of the premium applicable with respect to each cession after deducting a commission. Said agreements or treaties were to, take effect from certain specific dates and were to be in force until cancelled by either party upon previous notice of at least three (3) months by registered mail to the other party, the cancellation to take effect as of the 31st of December of the year in which the notice was given. On September 19, 1961( 1st letter) FIELDMEN'S, by means of registered mail, served notice to ASIAN to be relieved from all participation in its various treaties with the latter effective December 31, 1961. This communication, although admittedly received by ASIAN on September 25, 1961, did not elicit any reply from ASIAN. On December 7, 1961 (2nd letter) FIELDMEN'S sent another letter to ASIAN reiterated its position that it would consider itself "no longer at risk for any reinsurance and/or cession" given by ASIAN which might be in force on December 31, 1961. Not having received any formal reply from ASIAN, FIELDMEN'S sent anew a letter on February 17, 1962 (3rd letter) reminding ASIAN of the December 7 letter regarding the cancellation of all the reinsurance treaties and cessions as of December 31, 1961. At the same time FIELDMEN'S requested ASIAN to submit its final accounting of all cessions made to the former for the preceding months when the reinsurance agreements were in force. Meanwhile one of the risks reinsured with FIELDMENS under Cession No. 61-87, Policy No. RI-1236, issued in favor of the Government Service Insurance System, became a liability when the insured property was burned on February 16, 1962. Since the policy was issued on July 1, 1961, it was supposed to expire on July 1, 1962. 2 The next day, February 17, ASIAN immediately notified FIELDMEN'S of said fire loss. And on February 26, 1962 ASIAN sent its reply stating, among other things, as follows: ... we beg to reiterate that your letter of December 7, 1961, terminating said treaties by December 31, 1961, is not in accordance with the terms thereof, since there was no prior three months' notice. However, considering the attitude express (sic) in your aforesaid letter of December 7, 1961, we are willing to waive provision that said treaties may be cancelled on December 31st of any year, and will consider them cancelled at the end of three (3) months from December 7, 1961, by which time we shall be able to render the final accounting you desire. FIELDMEN'S, filed a petition for declaratory relief with the Court of First Instance of Manila to seek a declaration that all the reinsurance contracts entered into between them had terminated as of December 31, 1961 and to obtain an order directing ASIAN to render final accounting of the transactions between them with respect to said reinsurance treaties as of the cut-off date. Trial Court declared six 3 of the seven 4 insurance agreements in question cancelled as of December 31, 1961 and upheld ASIAN'S position that all cessions of reinsurance made by it to FIELDMEN'S prior to the cancellation of the reinsurance treaties continued in full force and effect until expiry dates Court of Appeals affirmed decision of RTC Issue: Whether or not said cancellation had the effect of terminating also the liability of FIELDMEN'S as reinsurer with respect to policies or cessions issued prior to the termination of the principal reinsurance contracts or treaties. Ruling: NO. Of the six reinsurance contracts under consideration two contain provisions, which clearly and expressly recognize the continuing effectivity of policies ceded under them for reinsurance notwithstanding the cancellation of the contracts themselves. Insofar as the two reinsurance agreements with the express stipulations aforequoted are concerned there is clearly no merit in FIELDMEN'S claim that their cancellation carried with it ipso facto the termination of all reinsurance cessions thereunder. Such cessions continued to be in force until their respective dates of expiration. Since it was

EH 403 Page 61

⦿ 2010-2011 ⦿ 2011-2012 ⦿

INSURANCE LAW CASE DIGESTS
under one of said agreements, namely, the Facultative Obligatory Reinsurance Treaty-Fire, that the reinsurance cession corresponding to the GSIS policy had been made, FIELDMEN'S cannot avoid liability which arose by reason of the burning of the insured property. With respect to the other four agreements, it would seem that the petition for declaratory relief is moot, and that no useful purpose would be served by defining the respective rights and obligations of the parties thereunder. The said agreements have been cancelled, and it does not appear that any claim by or liability in favor of the insured has actually arisen under any of the reinsurance cessions made prior to such cancellation. WHEREFORE, the decision appealed from is affirmed insofar as it refers to the Facultative-Obligatory, Reinsurance Treaty and the Personal Accident Reinsurance Treaty are concerned, and modified with respect to the others by declaring the issues concerning them as moot and academic Digested by: Kristine Oja Equitable Ins. & Casualty Co. v. Rural Insurance Facts: On November 11, 1957, plaintiff(Equitable) and defendant (Rural Insurance) entered into a reciprocal facultative reinsurance agreement. Pursuant to said agreement, plaintiff reinsured for P2,000.00 with defendant the stock covered by fire insurance Policy No. 5880 and also for P2,000.00 the stock covered by fire insurance Policy No. 6026. Stocks covered by Insurance Policy Nos. 5880 and 6026 were subsequently burned and the share of the loss assumed by defendant as per reinsurance agreement was computed at P2,024.87 and P1,334.80 respectively. Notwithstanding repeated demands, defendant refused and failed to pay plaintiff, and that for defendant's failure to pay its share of the losses assumed by it, plaintiff has been compelled to institute an action in court. The defendant moved for the dismissal of the case contending that the complaint states no cause of action, the matter not having been referred to the decision of two arbitrators or umpire, which, it is claimed, is the condition precedent agreed upon in Article VIII of the Reinsurance Agreement entered into between the parties, to wit: . ARTICLE VIII In the event of any question arising as to the meaning of, or any way connected with or relating to this Agreement, whether before or after its termination, the parties shall endeavor to arrive at a satisfactory compromise by amicable settlement rather than by court action. The dispute shall be referred to the decision of two arbitrators, of whom one shall be appointed in writing by each of the parties within thirty (30) days after having been required so to do by the other party in writing, and in case of disagreement between the arbitrators, to the decision of the umpire to be appointed by them in writing before entering on the reference. Each party shall submit its case with all particulars within thirty days after their appointment. The seat of arbitration shall be in Manila, Philippines, and the expenses of arbitration shall be borne in equal proportion by the parties. The decision of the arbitrators or umpire, as the case may be, shall be final and binding on both the Company and the Reinsurer. The arbitrators and umpire shall not be bound by the strict rules of evidence and by judicial formalities in making the award. The court rendered its decision in favor of plaintiff, hence this appeal. Issue: 1. WON the trial court erred in failing to rule that plaintiff-appellee has no causes of action against it, the matter not having been referred to the decision of two arbitrators or umpire, which, it is claimed, is the condition precedent agreed upon in Article VIII of the Reinsurance Agreement 2. WON the trial court erred in failing to rule that in a facultative obligation the right to choose an alternative remedy lies only with the debtor, who in this case is the herein defendant-appellant", and in support thereof, cites Article 1206 of the new Civil Code. Held: As to the first issue, the court finds no merit in this contention. Under the abovequoted provision of the Reinsurance Agreement, it would seem clear that the requirement of submitting for decision to two arbitrators or an umpire the matter of losses by fire or the liability of the parties thereto arises only if and when the same is disputed by one of the parties. It does not appear in the instant case that appellant did dispute appellee's claims. Consequently, appellant may not invoke said provision in avoidance of its liability to appellee. It is true that paragraph (Article VIII) of said Reciprocal Facultative Reinsurance Agreement required that 'in the event of any question arising as to the meaning of, or any way connected with or relating to this Agreement, whether before or after its termination, the parties shall endeavor to arrive at a satisfactory compromise by amicable settlement rather than by court action'; and that the dispute should be referred to the decision of two arbitrators and umpire, as provided, therein. However, in this particular case, there is absolutely no dispute between the two parties, because in the stipulation of facts, the defendant has admitted that plaintiff has paid its liability to the insured as per its fire insurance policies specified in the

EH 403 Page 62

⦿ 2010-2011 ⦿ 2011-2012 ⦿

INSURANCE LAW CASE DIGESTS
two causes of action of the complaint. Defendant has, likewise, admitted its liability as reinsurer under the Reciprocal Facultative Reinsurance Agreement (Annex "A" to the complaint) to pay to the plaintiff its proportional shares, the amounts of which are not disputed. Indeed, according to the complaint as admitted by the defendant, statements of account as to the amounts of its share as reinsurer and, for all that appears, said defendant has never questioned the correctness of said amounts. It is, likewise, admitted by the defendant in the stipulation of facts, that because of its failure to pay said amounts, the plaintiff, on April 11, 1959, complained to the Assistant Insurance Commissioner, for official intervention, but said defendant has continued to ignore plaintiff's demands for reimbursement under the reinsurance policies. As to the second issue, the court finds no connection whatsoever between Article 1206 of the Civil Code and the agreement subject of this action, except the word "facultative" used in both. The term "facultative" is used in reinsurance contracts, and it is so used in this particular case, merely to define the right of the reinsurer to accept or not to accept participation in the risk insured. But once the share is accepted, as it was in the case at bar, the obligation is absolute and the liability assumed thereunder can be discharged by one and only way — payment of the share of the losses. There is no alternative nor substitute prestation. Judgment appealed from was affirmed, with costs against the defendant-appellant. Digested by: Gayle Opsima IVOR ROBERT DAYTON GIBSON, petitioner, vs. HON. PEDRO A. REVILLA, in his official capacity as Presiding Judge of Branch XII, Court of First Instance of Rizal, and LEPANTO CONSOLIDATED MINING COMPANY, respondents [G.R. No. L-41432 1979 Jul 30] Facts: Lepanto Consolidated Mining Company filed a complaint against Malayan Insurance Company, Inc.The civil suit thus instituted by Lepanto against Malayan was founded on the fact that Malayan issued a Marine Open Policy covering all shipments of copper, gold and silver concentrates in bulk from Poro, San Fernando, La Union to Tacoma, Washington or to other places in the United States. Thereafter, Malayan obtained reinsurance abroad through Sedgwick, Collins & Co., Limited, a London insurance brokerage. The Memorandum of Insurance issued by Sedgwick to Malayan listed three groups of underwriters or reinsurers – Lloyds 62.808%, Companies (I.L.U.) 34.705%, Other Companies 2.487%. At the top of the list of underwriting members of Lloyds is Syndicate No. 448, assuming 2.48% of the risk assumed by the reinsurer, which syndicate number petitioner Ivor Robert Dayton Gibson claims to be himself. Petitioner Ivor Robert Dayton Gibson filed a motion to intervene as defendant, which motion was denied by the lower court. Issue: whether the lower court committed, reversible error in refusing the intervention of petitioner Ivor Robert Dayton Gibson in the suit between Lepanto and Malayan Ruling: We rule that the respondent Judge committed no error of law in denying petitioner's Motion to Intervene. And neither has he abused his discretion in his denial of petitioner's Motion for Intervention. We agree with the holding of the respondent Court that since movant Ivor Robert Dayton Gibson appears to be only one of several re-insurers of the risks and liabilities assumed by Malayan Insurance Company, Inc., it is highly probable that other re-insurers may likewise intervene. If petitioner is allowed to intervene, We hold that there is good and sufficient basis for the Court a quo to declare that the trial between Lepanto and Malayan would be definitely disrupted and would certainly unduly delay the proceedings between the parties especially at the stage where Lepanto had already rested its case and that the issues would also be compounded as more parties and more matters will have to be litigated. In other words, the Court's discretion is justified and reasonable. We also hold that respondent Judge committed no reversible error in further sustaining the fourth ground of Lepanto's Opposition to the Motion to Intervene that the rights, if any, of petitioner are not prejudiced by the present suit and will be fully protected in a separate action against him and his co-insurers by Malayan. Petitioner's contention that he has to pay once Malayan is finally adjudged to pay Lepanto because of the very nature of a contract of reinsurance and considering that the re-insurer is obliged 'to pay as may be paid thereon' (referring to the original policies), although this is subject to other stipulations and conditions of the reinsurance contract, is without merit. The general rule in the law of reinsurance is that the re-insurer is entitled to avail itself of every defense which the re-insured (which is Malayan) might urge in an action by the person originally insured (which is Lepanto). Specifically, the rule is stated thus "Sec. 1238. In an action on a contract of reinsurance, as a general rule the reinsurer is entitled to avail itself of every defense which the reinsured might urge in an action by the person originally insured; . . ."

EH 403 Page 63

⦿ 2010-2011 ⦿ 2011-2012 ⦿

INSURANCE LAW CASE DIGESTS
The same rule is stated otherwise in 44 Am. Jur. 2d, Sec. 1862, p. 793, as follows: "Moreover, where an action is brought against the reinsurer by the reinsured, the former may assert any defense that the latter might have made in an action on the policy of original insurance." (Eagle Ins. Co. vs. Lafayette Ins. Co., 91 Ind. 443) As to the effect of the clause "to pay as may be paid thereon" contained in petitioner's re-insurance contract, Arnould, on the Law of Marine Insurance and Average, 13th Ed., Vol. 1, Section 327, p. 315, states the rule, thus: "It has been decided that this clause does not preclude the reinsurer from insisting upon proper proof that a loss strictly within the terms of the original policy has taken place." "This clause does not enable the original underwriter to recover from his re-insurer to an extent beyond the subscription of the latter." WHEREFORE, IN VIEW OF THE FOREGOING, the petition is hereby dismissed. No costs. Digested by: Jr Abul Artex Development Co VS Wellington Insurance FACTS: Wellington insurance insured for P24,346,509 the building stocks and machinery of plaintiff Artex against loss or damage by fire or lightning upon august 2, 1963 with an additional sum of P833,034. Another insurance against business interruption (use and occupancy)for P5,200,000. On September 22, 1963 the building, and machineries were burned and a notice of loss and damage was given to Wellington. Insurance adjusters computed the loss for the fire as P10,106,544.40 and Wellington paid only 6,481,870.07, leaving a balance of 3,624,683.43 The computed business interruption loss was P3M but Wellington paid only P1,864,134.08 leaving a balance of P1,748,460 (computation based on case) Artex through counsel Norberto Quisumbing made a manifestation that only about P397,ooo is the remaining balance and liability which was the subject of reinsurance with Alexander and Alexander Inc, of New York, Artex acknowledging here the receipt of P3,600,000 as FINAL and FULL SETTLEMENT of all claims against Welllington Artex further prays to the court to affirm the lower court’s decision of liquidation and prayed for modification of the amount of liability to be fixed to P397,813.00 plus 12% interest per annum thereof for the late payment until april 10, 1969 and attorney’s fees of 15% of the recovery, expenses of litigation, no writ of execution however to be made within 3years from july 10, 1969 per collateral agreement of the parties. Wellington in its brief raises the issue that Artex deemed to have agreed to look SOLELY to the reinsurers for indemnity in case of loss since their paid up capital stock is only P500,000 and that they have to secure such reinsurance coverage the over P24M fire insurance coverage of the policy issued by Wellington to Artex. Issue: WON reinsurance contract of the parties makes the insured to look SOLELY to the reinsurers for indemnity in case of loss Ruling: NO, the insured who is not directly a party or privy to the reinsurance contract between Wellington and Alexander and Alexander Inc., cannot demand enforcement of such insurance contracts. The Contracts take effect only between the parties, their assigns and heirs as provide by Art 1311 of our civil code. Further it provides that a contract with stipulations pour autrui or in favor of a third person not a party to the contract, the parties must have CLEARLY and DELIBERATELY conferred favor upon a third person. The SC also stated that assuming that Artex directly sue the reinsurers for payment this does not in any way affect or cancel out Wellington’s direct contractual liability to Artex. The SC dispose the case by affirming the prayer of Artex. Digested by: Jr Abul Avon Insurance vs CA FACTS: It all started with Yupangco Cotton Mills engaged to secure with Worldwide Security and Insurance Co. Inc., several of its properties totaling P200 Million These contracts were covered by reinsurance treaties between Worldwide Surety and Insurance, and several foreign reinsurance companies including the petitioners through CJ Boatrwright acting as agent of Worldwide Surety and Insurance A Fire then razed the properties insured on December 1969 and May 2, 1981 A Deed of Assignment made by Worldwide Surety and Insurance acknowledged a remaining balance of P19,444,447.75 still due and assigned to Yupangco all reinsurance proceeds still collectible from all the foreign reinsurance companies. Yupangco then filed a collection suit on the above petitioners

EH 403 Page 64

⦿ 2010-2011 ⦿ 2011-2012 ⦿

INSURANCE LAW CASE DIGESTS
The service of summons were made through the office of the Insurance Commissioner but since the international reinsurers question the jurisdiction the trial court the case has not proceeded to trial on the merits The reinsurer is questioning also the service of summons through extraterritorial service under Sect 17 Rule 14 of the Rules of Court nor through the Insurance Commissioner under Sec 14 Yupangco also contends that since the reinsurers question the jurisdiction of the court they are deemed to have submitted to the jurisdiction of the court. ISSUE: WON the international reinsurers are “doing business in the Philippines”. WON the Philippine court has jurisdiction over these international reinsurers who are not doing business in the Philippines RULING: NO, international reinsurers are not “doing business in the Philippines” and the Philippine court has not acquired jurisdiction over them. The reinsurance treaties between the petitioners and Worldwide Surety and Insurance were made through an international insurance broker and NOT through any entity or means remotely connected with the Philippines Reinsurance company is not doing business in a certain state even if the property or lives which are insured by the original insurer company are located in that state. Reinsurance Contract is generally separate and distinct arrangement from the original contract of insurance. Doing business in the Philippines – must be judged in the light of its peculiar circumstances upon its peculiar facts and upon the language of the statute applicable. o True test: whether the foreign corporation is continuing the body or substance of the business or enterprise for which it was organized If there exist a domestic agent of the foreign corporation it can be served with summons through that agent without proving that such corporation is doing business in the phils or not. o NO allegation or demonstration of the existence of petitioners’ domestic agent but avers simply that they are doing business not only abroad but in the Phils o Petitioners had not performed any act which would give the general public the impression that it had been engaging or intends to engage in its ordinary and usual business undertaking in the country.

-

The purpose of the law in requiring that foreign corporations doing business in the country be licensed to do so, is to subject the foreign corporations doing business in the Philippines to the jurisdiction of the courts, 19 otherwise, a foreign corporation illegally doing business here because of its refusal or neglect to obtain the required license and authority to do business may successfully though unfairly plead such neglect or illegal act so as to avoid service and thereby impugn the jurisdiction of the local courts. Voluntary appearance before the lower court to question the jurisdiction is not equivalent to submission to jurisdiction The SC disposed the case in favor of the international insurers (petitioners’) declaring that the lower court has not acquired and cannot acquire jurisdiction over them and was ordered to desist from maintaining further proceeding against them. III. CLASSES OF INSURANCE A. Marine Insurance (Sections 99166) i. Definition (Section 99) ii. Insurable Interest (Sections 100-106) iii. Concealment (Sections 107-110) iv. Representations (Sections 111-112) v. Implied Warranties (Sections 113-120) vi. The Voyage and Deviation (Sections 121126) vii. Loss (Sections 127-137) viii. Abandonment (Sections 138-155) ix. Measure of Indemnity (Sections 156-166) B. Fire Insurance (Sections 167-173) C. Casualty Insurance (Section 174) i. Compulsory Motor Vehicle Liability Insurance D. Suretyship (Sections 175-178) E. Life Insurance (Sections 179-183)

EH 403 Page 65

⦿ 2010-2011 ⦿ 2011-2012 ⦿

Sponsor Documents

Or use your account on DocShare.tips

Hide

Forgot your password?

Or register your new account on DocShare.tips

Hide

Lost your password? Please enter your email address. You will receive a link to create a new password.

Back to log-in

Close